Sie sind auf Seite 1von 183

Panimalar Engineering College Downloaded From : www.EasyEngineering.

net Umar Ali

1.Partial Differential Equation

An equation involving partial derivatives of two or more independent variables is called a Partial differential equa-
tion(PDE)

Formation of PDE

PDE are formed in two ways

• by eliminating arbitrary constants.

• by eliminating arbitrary functions.

Notations:
If z = f (x, y) then

ne t
n g .
i
∂z ∂z
p= ,q =

r=
2
∂ z
∂x

,s =
2
∂ z
∂y

,t =
∂ z

e
2
e r
∂x2

i n
∂x∂y ∂y 2

ng
Formation of PDE by elimination of arbitrary constants and arbitrary functions

E
as y
1. If no.of arbitrary constants ≤ no.of independent variable then use p,q only.

. E
2. If no.of arbitrary constants > no.of independent variable then use p,q,r,s and t.

w
w w Form a PDE by eliminating arbitrary constants

1. z = a(x + y) + b
Solution:
Given z = a(x + y) + b...........(1) no.of a.c=no.of I.V
Differentiate (1) par. w.r.t x and y,
∂z ∂z
p= = a.......(2), q= = a........(3)
∂x ∂y
From (2) & (3)⇒ p = q which is the required p.d.e.

2. z = a2 x + ay 2 + b
Solution:
Given z = a2 x + ay 2 + b...........(1) no.of a.c=no.of I.V

Page 1

Downloaded From : www.EasyEngineering.net


Panimalar Engineering College Downloaded From : www.EasyEngineering.net Umar Ali

Differentiate (1) par. w.r.t x and y,

∂z ∂z
p= = a2 .......(2) q = = 2ay
∂x ∂y
q
⇒y =
2a
q2
y2 =
4a2
q2
y2 = using (2)
4p

⇒ 4py 2 = q 2 which is the required p.d.e.

3. z = axn + by n
Solution:
Given z = axn + by n ...........(1) no.of a.c=no.of I.V
Differentiate (1) par. w.r.t x and y,
p=
∂z
∂x
= naxn−1 , q=
∂z
∂y
= nby n−1

ne t
.
p q
a= , b=
Sub in (1)
nxn−1

p q
ny n−1

i n g
z=
nxn−1
xn + n−1 y n
ny
⇒ nz = px + qy which is the required p.d.e.

e e r
4. z = ax2 + by 2
i n
ng
Ans: 2z = px + qy

5. z = ax3 + by 3

E
Ans: 3z = px + qy

6. z = (x2 + a)(y 2 + b)

as y
E
Solution:

w .
Given z = (x2 + a)(y 2 + b)...........(1)
Differentiate (1) par. w.r.t x and y,
no.of a.c=no.of I.V

w w p=
∂z
∂x
= 2x(y 2 + b),

y2 + b =
p
2x
,
q=
∂z
∂y
x2 + a =
= 2y(x2 + a)
q
2y
Sub in (1)
q p
z=
2y 2x
⇒ 4xyz = pq which is the required p.d.e.

7. z = (x2 + a2 )(y 2 + b2 ) Ans: 4xyz = pq

8. (x − a)2 + (y − b)2 + z 2 = r2
Solution:
Given (x − a)2 + (y − b)2 + z 2 = r2 .......(1) no.of a.c=no.of I.V
Differentiate (1) par. w.r.t x and y,
∂z ∂z
2(x − a) + 2z = 0, 2(y − b) + 2z =0
∂x ∂y
x − a = −zp , y − b = −zq

Page 2

Downloaded From : www.EasyEngineering.net


Panimalar Engineering College Downloaded From : www.EasyEngineering.net Umar Ali

Sub in (1)
(−zp)2 + (−zq)2 + z 2 = r2
z 2 p2 + z 2 q 2 + z 2 = r2
⇒ z 2 (p2 + q 2 + 1) = r2 which is the required p.d.e.

9. (x − a)2 + (y − b)2 + z 2 = 1 Ans: z 2 (p2 + q 2 + 1) = 1

10. z = (x − a)2 (y − b)2 Ans: 4xyz = pq

11. z = (x − a)2 + (y − b)2 Ans:p2 + q 2 = 4z

12. z = (x + a)3 + (y − b)3


Solution:
Given z = (x + a)3 + (y − b)3 ...........(1) no.of a.c=no.of I.V
Differentiate (1) par. w.r.t x and y,

t
∂z ∂z
p= = 3(x + a)2 , q= = 3(y − b)2
∂x
(x + a)2 = ,
p
 p 312
∂y
(y − b)2 =
q
31
. ne
g
q 2
x+a= , y−b=

(x + a)3 =
3  3
p 2
,
ri n
(y − b)3 =
3  3
q 2

Sub in (1)
3
 p  32  q  32

n e e 3

i
⇒z= + which is the required p.d.e.
3 3

ng
 p  32  q 2
3 2
13. z = (x + a) + (y − b) Ans: z = +

y E 3 2

as
14. (x − a)2 + (y − b)2 = z 2 cot2 α
Solution:

w . E
Given (x − a)2 + (y − b)2 = z 2 cot2 α...........(1)
Differentiate (1) par. w.r.t x and y,
no.of a.c=no.of I.V

w w
Sub in (1)
2(x − a) = 2zp cot2 α,
(x − a) = zp cot2 α ,
2(y − b) = 2zq cot2 α
(y − b) = zq cot2 α

[zp cot2 α]2 + [zq cot2 α]2 = z 2 cot2 α


z 2 cot4 α(p2 + q 2 ) = z 2 cot2 α
⇒ p2 + q 2 = tan2 α which is the required p.d.e.

15. Find the PDE of the family of spheres having centres on the z-axis.
Solution:
Given that the sphere lies on the z-axis.
Centre is (0, 0, c). Let Radius = r.
∴ Eqn of the sphere
x2 + y 2 + (z − c)2 = r2 .........(1) no.of a.c=no.of I.V
Differentiate (1) par. w.r.t x and y,

Page 3

Downloaded From : www.EasyEngineering.net


Panimalar Engineering College Downloaded From : www.EasyEngineering.net Umar Ali

∂z ∂z
2x + 2(z − c) = 0, 2y + 2(z − c) =0
∂x ∂y
x + (z − c)p = 0, y + (z − c)q = 0
(z − c)p = −x.........(2), (z − c)q = −y.........(3)
Eqn(2) p −x
⇒ =
Eqn(3) q −y
⇒ py = xq which is the required p.d.e.

16. Find the PDE of all plane having equal intercept on the x and y axis.
Solution:
x y z
Intercept form of the plane eqn is + + =1
a b c
Given :a = b
x y z
⇒ + + = 1.........(1) no.of a.c=no.of I.V
a a c
Differentiate (1) par. w.r.t x and y,
1 1 ∂z 1 1 ∂z
+0+ = 0, 0+ + =0
a c ∂x a c ∂y
1 1
a c
1
+ p=0,
1
1
a
1
1
+ q=0
c
1

ne t
From (2) and (3)
a
= − p........(2)
c a
= − q........(3)

n
c

g .
i
1 1
− p=− q
c c
⇒ p = q which is the required p.d.e.

e e r
i n
17. Form the PDE of the family of sphere having their centres on the line x = y = z

ng
Hint:(x − a)2 + (y − a)2 + (z − a)2 = r2 Ans : (y − z)p + (z − x)q = x − y

y E
as
Formation of PDE by eliminating arbitrary function:

. E
Form the PDE by eliminating the arbitrary functions

w
w
 
x
1. z = f

w
y
Solution:
 
x
Given z = f
y
Diff par w.r.t. x and y,       
∂z x 1 ∂z x −x
= f0 , = f0 2
∂x   y y ∂y  y  y
x 1 x −x
p = f0 .......(1) q = f0 .......(2)
y y y y2
1
Eqn(1) p y
⇒ = −x
Eqn(2) q
2
 2 y
p 1 −y
=
q y x
p −y
=
q x
⇒ px = −qy
∴ px + qy = 0 which is the required p.d.e.

Page 4

Downloaded From : www.EasyEngineering.net


Panimalar Engineering College Downloaded From : www.EasyEngineering.net Umar Ali

y
2. z = f Ans : xp + yq = 0
x

3. z = f (xy) Ans : xp = yq

4. z = f (x2 + y 2 ) Solution:
Given z = f (x2 + y 2 )
Diff par w.r.t. x & y,
∂z ∂z
p= = f 0 (x2 + y 2 )2x.......(1), q= = f 0 (x2 + y 2 )2y........(2)
∂x ∂y
Eqn(1) p 2x
⇒ =
Eqn(2) q 2y
∴ py = qx which is the required p.d.e.
x
5. z 2 − xy = f
z
Solution:
 x
Let ϕ z 2 − xy, =0
z

t
2 x
where u = z − xy and v =
z
Hence the required PDE is of the form P p + Qq = R

. ne
Where

i n g
P
∂u ∂v ∂u ∂v
=
∂y ∂z

 x
∂z ∂y

e e r
iz
n
= (−x) − 2 − 2z(0)

ng
x2
P =
z2

y E Q =
∂u ∂v
∂z ∂x

∂u ∂v
 ∂x ∂z 

as
1 x
= (2z) − (−y) − 2
z z

w . E Q = 2− 2

R =
∂u ∂v
xy
z

∂u ∂v

w w ∂x ∂y ∂y ∂x

= (−y)(0) − (−x)

R =
x
 
1
z

The required equation is

x2
   xy  x
2
p+ 2− 2 q =
z z z
 2  2 
x 2z − xy x
p + q =
z2 z2 z
x2 p + (2z 2 − xy)q = xz which is the required p.d.e.

6. ϕ(x2 − y 2 , z) = 0 Ans: yp + xq = 0

7. ϕ(x2 + y 2 + z 2 , ax + by + cz) = 0 Ans: (bz − cy)p + (cx − az)q = ay − bx

Page 5

Downloaded From : www.EasyEngineering.net


Panimalar Engineering College Downloaded From : www.EasyEngineering.net Umar Ali

8. ϕ(x2 + y 2 + z 2 , x + y + z) = 0 Ans: (z − y)p + (x − z)q = y − x

Two Functions[use p,q,r,s,t]

9. z = g(y + x) + xf (y + x)
Solution:
Given z = xf (y + x) + g(y + x).....(1)
Diff par w.r.t. x & y,

p = f (y + x) + xf 0 (y + x) + g 0 (y + x).....(2)

q = xf 0 (y + x) + g 0 (y + x).....(3)

Again diff (2) and (3) w.r.t x and y,

r = f 0 (y + x) + xf 00 (y + x) + f 0 (y + x) + g 00 (y + x)

ne t
t = xf 00 (y + x) + g 00 (y + x)

n g .
Now r + t =
s =

e ri
f 0 (y + x) + xf 00 (y + x) + g 00 (y + x)

2[f 0 (y + x) + xf 00 (y + x) + g 00 (y + x)]

r+t = 2s

i n e
10. z = f (x + ct) + g(x − ct)

E ng
Solution:
Given z = f (x + ct) + g(x − ct)

as y
. E
Diff par w.r.t x & t,

w ∂z

w w ∂x
∂z
∂t
=

=
f 0 (x + ct) + g 0 (x − ct)

f 0 (x + ct)c + g 0 (x − ct)(−c)

Again diff par w.r.t x & t,

∂2z
= f 00 (x + ct) + g 00 (x − ct)
∂x2
∂2z
= f 00 (x + ct)c2 + g 00 (x − ct)(−c)2
∂t2
= c2 [f 00 (x + ct) + g 00 (x − ct)]
∂2z ∂2z
= c2
∂t2 ∂x2

11. z = x2 f (y) + y 2 g(x)


Solution:
Given z = x2 f (y) + y 2 g(x)......(1)

Page 6

Downloaded From : www.EasyEngineering.net


Panimalar Engineering College Downloaded From : www.EasyEngineering.net Umar Ali

Diff par w.r.t. x & y,

p = 2xf (y) + y 2 g 0 (x).....(2)

q = x2 f 0 (y) + 2yg(x).....(3)

Again diff (2) and (3) w.r.t x and y,

r = 2f (y) + y 2 g 00 (x).....(4)

t = x2 f 00 (y) + 2g(y).....(5)

s = 2xf 0 (y) + 2yg 0 (x).....(6)

(2) × x ⇒ px = 2x2 f (y) + xy 2 g 0 (x)

(3) × y ⇒ qy = x2 yf 0 (y) + 2y 2 g(x)

ne t
px + qy = 2[x2 f (y) + y 2 g(x)] + xy[xf 0 (y) + yg 0 (x)]

= 2z + xy
hsi

n g .
2(px + qy) = 4z + xys
2

e ri
12. z = f (x3 + 2y) + g(x3 − 2y)
i n e
Ans: 4xr = 9x5 t + 8p

E ng
TYPES OF SOLUTION

as y
. E
Complete solution (or) Complete Integral

w
A solution in which the number of arbitrary constants is equal to the number of independent variables.

w w
Particular Integral
In complete integral if we give particular values to the arbitrary constants.
Singular Integral
Let f (x, y, z, p, q) = 0 be a PDE whose complete integration is φ(x, y, z, a, b) = 0.....(1)
Diff (1) partially w.r.t a and b and then equal to zero, we get

∂φ
= 0.....(2)
∂a
∂φ
= 0.....(3)
∂b

Eliminating a and b by using eqn(1),(2) and (3).


The eliminant of a and b is called Singular Integral.

Page 7

Downloaded From : www.EasyEngineering.net


Panimalar Engineering College Downloaded From : www.EasyEngineering.net Umar Ali

Type 1: Form F (p, q) = 0

There is no Singular Integral for Type 1

√ √
1. Find the Complete integral of p+ q=1
Solution:
√ √
Given p+ q = 1.....(1)
This is type 1,
The complete integral is z = ax + by + c
put p = a and q = b in (1)

√ √
a+ b = 1
√ √
b = 1− a

b =
√ 2
(1 − a)

ne t
⇒ the complete integral is z = ax + (1 −

a)2 y + c,
g .
where a and c are arbitray constants.

n
2. Find the Complete integral of p + q = pq

e ri
Solution:
Given p + q = pq.....(1)
i n e
This is type 1,

E
The complete integral is z = ax + by + c ng
put p = a and q = b in (1)

as y
w . E a+b

b
= ab

= ab − a

w w b = a(b − 1)

⇒ the complete integral is z = ax + a(b − 1)y + c, where a and c are arbitray constants.

3. Find the Complete integral of p2 + q 2 = npq


Solution:
Given p2 + q 2 = npq.....(1)
This is type 1,
The complete integral is z = ax + by + c
put p = a and q = b in (1)

a2 + b2 = nab

b2 − nab + a2 = 0

Page 8

Downloaded From : www.EasyEngineering.net


Panimalar Engineering College Downloaded From : www.EasyEngineering.net Umar Ali


na ± n2 a2 − 4a2
⇒b =
√2
na ± a n2 − 4
=
2
√ !
n± n2 − 4
⇒ the complete integral is z = ax + ay + c, where a and c are arbitray constants.
2

Type 2: Form z = px + qy + f (p, q)[Clairaut’s Form]

1. Solve the equation z = px + qy + p2 − q 2


Solution:
Given z = px + qy + p2 − q 2
This is type 2[Clairaut’s Form],
put p = a and q = b in (1)
∴ The complete integral is z = ax + by + a2 − b2 .....(1)

ne t
To find Singular Integral
Diff (1) par w.r.t a and b, we get

n g .
0 = x + 2a ⇒ a = −
e r
x
i
0 =
n
y − 2b ⇒ b =

i e
y
2
2

substituting in (1), we get

E ng
as
z
y x
2
y
= − x+ y+
2

−x
2
2  

y 2
2

w . E x
= − +
2
x2
= − +
2
y
2
y2
2
+
x
4
2

y
4
2

w w 4z
4 4
= y 2 − x2 , which is the singular solution

2. solve z = px + qy + p2 q 2
Solution:
Given z = px + qy + p2 q 2
This is type 2[Clairaut’s Form],
put p = a and q = b in (1)
∴ The complete integral is z = ax + by + a2 b2 ..........(1)
To find Singular Integral
Diff (1) par w.r.t a and b, we get

∂z
= x + 2ab2 = 0
∂a

Page 9

Downloaded From : www.EasyEngineering.net


Panimalar Engineering College Downloaded From : www.EasyEngineering.net Umar Ali

∂z
= y + 2ba2 = 0
∂b
⇒x = −2ab2 ..........(2)

y = −2ba2 ..........(3)
x
(2) ⇒ a = − ..........(4)
2b2

Substitute (4) in (3)

 x 2
y = −2b − 2
 2b
x2

= −2b
4b4
2
x
= − 3
2b
x2
b3 = −
2y

b =
 2  13

x
2y
..........(5)

ne t
Substitute (5) in (4)

n g .
= − 
x

e ri
e
a 2
x2 3
2 −

i
2y
n
ng
x
= − 4
!
x3

E
2 2 2
23 y 3

a s y = −
2
x × 23 y 3
2x 3
4
2

.E
1
y2 3

= − ..........(6)

w
2x

w w
Substitute (5) and (6) in (1)

z = −x

y2
−y
 13 
+
x2
 13
2  2
y 2 3 x2 3


2x 2y
2x 2y
2
! 2
! 
2
2
y3 x3 y x2 3
= −x 1 1 −y 1 1 + ×
23 x3 23 y3 2x 2y
2 2
! 2 2
! 2 2
!
x3 y3 x3 y3 x3 y3
= − 1 − 1 + 2
23 23 43
27
z3 = − x2 y 2 , which is the singular solution
16

p
3. Find the singular solution of z = px + qy + 1 + p2 + q 2
Solution:
p
Given z = px + qy + 1 + p2 + q 2
This is type 2[Clairaut’s Form],

Page 10

Downloaded From : www.EasyEngineering.net


Panimalar Engineering College Downloaded From : www.EasyEngineering.net Umar Ali

put p = a and q = b in (1)



∴ The complete integral is z = ax + by + 1 + a2 + b2 ..........(1)
To find Singular Integral
Diff (1) par w.r.t a and b, we get

∂z 1
=x+ √ (2a) = 0
∂a 2 1 + a2 + b2
a
x+ √ = 0
1 + a2 + b2
a
x = −√ ..........(2)
1 + a2 + b2
∂z 1
=x+ √ (2b) = 0
∂b 2 1 + a2 + b2
b
y+ √ = 0
1 + a2 + b2
b
y = −√ ..........(3)
1 + a2 + b2

⇒ x2 =
a2
1 + a2 + b2

ne t
y 2
=
b2
1 + a2 + b2

n g .
Now x2 + y 2 =

e ri a2 + b2
1 + a2 + b2

e
2 2 a2 + b2
1 − (x + y ) = 1−

i n 1 + a 2 + b2
1 + a2 + b2 − a2 − b2

ng
1 − x2 − y 2 =
1 + a2 + b2
1

y E =
1 + a2 + b2
1

as
1 + a2 + b2 =
1 − x2 − y 2
1

E
p
1 + a2 + b2

.
= p .........(4)
1 − x2 − y 2

w w
From (2) and (3)

a

b

w
= −x = −y
1 + a2 + b2 1 + a2 + b2
√ √
a = −x 1 + a2 + b2 b = (−y) 1 + a2 + b2
−x −x
a= p a= p by (4)
1 − x2 − y 2 1 − x2 − y 2
substituting in (1)

−x2 y2 1
z = p −p +p
1 − x2 − y 2 1 − x2 − y 2 1 − x2 − y 2
1 − x2 − y 2
= p
1 − x2 − y 2
p
z = 1 − x2 − y 2

z2 = 1 − x2 − y 2

∴ x2 + y 2 + z 2 = 1which is the singular solution.

Page 11

Downloaded From : www.EasyEngineering.net


Panimalar Engineering College Downloaded From : www.EasyEngineering.net Umar Ali


4. solve z = px + qy + 2 pq Ans: xy = 1

p y
5. solve z = px + qy + −p Ans: z =
q 1−x

6. solve z = px + qy + p2 − q 2 Ans: y 2 − x2 = 4z

1
7. Find the complete integral of (z − px − qy)(p + q) = 1 Ans: z = ax + by +
a+b

Type 3: Form f (p, q, z) = 0 or f (p, q, x) = 0 or f (p, q, y) = 0

1. Solve p(1 + q) = qz
Solution:
Given p(1 + q) = qz..........(1)
This is type 3,[f (p, q, z) = 0]

substitute in (1)
Put q = ap

ne t
n g .
p(1 + ap) =apz

1 + ap =az
e ri
n
ap =az − 1

i e
ng
az − 1
p= q = ap = az − 1
a

E
We have dz = pdx + qdy,

y
as
az − 1
Z Z Z
dz = dx + (az − 1)dy
a

E
Z Z Z
dz 1

w . 1
Z
az − 1
adz
=

=
1
Z
a
dx + dy
Z
dx + dy

w w 1
a
a az − 1 a
1
log(az − 1) = x + y + c1
a
log(az − 1) =x + ay + c..........(2)

which is a complete integral, where a and c are constants.


To find the Singular Integral
Diff (2) par w.r.t a and c
when diff par w.r.t c, we get 0 = 1 which is not true.
So there is no Singular integral.

To find the General integral


Put c = φ(a) in (2)
⇒ log(az − 1) = x + ay + φ(a)..........(3)
Diff (3) w.r.t a and Eliminating a, we get the General Integral

Page 12

Downloaded From : www.EasyEngineering.net


Panimalar Engineering College Downloaded From : www.EasyEngineering.net Umar Ali

2. Solve z 2 (p2 + q 2 + 1) = 1
Solution:
Given z 2 (p2 + q 2 + 1) = 1..........(1)
This is type 3,[f (p, q, z) = 0] Put q = ap
substitute in (1)

z 2 (p2 + a2 p2 + 1) = 1

p2 z 2 (a2 + 1) + z 2 = 1
1 − z2
p2 =
z 2 a2 + 1
√ √
1 − z2 1 − z2
p = p q = ap = a p
z (a2 + 1)
2 z (a2 + 1)
2

We have dz = pdx + qdy

ne t
Z
dz =
Z √

1 − z2
z a2 + 1
Z
dx + a √

1 − z2
z a2 + 1
dy

n g .
i
Z Z Z
z 1 1

1 − z2
dz = √
2
a +1
dx + a √
2
a +1

e e r
dy.....(i)

put 1 − z 2

g i n = t2

En
−2zdz = 2tdt

a s y −zdz = tdt

sub in (i)

w . E Z
−tdt 1
Z
1
Z

w w p p
p
t
1 + a2 (−t) = x + ay + c
= √
a2 + 1
dx + a √
a2 + 1
dy.....(i)

− 1 + a2 1 − z 2 = x + ay + c...........(2)

which is a complete integral, where a and c are constants.

To find the Singular Integral


Diff (2) par w.r.t a and c
when diff par w.r.t c, we get 0 = 1 which is not true.
So there is no Singular integral.

To find the General integral


Put c = φ(a) in (2)
√ √
⇒ − 1 + a2 1 − z 2 = x + ay + φ(a)..........(3)

Page 13

Downloaded From : www.EasyEngineering.net


Panimalar Engineering College Downloaded From : www.EasyEngineering.net Umar Ali

Diff (3) w.r.t a and Eliminating a, we get the General Integral.

3. p(1 − q 2 ) = q(1 − z)
Solution:
Given p(1 − q 2 ) = q(1 − z) ..........(1)
This is type 3,[f (p, q, z) = 0]
Put q = ap
substitute in (1)

p(1 − a2 p2 ) = ap(1 − z)

1 − a2 p2 = a − az

a2 p2 = 1 − a + az
1 − a + az
p2 = 2
√ a
p =
1 − a + az
a

ne t
We have dz = pdx + qdy

n g .
Z Z √
1 − a + az
Z √

e ri
1 − a + az
Z

dz
dz =

=
1
Z

i n
a
Z
dx + a

dx + dy..........(2) e a
dy

ng
1 − a + az a

s y E put 1 − a + az = t2

.E a adz = 2tdt

w
sub in (2)

w
w
Z Z Z
2tdt 1
= dx + dy
at a
2t 1
= x+y+c
a a

2 1 − a + az = x + ay + c..........(3)

which is a complete integral, where a and c are constants.

To find the Singular Integral


Diff (2) par w.r.t a and c
when diff par w.r.t c, we get 0 = 1 which is not true.
So there is no Singular integral.

To find the General integral


Put c = φ(a) in (2)

Page 14

Downloaded From : www.EasyEngineering.net


Panimalar Engineering College Downloaded From : www.EasyEngineering.net Umar Ali


⇒ 2 1 − a + az = x + ay + φ(a)..........(3)
Diff (3) w.r.t a and Eliminating a, we get the General Integral.
p
4. p(1 + q 2 ) = q(z − a) Ans: 2 bz − (ab + 1) = x + by + c

3
5. Solve 9(p2 z + q 2 ) = 4 Ans: (z + a2 ) 2 = x + ay + c

1
6. Solve z 2 = 1 + p2 + q 2 Ans: cos h−1 z = √ (x + ay) + c
1 + a2

Type 4: Method of Separable[Form: f1 (x, p) = f2 (y, q)]

1. Solve p2 y(1 + x2 ) = qx2

t
Solution:
Given p2 y(1 + x2 ) = qx2 .....(1)
This is type 4,[f1 (x, p) = f2 (y, q)]
. ne
p2 (1 + x2 )
x2
q
= =a
y

i n g
p2 (1 + x2 )
x2
ax 2
=a

e e
q
yr=a

p2 =
1+ √x2
i n q = ya

ng
x a
p= √ q = ya
1 + x2

Z
y E
We have dz

s
= pdx + qdy
Z √
x a
Z

.E a dz

put 1 + x2
=

= t2

1 + x2
dx + yady...........(1)

w w 2xdx

sub in (1)
= 2tdt ⇒ xdx = tdt

w Z
dz =

z =


a
Z
tdt
t
y2
at + a + c
Z
+ a ydy

2
√ p y2
z = a 1 + x2 + a + c
2
2
p ay
z = a(1 + x2 ) + + c...........(2)
2

which is a complete integral, where a and c are constants

To find the Singular Integral


Diff (2) par w.r.t a and c
when diff par w.r.t c, we get 0 = 1 which is not true.
So there is no Singular integral.

Page 15

Downloaded From : www.EasyEngineering.net


Panimalar Engineering College Downloaded From : www.EasyEngineering.net Umar Ali

To find the General integral


Put c = φ(a) in (2)
p ay 2
⇒ z = a(1 + x2 ) + + φ(a).........(3)
2
Diff (3) w.r.t a and Eliminating a, we get the General Integral.

2. Solve p2 + q 2 = x + y
Solution:
Given p2 + q 2 = x + y.....(1)
This is type 4,[f1 (x, p) = f2 (y, q)]
p2 − x = y − q 2 = a

p2 − x = a y − q2 = a

p2 = a + x

p= a+x
q2 = y − a

q = y−a

ne t
n g .
We have dz = pdx + qdy

e ri
√ √

e
Z Z Z
dz = x + adx + y − ady

z =
g i n
3
(x + a) 2
+
3
(y − a) 2
+c

En
3 3
2 2
2 3 2 3
z = (x + a) 2 + (y − a) 2 + c.....(2)

a s y 3

which is a complete integral, where a and c are constants


3

. E
To find the Singular Integral

w
w
Diff (2) par w.r.t a and c

wwhen diff par w.r.t c, we get 0 = 1 which is not true.


So there is no Singular integral.

To find the General integral


Put c = φ(a) in (2)
2 3 2 3
⇒ z = (x + a) 2 + (y − a) 2 + φ(a).....(3)
3 3
Diff (3) w.r.t a and Eliminating a, we get the General Integral.

3. Solve q = 2px
Solution:
Given q = 2px.....(1)
This is type 4,[f1 (x, p) = f2 (y, q)]
q = 2px = a

Page 16

Downloaded From : www.EasyEngineering.net


Panimalar Engineering College Downloaded From : www.EasyEngineering.net Umar Ali

2px = a y=a
a
p= q=a
2x

We have dz = pdx + qdy


Z Z Z
a
dz = dx + ady
2x
a
z = log x + ay + c.....(2)
2

which is a complete integral, where a and c are constants

To find the Singular Integral


Diff (2) par w.r.t a and c
when diff par w.r.t c, we get 0 = 1 which is not true.
So there is no Singular integral.

To find the General integral


Put c = φ(a) in (2)

ne t
a
⇒ z = log x + ay + φ(a).....(3)
2

n g .
Diff (3) w.r.t a and Eliminating a, we get the General Integral.

e ri x2 1 y2
4. Find the complete integral of pq = xy

i n e Ans:z = a
2
+
a 2
+c

5. Find the complete integral of p + q = x + y

n g Ans:z =
(x + a)2
2
+
(y − a)2
2
+c

Hint:
R√
x2 + a2 dx =
x√ 2
x + a2 +
sy E
6. Find the complete integral of p2 + q 2 = x2 + y 2
a2
sin h−1 xa
R√
x2 − a2 dx =
x√ 2
2
.E
2

a
x − a2 −
a2
2
2
cos h−1 xa

w w Ans:z =
x√ 2
2
x + a2 +
a2
2
sin h−1 xa +
yp 2
2
y − a2 −
a2
2
cos h−1 ay + c

w
Solving second and higher order with constant coefficients homogeneous and non ho-
mogeneous differential equation

In this chapter z will always represent a function of x and y, i.e z = f (x, y)

Notations:
∂ ∂ ∂z ∂z ∂2z ∂2z ∂2z
D= , D0 = ,p = ,q = ,r = , s = &t =
∂x ∂y ∂x ∂y ∂x2 ∂x∂y ∂y 2

Definition:
A linear p.d.e with constant coefficient in which all the partial derivative are of the same order is called
homogeneous, otherwise it is called non-homogeneous

Page 17

Downloaded From : www.EasyEngineering.net


Panimalar Engineering College Downloaded From : www.EasyEngineering.net Umar Ali

(HOMOGENEOUS)Form: f (D, D0 )z = F (x, y)


The solution is z = C.F + P.I
To find C.F
Put D = m and D0 = 1

ne t
n g .
e ri
i n e
E ng
as y
w . E
w w

Page 18

Downloaded From : www.EasyEngineering.net


Roots C.F
real(imaginary) and Different

w
m1 6= m2 f1 (y + m1 x) + f2 (y + m2 x)
real(imaginary) and same

w
m1 = m2 = m f1 (y + mx) + xf2 (y + mx)

w .
To find P.I
Panimalar Engineering College

E
Form P.I Substitution
1

as
f (D, D0 ) = eax+by eax+by D → a,D0 → b
f (D, D0 )

y
1
= eax+by
f (a, b)

E
1
f (D, D0 ) = sin(ax + by) or cos(ax + by) sin(ax + by) or cos(ax + by) D2 → −a2 ,D02 → −b2 ,DD0 → −ab
f (D, D0 )
1

ng
f (D, D0 ) = xm y n xm y n Binomial theorem
f (D, D0 )

i
= [f (D, D0 )]−1 xm y n

n
1

e
f (D, D0 ) = eax+by sin(cx + dy) eax+by sin(cx + dy) D → D + a,D0 → D0 + b
f (D, D0 )
e 1
Downloaded From : www.EasyEngineering.net

= eax+by sin(cx + dy) Solve by form 2


r
f (D + a, D0 + b)
i
1
f (D, D0 ) = sin(ax + by)φ(x, y) Or eax+by φ(x, y) F (x, y)
n
f (D, D0 )
1
g
= F (x, y)
(D − m1 D0 )(D − m2 D0 )
.

1 R
= F (x, c − m2 x)dx y = c − m2 x
n

(D − m1 D0 )
e t

Note:

Downloaded From : www.EasyEngineering.net


If the Dr. is zero then put x in Nr. and differentiate w.r.t D in Dr.

Page 19
Umar Ali
Panimalar Engineering College Downloaded From : www.EasyEngineering.net Umar Ali

1. Solve (D3 − 7DD02 − 6D03 )z = e2x+y + sin(x + 2y)


Solution:
A.E : Put D = m and D0 = 1
i.e m3 − 7m − 6 = 0
1 0 −7 −6

−1 −1 1 6

1 −1 −6 0
2
i.e m − m − 6 = 0
(m + 2)(m − 3) = 0
⇒ m = −2, 3
∴ m = −1, −2, 3
C.F = f1 (y − x) + f2 (y − 2x) + f3 (y + 3x)

To find P.I

ne t
Ist,

P.I =
D3
1
02
− 7DD − 6D 03
e2x+y + 3
1
D − 7DD02 − 6D03
sin(x + 2y)

n g . D=a=2
D0 = b = 1

=
1
8−7×2×1−6
e2x+y +
1
−D + 28D + 24D0
sin(x + 2y)

e ri IInd,

=
1 2x+y
−12
e
−1 2x+y
+
1
27D + 24D0
D
sin(x + 2y)

i n e D2 = −a2 = −1
D02 = −b2 = −4

ng
= e + sin(x + 2y)
12 27D2 + 24DD0 D02 = −b2 = −4
−e2x+y D
=
12
+

y E
27(−1) + 24(−2)
sin(x + 2y)

as
−e2x+y cos(x + 2y)
= +
12 −27 − 48
=
−e2x+y
12

w
−e2x+y .
+


E cos(x + 2y)
−75
cos(x + 2y)

w
=
12 75

w∴z

z
= C.F + P.I

= f1 (y − x) + f2 (y − 2x) + f3 (y + 3x) −
e2x+y
12

cos(x + 2y)
75

2. Solve (D2 − D02 )z = ex+2y + sin(2x − y)


ex+2y sin(2x − y)
Ans: z = f1 (y + x) + f2 (y − x) − −
3 3

3. Solve (D3 + D2 D0 + 4DD02 + 4D03 )z = cos(2x + y)


1
Ans: z = f1 (y − x) + f2 (y + 2ix) + f3 (y − 2ix) − sin(2x + y)
24

4. Solve (D3 − 7DD02 − 6D03 )z = cos(x + y)


1
Ans: z = f1 (y − x) + f2 (y − 2x) + f3 (y + 3x) + sin(x + y)
12

5. Solve (D2 + DD0 − 6D02 )z = cos(x + 2y)


1
Ans: z = f1 (y + 2x) + f2 (y − 3x) + cos(x + 2y)
21

Page 20

Downloaded From : www.EasyEngineering.net


Panimalar Engineering College Downloaded From : www.EasyEngineering.net Umar Ali

6. Solve (D2 + 3DD0 − 4D02 )z = sin y


1
Ans: z = f1 (y − 4x) + f2 (y + x) + sin y
4

7. Solve (D2 − 2DD0 + D02 )z = ex+2y + sin(2x − 3y)


1
Ans: z = f1 (y + x) + xf2 (y + x) + ex+2y − sin(2x − 3y)
25

8. Solve (D2 + 2DD0 + D02 )z = ex+2y + sin(2x − y)


1
Ans: z = f1 (y − x) + xf2 (y − x) + ex+2y − sin(2x − y)
9
∂3z ∂3z
9. Solve − 2 = ex+2y + 4 sin(x + y)
∂x3 ∂x2 ∂y
1
Ans: z = f1 (y) + xf2 (y) + f3 (y + 2x) − ex+2y − 4 cos(x + y)
3

10. Solve (4D2 − 4DD0 + D02 )z = e3x−2y + sin x


   
1 1 1 1
Ans: z = f1 y + x + xf2 y + x + e3x−2y − sin x
2 2 64 4

11. Solve (D2 − DD0 − 20D02 )z = e5x+y + sin(4x − y)


Solution:

ne t
A.E : Put D = m and D = 1 0

n g .
i.e m2 − m − 20 = 0
(m − 5)(m + 4) = 0

e ri
⇒ m = 5, −4

i n e
ng
∴ C.F = f1 (y + 5x) + f2 (y − 4x)

To find P.I

y E Ist,

as
D=a=5
1 1
P.I = 0− 02
e5x+y + 2 sin(4x − y) D0 = b = 1
2
D − DD 20D D − DD0− 20D02
=

w . E
1
25 − 5 − 20
1 5x+y 1
e5x+y +
1
−16 − 4 + 20
sin(4x − y) IInd,
D2 = −a2 = −16

w w =

=
0
e
x
2D − D0
x
+ sin(4x − y)

e
0
5x+y
+
x
2D − D0
D
sin(4x − y)
D02 = −b2 = −1
D02 = −b2 = 4
= e5x+y + x 2 sin(4x − y)
10 − 1 2D + DD0
x 5x+y cos(4x − y)2
= e +x
9 2(−16) − 4
x 5x+y 4 cos(4x − y)2
= e +x
9 −36
x 5x+y x
= e − cos(4x − y)
9 9
∴z = C.F + P.I
x 5x+y x
z = f1 (y + 5x) + f2 (y − 4x) + e − cos(4x − y)
9 9

12. Solve (D2 − DD0 − 30D02 )z = xy + e6x+y


Solution:
A.E : Put D = m and D0 = 1

Page 21

Downloaded From : www.EasyEngineering.net


Panimalar Engineering College Downloaded From : www.EasyEngineering.net Umar Ali

i.e m2 − m − 30 = 0
(m − 6)(m + 5) = 0
⇒ m = 6, −5
∴ C.F = f1 (y + 6x) + f2 (y − 5x)

To find P.I D=a=6


D0 = b = 1
1 1
P.I = 0− 02
e6x+y + 2 xy
2
D − DD 30D D − DD0− 30D02
1 1
= e6x+y + 0
 xy
− 30D02
 2
36 − 6 − 30 2
D − DD
D
D2
1 6x+y 1
= e +  0  xy
30D02

0 2
D
D 1− +
D D2
 0 −1
30D02

x 6x+y 1 D

t
= e + 1 + xy
2D − D0 D2 D D2

=
x
12 − 1
e 6x+y 1
+ 2 1+
D
"  0
D
D
+
30D02
D2
  0
+
D
D
+
30D02
D2
2 #
+ ... xy

. ne
=
x 6x+y
e
1
+ 2 1+
  0 
D

i n
xy Since power of y is 1, D’ enogh g
r
11 D D
=
x 6x+y
11
e
1 h
+ 2 xy +
D 
xi
D
x2
n e e
i
x 6x+y 1
= e + 2 xy +

ng
11 D 2
 2
x3

x 6x+y 1 x
= e + y. +
11
x 6x+y
D
x3
y E
2
x4
6

as
= e + y. +
11 6 24

E
∴z = C.F + P.I

w .
= f1 (y + 6x) + f2 (y − 5x) +
x 6x+y x3 y x4
11
e +
6
+
24

w w
13. Solve (D2 − 4DD0 + 4D02 )z = xy + e2x+y
Solution:
A.E : Put D = m and D0 = 1
i.e m2 − 4m + 4 = 0
(m − 2)(m − 2) = 0
⇒ m = 2, 2
∴ C.F = f1 (y + 2x) + xf2 (y + 2x)
To find P.I

1 1
P.I = e2x+y + 2 xy
D2 − 4DD0+ 4D02 D − 4DD0+ 4D02
1 1
= e2x+y +  xy
D − 4DD0+ 4D02
 2
4−8+4
D2
D2

Page 22

Downloaded From : www.EasyEngineering.net


Panimalar Engineering College Downloaded From : www.EasyEngineering.net Umar Ali

D=a=2
D0 = b = 1
1 2x+y 1
= e +  0  xy
4D02

0 4D
D2 1 − −
D D2
 0 −1
4D02

x 2x+y 1 4D
= e + 2 1− − xy
2D − 4D0 D D D2
"  0 2 #
4D02
  0
x 2x+y 1 4D 4D 4D02
= e + 2 1+ − + − + ... xy
4−4 D D D2 D D2
  0 
x 2x+y 1 4D
= e + 2 1+ xy Since power of y is 1, D’ enogh
0 D D
x2 2x+y
 
1 4x
= e + 2 xy +
2 D D
2
4x2
 
x 2x+y 1
= e + 2 xy +
2 D 2
2
 2
4x3

x 2x+y 1 x
= e + y. +

t
2 D 2 6

e
2 3 4
x 2x+y x 4x
= e + y. +

=
2
x2 2x+y x3 y x4
e +
6
+
24

g . n
n
2 6 6
∴z = C.F + P.I
x2 2x+y x3 y x4
e ri
z = f1 (y + 2x) + xf2 (y + 2x) +
2
e +
6
+

i
6

n e
ng
14. Solve (D2 + DD0 − 6D02 )z = x2 y + e3x+y
4 5
x y x

y E
Ans: z = f1 (y + 2x) + f2 (y − 3x) + 16 e3x+y +
12

60

as
15. Solve (D3 − 2D2 D0 )z = 2e2x + 3x2 y
1 x5 y x6
Ans: z = f1 (y) + xf2 (y) + f3 (y + 2x) + e2x +

E

w .
16. Solve (D2 − 2DD0 )z = e2x−y + x3 y
4

x5 y x6
60 60

w
1
Ans: z = f1 (y) + f2 (y + 2x) + e2x−y + +
8 20 60

w
17. Solve (D2 + 3DD0 + 2D02 )z = sin(2x + y) + x + y
Solution:
A.E : Put D = m and D0 = 1
i.e m2 + 3m + 2 = 0
(m + 1)(m + 2) = 0
⇒ m = −1, −2
∴ C.F = f1 (y − x) + f2 (y − 2x)

To find P.I

1 1
P.I = sin(2x + y) + 2 (x + y)
D2 + 3DD0 + 2D02 D + 3DD0 + 2D02
1 1
= sin(2x + y) + 0
 (x + y)
+ 2D02
 2
−4 − 6 − 2 2
D + 3DD
D
D2
Page 23

Downloaded From : www.EasyEngineering.net


Panimalar Engineering College Downloaded From : www.EasyEngineering.net Umar Ali

1 1
= sin(2x + y) +  0  (x + y) D2 = −a2 = −4
2D02

−12 3D
D2 1 + + D02 = −b2 = −1
D D2
−1
D02 = −b2 = −2
 0
2D02

1 1 3D
= − sin(2x + y) + 2 1 + + (x + y)
12 D D D2
"  0 2 #
2D02
  0
1 1 3D 3D 2D02
= − sin(2x + y) + 2 1 − + + + − ... (x + y)
12 D D D2 D D2
  0 
1 1 3D
= − sin(2x + y) + 2 1 − (x + y)
12 D D
 
1 1 3
= − sin(2x + y) + 2 x + y −
12 D D
1 1
= − sin(2x + y) + 2 [x + y − 3x]
12 D
1 1
= − sin(2x + y) + 2 [y − 2x]
12 D
2x2

1 1
= − sin(2x + y) + yx −
12 D 2

t
2 3
1 x 2x
= − sin(2x + y) + y. −

=
12
1
− sin(2x + y) +
2
x2 y x3

6

. ne
g
12 2 3
∴z = C.F + P.I
1 x2 y x3

ri n
e
z = f1 (y − x) + f2 (y − 2x) − sin(2x + y) + −
12 2 3

18. Solve (D2 + 3DD0 + 2D02 )z = x + y


i n e
ng
x2 y x3
Ans: z = f1 (y − x) + f2 (y − 2x) + 2 −
3

y E
19. Solve (D2 − DD0 − 2D02 )z = 2x + 3y + e3x+4y

as
1 5x3 3x2 y
Ans: z = f1 (y + 2x) + f2 (y − x) − e3x+4y + +
35 6 2

. E
20. Solve (D2 − 6DD0 + 5D02 )z = ex sin hy + xy

w
i.e,(D2 − 6DD0 + 5D02 )z = ex
e − e−y
 y 
+ xy

w
2
1
(D2 − 6DD0 + 5D02 )z = (ex+y − ex−y ) + xy

w 2
1 1
(D2 − 6DD0 + 5D02 )z = ex+y − ex−y + xy
Solution:
2 2

A.E : Put D = m and D0 = 1


i.e m2 − 6m + 5 = 0
(m − 1)(m − 5) = 0
⇒ m = 1, 5
∴ C.F = f1 (y + x) + f2 (y + 5x)

To find P.I

1 1 x+y 1 1 x−y 1
P.I = e + 2 e + 2 xy
D2 − 6DD0 + 5D02 2 D − 6DD0 + 5D02 2 D − 6DD0 + 5D02

Page 24

Downloaded From : www.EasyEngineering.net


Panimalar Engineering College Downloaded From : www.EasyEngineering.net Umar Ali

   
1 1 1 1 1
= ex+y + ex−y +  xy
D2 − 6DD0 + 5D02

2 1−6+5 2 1+6+5
D2
D2
   
1 1 x+y 1 1 1
= e + ex−y +  0  xy
5D02

2 0 2 12 6D
D2 1 − −
D D2
 0 −1
5D02
  
1 x x+y 1 x−y 1 6D
= e + e + 2 1− − xy
2 2D − 6D0 24 D D D2
"  0 2 #
5D02
  0
5D02
 
1 x x+y 1 x−y 1 6D 6D
= e + e + 2 1+ − + − + ... xy
2 2−6 24 D D D2 D D2
    0 
1 x 1 1 6D
= ex+y + ex−y + 2 1 + xy
2 −4 24 D D
 
1 1 1 6x
= − ex+y + ex−y + 2 xy +
8 24 D D
6x2
 
1 1 1
= − ex+y + ex−y + 2 xy +
8 24 D 2
 2
6x3

1 1 1 x
= − ex+y + ex−y +
8
1
24
1
D
x 3
y
2
6x
+
4
6

ne t
∴z
=

=
− ex+y + ex−y + y
8
C.F + P.I
24 6
+
24

n g .
z =
1
8
1
f1 (y + x) + f2 (y + 5x) − ex+y + ex−y + y
24
x3
6
+
x4
4

e ri
21. Solve (D2 + 2DD0 + D02 )z = sin h(x + y) + ex+2y
i n e
ng
1 1
Ans: z = f1 (y − x) + xf2 (y − x) + [ex+y − e−(x+y) ] + ex+2y (OR)
8 9
1

E 1
Ans: z = f1 (y − x) + xf2 (y − x) + sin h(x + y) + ex+2y

y
4 9

Solution:

. E as
22. Solve (D3 + D2 D0 − DD02 − D03 )z = sin 2x cos y

w w
(D3 + D2 D0 − DD02 − D03 )z = 21 [sin(2x + y) + sin(2x − y)]

2
1 1
(D3 + D2 D0 − DD02 − D03 )z = sin(2x + y) + sin(2x − y)
2

wA.E : Put D = m and D0 = 1


i.e m3 + m2 − m − 1 = 0
1 1 −1 −1

1 1 2 1

1 2 1 0
i.e m2 + 2m + 1 = 0
(m + 1)(m + 1) = 0
⇒ m = −1, −1
∴ m = 1, −1, −1
C.F = f1 (y + x) + f2 (y − x) + xf3 (y − x)

Page 25

Downloaded From : www.EasyEngineering.net


Panimalar Engineering College Downloaded From : www.EasyEngineering.net Umar Ali

To find P.I D2 = −a2 = −4


D02 = −b2 = −1
1 1 1 1
P.I = 3 2 0 02 03
sin(2x + y) + 3 2 0 02 03
sin(2x − y) D02 = −b2 = −2
D + D D − DD − D 2 D + D D − DD − D 2
   
1 1 1 1
= sin(2x + y) + sin(2x − y)
2 −4D − 4D0 + D + D0 2 −4D − 4D0 + D + D0
   
1 1 1 1
= sin(2x + y) + sin(2x − y)
2 −3D − 3D0 2 −3D − 3D0
   
1 D 1 D
= sin(2x + y) + sin(2x − y)
2 −3D2 − 3DD0 2 −3D2 − 3DD0
   
1 cos(2x + y)2 1 cos(2x − y)(2)
= +
2 −3(−4) − 3(−2) 2 −3(−4) − 3(2)
cos(2x + y) cos(2x − y)
= +
18 6
∴z = C.F + P.I
cos(2x + y) cos(2x − y)
z = f1 (y + x) + f2 (y − x) + xf3 (y − x) + +
18 6

23. Solve (D3 + D2 D0 − DD02 − D03 )z = ex cos 2y

ne t
Solution:
A.E : Put D = m and D0 = 1
n g .
i.e m3 + m2 − m − 1 = 0

e ri
1 1 −1

i n
−1
e
g
1 1 2 1

E n 1 2 1 0

sy
i.e m2 + 2m + 1 = 0

.E a
(m + 1)(m + 1) = 0
⇒ m = −1, −1

w w ∴ m = 1, −1, −1
C.F = f1 (y + x) + f2 (y − x) + xf3 (y − x)

wTo find P.I

1
D =D+a=D+1
D0 = D0 + b = D0
P.I = ex cos 2y
D3 + D2 D0 − DD02 − D03
1
= ex cos 2y
(D + 1)3 + (D + 1)2 D0 − (D + 1)D02 − D03
R.P of ei2y
= ex
(D + 1)3 + (D + 1)2 D0 − (D + 1)D02 − D03
R.P of ei2y
= ex
(0 + 1)3 + (0 + 1)2 (2i) − (0 + 1)(2i)2 − (2i)3
R.P of ei2y
= ex
1 + 2i + 4 + 8i
i2y
x R.P of e
= e
5 + 10i
R.P of ei2y
= ex
5(1 + 2i)
ex (1 − 2i)(cos 2y + i sin 2y)
= R.P of
5 (1 + 2i)(1 − 2i)
Page 26

Downloaded From : www.EasyEngineering.net


Panimalar Engineering College Downloaded From : www.EasyEngineering.net Umar Ali

ex cos 2y + 2 sin 2y
=
5 1+4
ex
= (cos 2y + 2 sin 2y)
25
∴z = C.F + P.I
ex
z = f1 (y + x) + f2 (y − x) + xf3 (y − x) + (cos 2y + 2 sin 2y)
25

24. r + s − 6t = y cos x
Solution:
Given : (D2 + DD0 − 6D02 ) = y cos x A.E : Put D = m and D0 = 1
i.e m2 + m − 6 = 0
(m + 3)(m − 2) = 0
⇒ m = −3, 2
C.F = f1 (y − 3x) + f2 (y + 2x)

ne t
To find P.I

n g .
P.I =
1
D2 + DD0 − 6D02
y cos x

e ri
=
1

i n
(D + 3D0 )(D − 2D0 ) e
y cos x

ng
Z
1
= (c − 2x) cos xdx where y = c − 2x
(D + 3D0 )

y E=
1
(D + 3D0 )
[(c − 2x) sin x − (−2)(− cos x)]

as
1
= [y sin x − 2 cos x)]
(D + 3D0 )

w . E =
Z
[(c + 3x) sin x − 2 cos x)] dx where y = c + 3x

[(c + 3x)(− cos x) − 3(− sin x) − 2 sin x]

w
=

w ∴z
=

=
[y(− cos x) + 3 sin x) − 2 sin x]

[−y cos x) + sin x]

= C.F + P.I

z = f1 (y − 3x) + f2 (y + 2x) − y cos x) + sin x

25. (D2 + 2DD0 + D02 ) = −x sin y


Solution:
A.E : Put D = m and D0 = 1
i.e m2 + 2m + 1 = 0
(m + 1)(m + 1) = 0
⇒ m = −1, −1
C.F = f1 (y − x) + xf2 (y − x)

Page 27

Downloaded From : www.EasyEngineering.net


Panimalar Engineering College Downloaded From : www.EasyEngineering.net Umar Ali

To find P.I

1
P.I = − x sin y
D2 + 2DD0 + D02
1
= − x sin y
(D + D )(D + D0 )
0
Z
1
= − x sin(c + x)dx where y = c + x
(D + D0 )
1
= − [x(− cos(c + x) − (1)(− sin(c + x))]
(D + D0 )
1
= − [−x cos y + sin y)]
(D + D0 )
Z
= − [−x cos(c + x) + sin(c + x)] dx where y = c + x

= − [(−x)(sin(c + x)) − (−1)(− cos(c + x)) + (− cos(c + x))]

= − [−x sin y) − cos y) − cos y]

t
= − [−x sin y) − 2 cos y]

= x sin y + 2 cos y

. ne
∴z

z
= C.F + P.I

i
= f1 (y − x) + xf2 (y − x) + x sin y + 2 cos y
n g
∂2z ∂2z ∂2z
e e r
26. Solve
∂x2

∂x∂y
− 2
∂y 2
= (y − 1)ex

i n
ng
Solution:
2 0 02 x
Given: (D − DD − 2D ) = (y − 1)e
A.E : Put D = m and D0 = 1

y E
i.e m2 − m − 2 = 0

. E
(m + 1)(m − 2) = 0 as
w w
⇒ m = −1, 2
C.F = f1 (y − x) + f2 (y + 2x)

wTo find P.I

P.I =
1
(y − 1)ex
D2 − DD0 − 2D02
1
= d (y − 1)ex
(D + D )(D − 2D0 )
0
Z
1
= (c − 2x − 1)ex dx where y = c − 2x
(D + D0 )
1
= [(c − 2x − 1)ex − (−2)(ex )]
(D + D0 )
1
= [(y − 1)ex + 2ex ]
(D + D0 )
Z
= [(c + x − 1)ex + 2ex ] dx where y = c + x

= [(c + x − 1)ex − (1)ex + 2ex ]

= (y − 1)(ex ) + ex

Page 28

Downloaded From : www.EasyEngineering.net


Panimalar Engineering College Downloaded From : www.EasyEngineering.net Umar Ali

= yex − ex + ex

= yex

∴z = C.F + P.I

z = f1 (y − x) + f2 (y + 2x) + yex

27. Solve (D2 − 5DD0 + 6D02 )z = y sin x


Ans: z = f1 (y + 2x) + f2 (y + 3x) + 5 cos x − y sin x

28. Solve (D2 − 2DD0 + D02 )z = x2 y 2 ex+y


 4 2
x5 y x6 x+y

x y
Ans: z = f1 (y + x) + xf2 (y + x) + + + e
12 15 60

(Non-Homogeneous)Form:

Form
(D − m1 D0 − c1 )(D − m2 D0 − c2 )z = F (x, y)
C.F

ne t
ec1 x f1 (y + m1 x) + ec2 x f2 (y + m2 x)
(D − mD0 − c)2 z = F (x, y)
g .
ecx f1 (y + mx) + xecx f2 (y + mx)

n
(D0 − m1 D − c1 )(D0 − m2 D − c2 )z = F (x, y)
(D0 − mD − c)2 z = F (x, y)
e ri
ec1 y f1 (x + m1 y) + ec2 y f2 (x + m2 y)
ecy f1 (x + my) + yecy f2 (x + my)

To find P.I
i n e
P.I is same as Homogeneous

E ng
Solution:

as y
29. (D2 − 2DD0 + D02 − 3D + 3D0 + 2)z = e2x−y

w . E 
(D − D0 )2 − 1 − 3D + 3D0 + 3 z

= e2x−y

w w [(D − D0 − 1)(D − D0 + 1) − 3(D − D0 − 1)] z

(D − D0 − 1) [(D − D0 + 1) − 3] z

(D − D0 − 1)(D − D0 − 2)z
= e2x−y

= e2x−y

= e2x−y

Here m1 = 1, c1 = 1

m2 = 1, c2 = 2

C.F = ex f1 (y + x) + ex f2 (y + 2x)

To find P.I D=a=2


D0 = b = −1
1
P.I = e2x−y
D2 − 2DD0 + D02 − 3D + 3D0 + 2
1
= e2x−y
4+4+1−6−3+2
1 2x−y
= e
2
∴z = C.F + P.I
1 Page 29
z = ex f1 (y + x) + ex f2 (y + 2x) + e2x−y
2

Downloaded From : www.EasyEngineering.net


Panimalar Engineering College Downloaded From : www.EasyEngineering.net Umar Ali

30. (D2 + 2DD0 + D02 − 2D − 2D0 )z = sin(x + 2y)


Solution:
(D + D0 )2 − 2(D + D0 ) z = sin(x + 2y)
 

(D + D0 ) [(D + D0 ) − 2] z = sin(x + 2y)


(D − (−1)D0 )(D − (−1)D0 − 2)z = sin(x + 2y)
Here m1 = −1, c1 = 0
m2 = −1, c2 = 2
C.F = f1 (y − x) + e2x f2 (y − x)

To find P.I D2 = −a2 = −1


D02 = −b2 = −4
1
P.I = sin(x + 2y) DD0 = −ab = −2
D2 + 2DD0 + D02 − 2D − 2D0
1
= sin(x + 2y)
−1 − 4 − 4 − 2D − 2D0
=
D
−9D − 2D2 − 2DD0
sin(x + 2y)

ne t
=
cos(x + 2y)
−9D + 2 + 4

n g .
=
(−9D − 6) cos(x + 2y)
(−9D + 6)(−9D − 6)

e ri
e
(−9(− sin(x + 2y)) − 6 cos(x + 2y)
=
81D2 − 36
9 sin(x + 2y) − 6 cos(x + 2y)
i n
ng
=
−81 − 36
9 sin(x + 2y) − 6 cos(x + 2y)
=
3
−117
2
y E
as
= − sin(x + 2y) + cos(x + 2y)
39 39
∴z

z
= C.F + P.I

. E
= f1 (y − x) + e2x f2 (y − x) −

w
3
39
sin(x + 2y) +
2
39
cos(x + 2y)

w w
31. (D2 − 2DD0 + D02 − 3D + 3D0 + 2)z = e2x−y
Solution:
(D − D0 )2 − 1 − 3D + 3D0 + 3 z = e2x−y
 

[(D − D0 − 1)(D − D0 + 1) − 3(D − D0 − 1)] z = e2x−y


(D − D0 − 1) [(D − D0 + 1) − 3] z = e2x−y
(D − D0 − 1)(D − D0 − 2)z = e2x−y
Here m1 = 1, c1 = 1
m2 = 1, c2 = 2
C.F = ex f1 (y + x) + ex f2 (y + 2x)

To find P.I D=a=2


D0 = b = −1
1
P.I = e2x−y
D2 − 2DD0 + D02 − 3D + 3D0 + 2
1
= e2x−y
4+4+1−6−3+2 Page 30

Downloaded From : www.EasyEngineering.net


Panimalar Engineering College Downloaded From : www.EasyEngineering.net Umar Ali

1 2x−y
= e
2
∴z = C.F + P.I
1
z = ex f1 (y + x) + ex f2 (y + 2x) + e2x−y
2

32. (2D2 − DD0 − D02 + 6D + 3D0 )z = xey


Solution:
2D − 2DD0 + DD0 − D02 + 6D + 3D0 z = xey
 2 

2D − 2DD0 + 6D + DD0 − D02 + 3D0 z = xey


 2 

[2D(D − D0 + 3) + D0 (D − D0 + 3)] z = xey


[(D − D0 + 3)(2D + D0 )] z = xey
C.F ⇒ [(D − D0 + 3)(2D + D0 )] z = 0

(D − D0 + 3)(D + 21 D0 ) z = 0


ne t
Here m1 = 1, c1 = −3
m2 = −1
2 , c2 =0

n g .
C.F = e−3x f1 (y + x) + e0x f2 y − 12 x


e ri
To find P.I

i n e D =D+a=D

ng
D0 = D0 + b = D0 + 1
1
P.I = xey

ey
y E
2D2 − DD0 − D02 + 6D + 3D0
1

as
= x
2D2 − D(D0 + 1) − (D0 + 1)2 + 6D + 3(D0 + 1)
D
=

=
ey

ey

w .
2

E
2D − DD − D − D − 2D0 − 1 + 6D + 3D0 + 3
0

1
02

2D2 − DD0 + 5D − D02 − 2D0 + D0 + 2


x
x

w w =

=
ey 

ey

1+
 2
2 0
2 1 + 2D −DD +5D−D
1
2
02 −2D 0 +D 0 +2  x

2D − DD0 + 5D − D02 − 2D0 + D0 + 2


−1
x
2 2
" 2 #
ey 2D − DD0 + 5D − D02 − 2D0 + D0 + 2 2D − DD0 + 5D − D02 − 2D0 + D0 + 2
 2   2
= 1− + x
2 2 2
ey
 
5
= 1− D x
2 2
ey
 
5
= x−
2 2
∴z = C.F + P.I
ey
   
1 5
z = e−3x f1 (y + x) + e0x f2 y − x + x−
2 2 2

33. (D2 − 3DD0 + 2D02 + 2D − 2D0 )z = x + y + sin(2x + y)


1 1
Ans: z = f1 (y + x) + e2x f2 (y + 2x) − (x2 + xy) − cos(2x + y)
2 2

Page 31

Downloaded From : www.EasyEngineering.net


Panimalar Engineering College Downloaded From : www.EasyEngineering.net Umar Ali

Lagrange’s Method

Form: P p + Qq = R
Auxiliary Equation:
dx dy dz
= =
P Q R

34. Solve x(y − z)p + y(z − x)q = z(x − y)


Solution:
Lagranges’s type: P p + Qq = R
Here P = x(y − z), Q = y(z − x), R = z(x − y)
Auxiliary Equation:
dx dy dz
= =
P Q R
dx dy dz
= =
x(y − z) y(z − x) z(x − y)
Use Lagrangian Multiplier 1,1,1

ne t
dx + dy + dz
x(y − z) + y(z − x) + z(x − y)
=
dx + dy + dz

n
0
g .
⇒ dx + dy + dz

Integrating
=

e
0

ri
n
⇒x+y+z

i e
= c1

1 1 1
Use Lagrangian Multiplier , ,
x y z

E ng
as y 1
x
1 1
dx + dy + dz
y z
1
x
1 1
dx + dy + dz
y z

E
=

.
y−z+z−x+x−y 0
1 1 1

w w ⇒ dx + dy + dz
x y z
Integrating
= 0

w ⇒ log x + log y + log z

log xyz
=

=
log c2

log c2

⇒ xyz = c2

∴ General Solution: φ(x + y + z, xyz) = 0

35. Solve x(y 2 − z 2 )p + y(z 2 − x2 )q = z(x2 − y 2 )


Solution:
Lagranges’s type: P p + Qq = R
Here P = x(y 2 − z 2 ), Q = y(z 2 − x2 ), R = z(x2 − y 2 )
Auxiliary Equation:
dx dy dz
= =
P Q R

Page 32

Downloaded From : www.EasyEngineering.net


Panimalar Engineering College Downloaded From : www.EasyEngineering.net Umar Ali

dx dy dz
= =
x(y 2
−z )2 2 2
y(z − x ) z(x − y 2 )
2

Use Lagrangian Multiplier x,y,z

xdx + ydy + zdz xdx + ydy + zdz


=
x2 (y 2 − z 2 ) + y 2 (z 2 − x2 ) + z 2 (x2 − y 2 ) 0
⇒ xdx + ydy + zdz = 0

Integrating
2
x y2 z2 c1
⇒ + + =
2 2 2 2
x2 + y 2 + z 2 = c1

1 1 1
Use Lagrangian Multiplier , ,
x y z

1
x
1
dx + dy + dz
y
1
z
1
x
1 1
dx + dy + dz
y z

ne t
.
=
y 2 − z 2 + z 2 − x2 + x2 − y 2 0
1
x
1
⇒ dx + dy + dz
y
1
z
= 0

i n g
Integrating

⇒ log x + log y + log z =


e
log c2e r
i n
ng
log xyz = log c2

⇒ xyz = c2

y E
as
∴ General Solution: φ(x2 + y 2 + z 2 , xyz) = 0

. E
36. Solve x2 (y − z)p + y 2 (z − x)q = z 2 (x − y)
Solution:
w
w w
Lagranges’s type: P p + Qq = R
Here P = x2 (y − z), Q = y 2 (z − x), R = z 2 (x − y)
Auxiliary Equation:
dx dy dz
= =
P Q R
dx dy dz
= 2 = 2
x2 (y − z) y (z − x) z (x − y)
1 1 1
Use Lagrangian Multiplier 2 , 2 , 2
x y z

1 1 1 1 1 1
2
dx + 2 dy + 2 dz 2
dx + 2 dy + 2 dz
x y z x y z
=
y−z+z−x+x−y 0
1 1 1
⇒ 2 dx + 2 dy + 2 dz = 0
x y z
Integrating
−1 1 1
⇒ − − = −c1
x y z

Page 33

Downloaded From : www.EasyEngineering.net


Panimalar Engineering College Downloaded From : www.EasyEngineering.net Umar Ali

1 1 1
+ + = c1
x y z

1 1 1
Use Lagrangian Multiplier , ,
x y z

1 1 1 1 1 1
dx + dy + dz dx + dy + dz
x y z x y z
=
x(y − z) + y(z − x) + z(x − y) 0

1 1 1
⇒ dx + dy + dz = 0
x y z
Integrating

⇒ log x + log y + log z = log c2

log xyz = log c2

⇒ xyz = c2

ne t
.
 
1 1 1
∴ General Solution: φ + + , xyz =0
x y z

37. Solve (mz − ny)p + (nx − lz)q = (ly − mx)


i n g
Solution:

e e r
Lagranges’s type: P p + Qq = R

i n
ng
Here P = mz − ny, Q = nx − lz, R = ly − mx
Auxiliary Equation:
dx
=
dy
=
dz

y E
as
P Q R
dx dy dz
= =

E
mz − ny nx − lz ly − mx

.
Use Lagrangian Multiplier x,y,z

w
w w xdx + ydy + zdz
x(mz − ny) + y(nx − lz) + z(ly − mx)
⇒ xdx + ydy + zdz
=

=
xdx + ydy + zdz

0
0

Integrating
2
x y2 z2 c1
⇒ + + =
2 2 2 2
x2 + y 2 + z 2 = c1

Use Lagrangian Multiplier l, m, n

ldx + mdy + ndz ldx + mdy + ndz


=
l(mz − ny) + m(nx − lz) + n(ly − mx) 0
⇒ ldx + mdy + ndz = 0

Integrating

⇒ lx + my + nz = c2

Page 34

Downloaded From : www.EasyEngineering.net


Panimalar Engineering College Downloaded From : www.EasyEngineering.net Umar Ali

∴ General Solution: φ(x2 + y 2 + z 2 , lx + my + nz) = 0

38. Solve (3z − 4y)p + (4x − 2z)q = (2y − 3x)


Refer previous question here l = 2, m = 3, n = 4

39. Solve (y − xz)p + (yz − x)q = (x + y)(x − y)


Solution:
Lagranges’s type: P p + Qq = R
Here P = y − xz, Q = yz − x, R = x2 − y 2
Auxiliary Equation:
dx dy dz
= =
P Q R
dx dy dz
= = 2
y − xz yz − x x − y2
Use Lagrangian Multiplier x,y,z

xdx + ydy + zdz


x(y − xz) + y(yz − x) + z(x2 − y 2 )
=
xdx + ydy + zdz
0

ne t
⇒ xdx + ydy + zdz = 0

n g .

x2
+
Integrating
y2
+
z2
=
e ri
c1
2

i
2

n
2
x2 + y 2 + z 2
e
=
2
c1

Use Lagrangian Multiplier y, x, 1

E ng
as y ydx + xdy + dz
y(y − xz) + x(yz − x) + (x2 − y 2 )
=
ydx + xdy + dz
0

w . E ⇒ ydx + xdy + dz

d(xy) + dz
=

=
0

w w Integrating

⇒ xy + z = c2

∴ General Solution: φ(x2 + y 2 + z 2 , xy + z) = 0

40. Solve x(y 2 + z)p − y(x2 + z)q = z(x2 − y 2 )


1 1 1
Hint: Multipliers, , , and x, y, −1
x y z
Ans: φ(x2 + y 2 − 2z, xyz) = 0

41. Solve (y 2 + z 2 )p − xyq + xz = 0


Solution:
Lagranges’s type: P p + Qq = R
Here P = y 2 + z 2 , Q = −xy, R = −xz
Auxiliary Equation:

Page 35

Downloaded From : www.EasyEngineering.net


Panimalar Engineering College Downloaded From : www.EasyEngineering.net Umar Ali

dx dy dz
= =
P Q R
dx dy dz
2 2
= =
y +z −xy −xz
Taking 2nd and 3rd member,

dy dz
=
−xy −xz

dy dz
=
y z
dy dz
− = 0
y z
log y − log z = log c1
y
= c1
z

Use Lagrangian Multiplier x,y,z

ne t
xdx + ydy + zdz
x(y 2 + z 2 ) + y(−xy) + z(−xz)
=
xdx + ydy + zdz

n0
g .
⇒ xdx + ydy + zdz = 0

e ri

x2
+
Integrating
y2

i n
+
z2
= ec2

ng
2 2 2 2
x2 + y 2 + z 2 = c2

y
y
E
as

∴ General Solution: φ , x2 + y 2 + z 2 = 0
z

Solution:

w . E
42. Solve (x2 − y 2 − z 2 )p + 2xyq − 2xz = 0

w w
Lagranges’s type: P p + Qq = R
Here P = x2 − y 2 − z 2 , Q = 2xy, R = 2xz
Auxiliary Equation:
dx dy dz
= =
P Q R
dx dy dz
2 2 2
= =
x −y −z 2xy 2xz
Taking 2nd and 3rd member,

dy dz
=
2xy 2xz
dy dz
=
y z
dy dz
− = 0
y z
log y − log z = log c1
y
= c1
z

Page 36

Downloaded From : www.EasyEngineering.net


Panimalar Engineering College Downloaded From : www.EasyEngineering.net Umar Ali

Use Lagrangian Multiplier x,y,z

xdx + ydy + zdz dy


=
x(x2 − − z 2 ) + y(2xy) + z(2xz)
y2 2xy
xdx + ydy + zdz dy
=
x(x2 − y 2 − z 2 + 2y 2 + 2z 2 ) 2xy
xdx + ydy + zdz dy
=
x2 + y 2 + z 2 2y
2xdx + 2ydy + 2zdz dy
=
x2 + y 2 + z 2 y
2xdx + 2ydy + 2zdz dy
= 0
x2 + y 2 + z 2 y
Integrating

⇒ log(x2 + y 2 + z 2 ) − log y = log c2


2 2 2
x +y +z
log = log c2
y
x2 + y 2 + z 2
y
= c2

ne t
y x2 + y 2 + z 2
.
 
∴ General Solution: φ
z
,
y
=0

i n g
43. Solve (x2 − yz)p + (y 2 − zx)q = z 2 − xy
Solution:

e e r
Lagranges’s type: P p + Qq = R

i n
ng
Here P = x2 − yz, Q = y 2 − zx, R = z 2 − xy

E
Auxiliary Equation:
dx
P
=
dx
dy
Q
=

= 2
dz
R
dy

as y
= 2
dz

E
2
x − yz y − zx z − xy

w . dx − dy
x2 − yz − y 2 + zx
= 2
dy − dz
y − zx − z 2 + xy
= 2
dx + dy + dz
x − yz + y 2 − zx + z 2 − xy
= 3
xdx + ydy + zdz
x − xyz + y 3 − xyz + z 3 − xyz

w w
Taking 1nd and 2nd member,

dx − dy
x2 − y 2 − yz + zx
=
dy − dz
y 2 − z 2 − zx + xy
d(x − y) d(y − z)
=
(x + y)(x − y) + z(x − y) (y + z)(y − z) + x(y − z)
d(x − y) d(y − z)
=
(x − y)[x + y + z] (y − z)[y + z + x]
d(x − y) d(y − z)
=
x−y y−z
d(x − y) d(y − z)
− = 0
x−y y−z
Integrating

log(x − y) − log(y − z) = log c1


x−y
log = log c1
y−z
x−y
= c1
y−z

Page 37

Downloaded From : www.EasyEngineering.net


Panimalar Engineering College Downloaded From : www.EasyEngineering.net Umar Ali

Taking 3rd and 4th member,

dx + dy + dz xdx + ydy + zdz


=
x2 − yz + y 2 − zx + z 2 − xy x3 − xyz + y 3 − xyz + z 3 − xyz
dx + dy + dz xdx + ydy + zdz
=
x2 + y 2 + z 2 − xy − yz − zx (x + y + z)(x2 + y 2 + z 2 − xy − yz − zx)
xdx + ydy + zdz
dx + dy + dz =
x+y+z
(x + y + z)dx + dy + dz = xdx + ydy + zdz

(x + y + z)dx + dy + dz − xdx − ydy − zdz = 0

Integrating
2 2
(x + y + z) x y2 z2 c2
− − − =
2 2 2 2 2
(x + y + z)2 − x2 − y 2 − z 2 = c2

x2 + y 2 + z 2 + 2xy + 2yz + 2xz − x2 − y 2 − z 2 = c2

2xy + 2yz + 2xz = c2

ne t
xy + yz + xz = c

n g .
i
 
x−y

r
∴ General Solution: φ , xy + yz + xz =0
y−z
2 2 2 2
44. Solve (x + y + yz)p + y(x + y − zx)q = z(x + y)
x2 + y 2
n e e
Ans: φ(x − y − z,
z2
)=0

ng i
y E
. E as
w w
w

Page 38

Downloaded From : www.EasyEngineering.net


Downloaded From : www.EasyEngineering.net Umar Ali

Unit-2

Fourier Series

Condition for a Fourier Expansion:[Dirichlet’s Conditions]


a0 P∞ P∞
Any function f (x) can be developed as a Fourier series + an cos nx + bn sin nx
2 n=1 n=1
where a0 , an , bn are constants, provided.

1. f (x) is periodic, single-valued and finite.

2. f (x) has a finite number of finite discontinuities in any one period and no infinite discontinuity.

3. f (x) has at the most number of maxima and minima.

ne t
Euler’s Formula for the Fourier Coefficients

n g .
i
If a function f (x) defined in c < x < c+2π can be expanded as the infinite trigonometric
series
a0 P ∞ ∞

e e r
n
P
f (x) = + an cos nx + bn sin nx
2 n=1 n=1

ng i
E
c+2π
Z
1

a
where a0

s y =
π
c
f (x) dx

.E
c+2π
Z
1
an = f (x) cos nx dx

w w π

1
c
c+2π
Z

w bn =
π
c
f (x) sin nx dx

NOTE:1
Deduction:[Continuity and Discontinuity]

 f1 (x), for a ≤ x < c;
Consider the function f (x) =
 f (x), for c < x ≤ b.
2

1. If x = a is a point of continuity then use f (x) = f (a)


f (a) + f (b)
2. If x = a is a point of discontinuity at the end point then use f (x) =
2
f (−c) + f (+c)
3. If x = c is a point of discontinuity at the mid point then use f (x) =
2

Page 1

Downloaded From : www.EasyEngineering.net


Downloaded From : www.EasyEngineering.net Umar Ali

NOTE:2

P ∞
P
1. To change to replace n = 2n − 1
n=odd n=1
P∞ ∞
P
2. To change to replace n = 2n
n=even n=1

P ∞
P
3. To change to replace n = 2n + 1
n=odd n=0

NOTE 3: PARSEVAL’S IDENTITY(higher power derivation)

1 R2π a2 1 P 2
For Full Range: [f (x)]2 dx = 0 + (an + b2n )
2π 0 4 2

1 Rπ a2 1 P 2
For Half Range Cosine: [f (x)]2 dx = 0 + an
π0 4 2

1 Rπ 1P 2
ne t
For Full Range Sine:
π0
[f (x)]2 dx =
2
bn

n g .
Full range π Type
e ri
1
i n e
ng
1. If f (x) = (π − x) find the Fourier series of the period 2π in the interval (0, 2π).Hence deduce
2
1 1 1 π
that 1 − + − + . . . =
3 5 7 4

y E
as
Solution:

w E
The Fourier series of f (x) is

. a0 X
∞ ∞

w
X
f (x) = + an cos nx + bn sin nx.................(1)
2

w To find a0
n=1 n=1

Z2π
1
a0 = f (x)dx
π
0
Z2π
1 1
= (π − x)dx
π 2
0
2π
x2

1
= πx −
2π 2 0
1
= [2π 2 − 2π 2 − {0 − 0}]

= 0

Page 2

Downloaded From : www.EasyEngineering.net


Downloaded From : www.EasyEngineering.net Umar Ali

To find an

Z2π
1
an = f (x) cos nxdx
π
0
Z2π
1 1
= (π − x) cos nxdx
π 2
0
 2π
1 sin nx (− cos nx)
= (π − x) − (−1)
2π n n2 0
 2π
1 sin nx cos nx
= (π − x) −
2π n n2 0
  
1 1 1
= 0− 2 − 0− 2 −0
2π n n
= 0

ne t
To find bn

n g .
bn =
1
Z2π
f (x) sin nxdx
e ri
π
0

i n e
ng
Z2π
1 1
= (π − x) sin nxdx
π 2

y E 0
2π

as

1 (− cos nx) (− sin nx)
= (π − x) − (−1)
2π n n2

E
0

.
 2π
1 cos nx sin nx
= −(π − x) −

w w =

1

1
n

−(−π) − 0 − −π − 0
n2 0
1


w =

1 π π
h
+
2π  n  n
1 2π
i
n n

=
2π n
1
=
n

Substituting in (1),

X 1
⇒ f (x) = sin nx
n=1
n

Deduction:

Page 3

Downloaded From : www.EasyEngineering.net


Downloaded From : www.EasyEngineering.net Umar Ali

π
Here x = is a point of continuity.
2

π  ∞
X 1 π
f = sin n
2 n=1
n 2
1 π π 1 2π 1 3π 1 4π 1 5π
π− = sin + sin + sin + sin + sin ...
2 2 2 2 2 3 2 4 2 5 2
1 π
  1 1
= 1 + 0 + (−1) + 0 + (1) + . . .
2 2 3 5
1 1 π
1 − + + ... =
3 5 4

1 1 1 π2
2. Find the Fourier series of f (x) = (π − x)2 in (0, 2π).Deduce that + + + . . . ∞ =
12 22 32 6
Solution:
The Fourier series of f (x) is

ne t
f (x) =
a0 X
2
+

an cos nx +
X∞
bn sin nx.................(1)

n g .
n=1 n=1

e ri
To find a0

i n e
ng
Z2π
1
a0 = f (x)dx

E
π

y
0

as
Z2π
1
= (π − x)2 dx

w . E =
π
0

1 (π − x)3
 2π

w w =
π 3(−1) 0
1
−3π
1
[(−π)3 − π 3 ]

= [−2π 3 ]
−3π
2π 2
=
3

To find an

Z2π
1
an = f (x) cos nxdx
π
0
Z2π
1
= (π − x)2 cos nxdx
π
0

Page 4

Downloaded From : www.EasyEngineering.net


Downloaded From : www.EasyEngineering.net Umar Ali

 2π
1 2 sin nx (− cos nx) (− sin nx)
= (π − x) − 2(π − x)(−1) + 2(−1)(−1)
π n n2 n3 0
 2π
1 sin nx cos nx sin nx
= (π − x)2 − 2(π − x) 2
−2 3
π n n n 0
  
1 1 1
= 0 − 2(−π) 2 − 0 − 0 − 2π 2 − 0
π n n
 
1 2π 2π
= + 2
π n2 n
 
1 4π
=
π n2
4
=
n2

To find bn

1
Z2π

ne t
bn =
π
0
f (x) sin nxdx

n g .
=
1
π
Z2π
(π − x)2 sin nxdx

e ri
1
0

(π − x)2
(− cos nx)
− 2(π − x)(−1)
i n
(− sin nx) e (+ cos nx)
2π

ng
= + 2(−1)(−1)
π n n2 n3 0
 2π

E
1 cos nx sin nx (cos nx)
= −(π − x)2 − 2(π − x) 2 + 2

=
π
1

1
n

as
2 y 
1
−π 2 − 0 + 3 − −π 2 − 0 + 3
n
2

n3 0

π
= 0

w . E n n n n

w w
Substituting in (1)
⇒ f (x) =
π2 X 4
+

cos nx
3 n 2
n=1

Deduction:
Here x = 0 is a point of discontinuity at the end point.


f (0) + f (2π) π2 X 1
= +4 2
cos 0
2 3 n=1
n

0+0 π2 X 1
= +4
2 3 n=1
n2

π 2 + (−π)2 π2 X 1
= +4
2 3 n=1
n2

Page 5

Downloaded From : www.EasyEngineering.net


Downloaded From : www.EasyEngineering.net Umar Ali


X 1 2 π2
4 = π −
n=1
n2 3

X 1 2π 2
4 =
n=1
n2 3

X 1 π2
=
n=1
n2 6
1 1 1 π2
+ + + . . . =
12 22 32 6

3. Expand in Fourier series of periodic 2π of f (x) = x2 for 0 < x < 2π. Deduce the sum
P∞ 1 π2
2
=
n=1 n 6
4π 2 P∞ cos nx ∞ sin nx
P
Ans:f (x) = +4 − 4π
3 n=1 n2 n=1 n

ne t
.
 x, for 0 < x < π;

g
4. Find the Fourier series expansion of f (x) =

n
 2π − x, for π < x < 2π.
1 1 1
Deduce that 2 + 2 + 2 + . . . ∞ = .
1 3 5
π2
8
e ri
Solution:

i n e
ng
The Fourier series of f (x) is

y E
a0 X
∞ X∞

as
f (x) = + an cos nx + bn sin nx.................(1)
2 n=1 n=1

To find a0

w . E
w w a0 =
1
π
Z2π

0
f (x)dx
 π 
Z Z2π
1
= xdx + (2π − x)dx
π
0 π
" 2π #
2 π
x2
 
1 x
= + 2πx −
π 2 0 2 π
 2
π2
 
1 π 2 2 2
= + 4π − 2π − 2π −
π 2 2
 2 2

1 π π
= + 2π 2 − 2π 2 +
π 2 2
1 2
= [π ]
π
= π

Page 6

Downloaded From : www.EasyEngineering.net


Downloaded From : www.EasyEngineering.net Umar Ali

To find an

Z2π
1
an = f (x) cos nxdx
π
0
 π 
Z Z2π
1
= x cos nxdx + (2π − x) cos nxdx
π
0 π
" π  2π #
1 sin nx (− cos nx) sin nx (− cos nx)
= x − (1) + (2π − x) − (−1)
π n n2 0 n n2 π
" π  2π #
1 sin nx cos nx sin nx cos nx
= x + 2
+ (2π − x) −
π n n 0 n n2 π
n n
    
1 (−1) 1 1 (−1)
= 0+ 2
− 0+ 2 +0− 2 − 0−
π n n n n2
1 (−1)n
t
 
1
=
π
2
2 2 −2 2
n n

. ne
=
n2 π
[(−1)n − 1]

i n g
r

 0, when ’n’ is even;
=
 −4 , when ’n’ is odd.
n2 π
n e e
To find bn
ng i
y E
as
Z2π
1
bn = f (x) sin nxdx
π

1
w
 π
Z
0

. E Z2π

w w =

=
π

1
"0

x
x sin nxdx + (2π − x) sin nxdx

(− cos nx)
π

− (1)
(− sin nx)
π 
+ (2π − x)
(− cos nx)
− (−1)
(− sin nx)
2π #

π n n2 0 n n2 π
" π  2π #
1 cos nx sin nx cos nx sin nx
= −x + 2
+ −(2π − x) −
π n n 0 n n2 π
n n
  
1 (−1) (−1)
= −π + 0 − {0 − 0} − 0 − 0 − −π −0
π n n
= 0

Substituting in (1)


π X −4
⇒ f (x) = + cos nx
2 n=odd n2 π

Page 7

Downloaded From : www.EasyEngineering.net


Downloaded From : www.EasyEngineering.net Umar Ali


π 4 X cos nx
= −
2 π n=1,3,5,...
n2

Deduction:
Here x = 0 is a point of discontinuity at the end point.


f (0) + f (2π) π 4 X cos 0
= −
2 2 π n=1,3,5,...
n2

0+0 π 4 1 X
= −
2 2 π n=1,3,5,...
n2
 
π 4 1 1 1
0 = − + + + ...
2 π 12 32 52
 
4 1 1 1 π
+ 2 + 2 + ... =
π 12

1
3
1
5
1
2
π2

ne t
+
12 32 52
+ + . . . =
8

n g .
5. Find the Fourier series of f (x) = x(2π − x). Deduce
1

e
+
r
1
i+
12 32 52
1
+ . . . =
π2
.

e
8

i n
6. Express f (x) = x sin x as a Fourier series in 0 ≤ x ≤ 2π.
Solution:

E ng
as y
The Fourier series of f (x) is

∞ ∞

. E a0 X X
f (x) = + an cos nx + bn sin nx.................(1)
2

w
n=1 n=1

w w
To find a0

Z2π
1
a0 = f (x)dx
π
0
Z2π
1
= x sin xdx
π
0
1
= [x(− cos x) − (1)(− sin x)]2π
0
π
1
= [−x cos x + sin x]2π
0
π
1
= [−2π − 0]
π
= −2

Page 8

Downloaded From : www.EasyEngineering.net


Downloaded From : www.EasyEngineering.net Umar Ali

To find an

Z2π
1
an = f (x) cos nx dx
π
0
Z2π
1
= x sin x cos nx dx
π
0
Z2π
1
= x [sin (1 + n)x + sin (1 − n)x] dx

0
Z2π
1
= x [sin (1 + n)x − sin (n − 1)x] dx

0
    2π
1 − cos(n + 1)x − cos(n − 1)x − sin(n + 1)x sin(n − 1)x
− − (1)

t
= x +
2π n+1 n−1 (n + 1)2 (n − 1)2

=
1


x cos(n + 1)x x cos(n − 1)x sin(n + 1)x sin(n − 1)x
+ + −
2π
0

. ne

1


n+1


n−1 (n + 1)2 (n − 1)2 0

i n g
=

1

2π −

n+1 n−1
1
+

1


e e r
=

+
n+1 n−1
i n
ng
−(n − 1) + n + 1
=
(n + 1)(n − 1)
=
2

y E Provided n 6= 1

as
2
n −1

To find a1

w . E
w w a1 =
1
π
Z2π

0
f (x) cos x dx

Z2π
1
= x sin x cos x dx
π
0
Z2π
1
= x sin 2x dx

0
    2π
1 − cos 2x − sin 2x
= x − (1)
2π 2 4 0
 2π
1 x cos 2x sin 2x
= − +
2π 2 4 0
 
1 2π
= −
2π 2

Page 9

Downloaded From : www.EasyEngineering.net


Downloaded From : www.EasyEngineering.net Umar Ali

1
= −
2

To find bn

Z2π
1
bn = f (x) sin nx dx
π
0
Z2π
1
= x sin x sin nx dx
π
0
Z2π
1
= x [cos (n − 1)x − cos (n + 1)x] dx

0
    2π
1 sin(n − 1)x sin(n + 1)x − cos(n − 1)x − cos(n + 1)x
=

x
n−1

n+1
− (1)
(n − 1)2

(n + 1)2
2π

ne0
t
.

1 x sin(n − 1)x x sin(n + 1)x cos(n − 1)x cos(n + 1)x
= − + −

1

n−1
1 1
n+1

1
(n − 1)2
1

(n + 1)2 0

i n g
=

= 0
(n − 1) 2

(n + 1) 2

(n − 1)2

(n + 1)2

e e r
Provided n 6= 1

i n
To find b1

E ng
b1 =
as y 1
Z2π
f (x) sin x dx

w . E π

1
0
Z2π

w w =
π

1
0
Z2π
x sin x sin x dx

= x sin2 x dx
π
0
Z2π
1 1 − cos 2x
= x dx
π 2
0
    2 2π
1 sin 2x x − cos 2x
= x x− − (1) −
2π 2 2 4 0
 2
 2π
1 x sin 2x x cos 2x
= x2 − − −
2π 2 2 4
 2
  0
1 4π 1 1
= 4π 2 − − − −
2π 2 4 4

Page 10

Downloaded From : www.EasyEngineering.net


Downloaded From : www.EasyEngineering.net Umar Ali

1  2
= 2π

= π

Substituting in (1)


1 X 2
⇒ f (x) = −1 − cos x + π sin x + 2
cos nx
2 n=2
n −1


 sin x, for 0 ≤ x ≤ π; 1
7. Find the Fourier series expansion of f (x) = Deduce that +
 0, for π ≤ x ≤ 2π. 1·3
1 1
+ + . . . ∞.
3·5 5·7
Solution:

ne t
The Fourier series of f (x) is

n g .
f (x) =
a0 X
+

an cos nx +
X∞

e ri
bn sin nx.................(1)
2 n=1

i
n=1

n e
To find a0

E ng
as y a0 =
1
Z2π
f (x)dx

w . E =
π

1
0

sin xdx

w w =
1
π
π
0

[− cos x]π0
−1
= [−1 − 1]
π
2
=
π

To find an

Z2π
1
an = f (x) cos nxdx
π
0

1
= sin x cos nxdx
π
0

Page 11

Downloaded From : www.EasyEngineering.net


Downloaded From : www.EasyEngineering.net Umar Ali


1
= [sin (1 + n)x + sin (1 − n)x] dx

0

1
= [sin (1 + n)x − sin (n − 1)x] dx

0 π
1 − cos(n + 1)x − cos(n − 1)x
= −
2π n+1 n−1
 π 0
1 cos(n + 1)x cos(n − 1)x
= − +
2π n+1 n−1 0
n+1 n−1
  
1 −(−1) (−1) −1 1
= + − +
2π n+1 n−1 n+1 n−1
1 −(−1)n+1 (−1)n+1
 
1 1
= + + −
2π n+1 n−1 n+1 n−1
   
1 n+1 −(n − 1) + (n + 1) n − 1 − (n + 1)
= (−1) +

1

2
(n + 1)(n − 1)
−2)

(n + 1)(n − 1)

ne t
=

1
(−1)n+1 2
2
+ 2
(n − 1) (n − 1)

n g .
=
1
2
2π (n − 1)
[(−1)n+1 − 1]

e ri
= 2
(n − 1)π

[(−1)n+1 − 1]

i n e
ng
 0,
 when ’n’ is odd;
= −2 provided n 6= 1

E , when ’n’ is even.



 2
(n − 1)π

To find a1
as y
w . E 1
Z2π

w w a1 =

=
π

1
0

f (x) cos xdx

sin x cos xdx


π
0

1 sin 2x
= dx
2π 2
0 π
1 − cos 2x
=
2π 2 0
−1
= [1 − 1]

= 0

Page 12

Downloaded From : www.EasyEngineering.net


Downloaded From : www.EasyEngineering.net Umar Ali

To find bn


1
bn = f (x) sin nxdx
π
0

1
= sin x sin nxdx
π
0

1
= [cos (n − 1)x − cos (n + 1)x] dx

0 π
1 sin(n − 1)x sin(n + 1)x
= −
2π n−1 n+1 0
= 0 provided n 6= 1

To find b1

ne t
b1 =
1

f (x) sin xdx
n g .
π
0

e ri
e

1
=


n sin x sin xdx

ng
0

1

y E =
π
sin2 xdx

as
0

1 1 − cos 2x

w . E =

1
π
0

2

sin 2x
dx

w w =

=

1

[π]
x−
2 0

1
=
2

Substituting in (1)

2 ∞
π 1 2 X 1
⇒ f (x) = + sin x − 2
cos nx
2 2 π n=2,4,... n − 1

1 1 2X 1
= + sin x − cos 2nx
π 2 π n=1 (2n)2 − 1
 
1 1 2 cos 2x cos 4x cos 6x
= + sin x − + + + ...
π 2 π 22 − 1 42 − 1 62 − 1

Page 13

Downloaded From : www.EasyEngineering.net


Downloaded From : www.EasyEngineering.net Umar Ali

Deduction:
Here x = 0 is a point of continuity.

 
1 1 2 1 1 1
f (0) = + sin 0 − + + + ...
π 2 π 22 − 1 42 − 1 62 − 1
 
1 2 1 1 1
0 = − + + + ...
π π 3 15 35
 
2 1 1 1 1
+ + + ... =
π 3 15 35 π
1 1 1 1
+ + + ...∞ =
1·3 3·5 5·7 2

Note:
1. If f (−x) = f (x) then f(x) is said to be even funtion.
example: cos x, x2 , x4 , |x|, x sin x, sin |x|, etc,

ne t
2. If f (−x) = −f (x) then f(x) is said to be odd function.

n g .
example: sin x, x, x3 , x cos xetc,.
R∞
e ri
R∞
3. If f(x) is an even function then

i n −∞
R∞ e
f (x)dx = 2 f (x)dx
0

ng
4. If f(x) is an odd function then f (x)dx = 0
−∞

E
5. If f(x) is an even function in (−π, π) then find a0 , an (bn = 0).
6.

as y
If f(x) is an odd function in (−π, π) then find bn (a0 = an = 0).

(i).
1

w .+
E
8. Find the Fourier series of f (x) = x2 in (−π, π) of periodicity 2π. Hence deduce that

12 22 32
1
+
1
+ ... =
π2
6

w w
(ii).

(iii).
1

12 22 32
1
+
1

1
+

+
1

1

+
.

.
.

.
.

.
=

=
π2
12
π2
12 32 32 8
1 1 1 π4
(iv). + + + ... =
14 24 34 90
Solution:
Since f (x) is an even function,
bn = 0.
The Fourier series of f (x) is


a0 X
f (x) = + an cos nx.................(1)
2 n=1

Page 14

Downloaded From : www.EasyEngineering.net


Downloaded From : www.EasyEngineering.net Umar Ali

To find a0


1
a0 = f (x)dx
π
−π

1
= 2 x2 dx
π
0
 3 π
2 x
=
π 3 0
2 3
= [π ]

2π 2
=
3

To find an

ne t
an =
1
π

f (x) cos nxdx

n g .
2
−π

e ri
=
π
0
2
x cos nxdx

i n e
ng

2 2 sin nx (− cos nx) (− sin nx)
= x − (2x) 2
+2
π n n n3

y E
2 2 sin nx cos nx sin nx
π 0

as
= x + (2x) 2
−2 3
π n n n
 0

E
n

2 (−1)

w . =
π
0 + 2π 2 − 0 − {0 + 0 − 0}

4(−1) n
n

w w =
n2

Substituting in (1),


2π 2 X 4(−1)n
⇒ f (x) = + cos nx
3 × 2 n=1 n2

π2 X (−1)n
= +4 cos nx
3 n=1
n2

Deduction:
(i). Here x = π is a point of continuity.

Page 15

Downloaded From : www.EasyEngineering.net


Downloaded From : www.EasyEngineering.net Umar Ali


π2 X (−1)n
f (π) = +4 2
(−1)n
3 n=1
n

2 π2 X 1
π − = 4
3 n=1
n2

2π 2 X 1
= 4
3 n=1
n2
π2 1 1 1
= 2 + 2 + 2 + . . . .....(3)
6 1 2 3

(ii). Here x = 0 is a point of continuity.


π2 (−1)n

t
X
f (0) = +4

π2
3
X (−1)n
n=1

n2

. ne
0−
3
= 4
n=1
n2

i n g

π2
12
=
(−1)
12
+
1
22
+
(−1)
32
e
+ ...

e r
π2
n

1 1 1

12
π 2
1
1

ng
1
i
= − 2 − 2 + 2 − ...
2
1
3

= 2 − 2 + 2 − . . . .....(4)

y E 12 1 2 3

(iii). (3) + (4)

. E as
w π2 π2
 
1 1 1
2 2 + 2 + 2 + ... + =

w w 1 3 5

=
6

12
2
12

π2
=
4
1 1 1 π2
+ + + ... =
12 32 52 8

(iv). By parseval’s identity,

Zπ ∞
1 2 a20 1 X 2
[f (x)] dx = + (an + b2n )
2π 4 2 n=1
−π
Zπ 4π 4 ∞
1 4 9 1 X 16
x dx = +
2π 4 2 n=1 n4
−π

Page 16

Downloaded From : www.EasyEngineering.net


Downloaded From : www.EasyEngineering.net Umar Ali

Zπ ∞
2 4 π4 X 1
x dx = +8
2π 9 n=1
n4
0
 π ∞
1 x5 4π 4 X 1
= +8
π 5 0 9 n=1
n4

1 π5 π4
  X 1
= +8
π 5 9 n=1
n4

π4 π4 X 1
= +8
5 9 n=1
n4

X 1 π4 π4
8 = −
n=1
n4 5 9

X 1 4π 4
8 =
n=1
n4 45
1
+
1
+
14 24 34
1
+ . . . =
π4
90

ne t
n g .
9. Obtain the Fourier series to represent the function f (x) = |x|, , −π < x < π and deduce

P 1
=
π2
.
e ri
e
2
0 (2n − 1) 8
Solution:

i n
ng
Since f (x) is an even function⇒ bn = 0.

E
The Fourier series of f (x) is

y
. E as f (x) =
a0 X
2
+
n=1

an cos nx.................(1)

w
To find a0 w
w a0 =
1
π

f (x)dx
−π

2
= xdx
π
0

2 x2

=
π 2 0
1 2
= [π ]
π
= π

Page 17

Downloaded From : www.EasyEngineering.net


Downloaded From : www.EasyEngineering.net Umar Ali

To find an


1
an = f (x) cos nxdx
π
−π

2
= x cos nxdx
π
0 π
2 sin nx (− cos nx)
= x − (1)
π n n2 0
 π
2 sin nx cos nx
= x +
π n n2 0
(−1)n
  
2 1
= 0+ − 0+ 2
π n2 n
2
= [(−1)n − 1]
πn2
 −4
, when ’n’ is odd;
ne t
.

= n2 π

0, when ’n’ is even.

i n g
Substituting in (1),

e e r
π
n
−4
X
⇒ f (x) = +
i
2 n=1,3,5,... n2 π

ng
cos nx

y E
as

π 4 X 1
⇒ f (x) = − cos nx

w . E 2 π n=1,3,5,...
n2

w w
Deduction:
Here x = 0 is a point of continuity.


π 4 X 1
f (0) = − cos 0
2 π n=1,3,5,...
n2

π 4 X 1
0 = −
2 π n=1,3,5,...
n2

4 X 1 π
2
=
π n=1,3,5,...
n 2

X 1 π2
=
n=1,3,5,...
n2 8
1 1 1 π2
+ + + . . . =
12 32 52 8

Page 18

Downloaded From : www.EasyEngineering.net


Downloaded From : www.EasyEngineering.net Umar Ali

10. Find the Fourier series of f (x) = x in (−π, π).

11. Find the Fourier series of f (x) = x + x2 in (−π, π) of periodicity 2π


Solution:
Given: f(x) is neither odd nor even.
The Fourier series of f (x) is

∞ ∞
a0 X X
f (x) = + an cos nx + bn sin nx.................(1)
2 n=1 n=1

To find a0


1
a0 =
π
−π
f (x)dx

ne t
=
1
π

(x + x2 )dx

n g .
1
 π
Z
−π

1

e ri
=
π
−π
xdx +

i nπ
−π
x2 dx
e
ng
 π

Z
1
= 0 + 2 x2 dx

y E π
0

as
 3 π
2 x
=

w . E =
π 3 0
2 3

[π ]

w w =
2π 2
3

To find an


1
an = f (x) cos nxdx
π
−π

1
= (x + x2 ) cos nxdx
π
−π
 π 
Z Zπ
1
= x cos nxdx + x2 cos nxdx
π
−π −π

Page 19

Downloaded From : www.EasyEngineering.net


Downloaded From : www.EasyEngineering.net Umar Ali

 

1
= 0 + 2 x2 cos nxdx
π
0
 π
2 2 sin nx (− cos nx) (− sin nx)
= x − (2x) +2
π n n2 n3 0
 π
2 2 sin nx cos nx sin nx
= x + (2x) −2 3
π n n2 n
 n
 0
2 (−1)
= 0 + 2π 2 − 0 − {0 + 0 − 0}
π n
n
4(−1)
=
n2

To find bn


1
bn =
π
−π
f (x) sin nxdx

ne t
=
1

(x + x2 ) sin nxdx
n g .
π
 π
−π

e ri 

e
Z
1
x sin nxdx + x2 sin nxdx
=
π
i n
ng
−π −π
 π 
Z
1

y E =
π
2 x sin nxdx + 0

as
 0 π
2 (− cos nx) (− sin nx)
= x − (1)

w . E =
π
2
π

−x
n
cos nx sin nx
n
+
n2 0

n2 0

w w =
2
π

−π

2(−1)n
(−1)n
n
− 0 − {0 − 0}


= −
n

Substituting in (1),

∞ ∞
2π 2 X 4(−1)n X 2(−1)n
⇒ f (x) = + cos nx − sin nx
3 × 2 n=1 n2 n=1
n
∞ ∞
π2 X (−1)n X (−1)n
= +4 cos nx − 2 sin nx
3 n=1
n2 n=1
n

Note:

Page 20

Downloaded From : www.EasyEngineering.net


Downloaded From : www.EasyEngineering.net Umar Ali


 φ1 (x), −π ≤ x ≤ 0;
Let f (x) =
 φ (x), for 0 ≤ x ≤ π.
2

If φ1 (−x) = φ2 (x) then f (x) is even.


If φ1 (−x) = −φ2 (x) then f (x) is odd.

 1 + 2x , −π ≤ x ≤ 0;

8

cos 3x cos 5x

12. If f (x) = π show that f (x) = cos x + + + ...
2x π 32 52
 1 − , for 0 ≤ x ≤ π.

π

P 1 π2
Deduce that 2
= .
1 (2n − 1) 8
Solution:
Given:

φ1 (x) = 1 +
2x

ne t
φ1 (−x) = 1 −
π
2x
π
n g .
= φ2 (x)

e ri
⇒ f(x) is an even function ⇒ bn = 0.

i n e
The Fourier series of f (x) is

E ng
as yf (x) =
a0 X
+

an cos nx.................(1)

To find a0
w . E 2 n=1

w w a0 =
1

f (x)dx
π
−π

 
2 2x
= 1− dx
π π
0

2x2

2
= x−
π 2π 0
2
= [π − π]
π
= 0

Page 21

Downloaded From : www.EasyEngineering.net


Downloaded From : www.EasyEngineering.net Umar Ali

To find an


1
an = f (x) cos nxdx
π
−π
Zπ  
2 2x
= 1− cos nxdx
π π
0
    π
2 2x sin nx 2 (− cos nx)
= 1− − −
π π n π n2 0
  π
2 2x sin nx 2 cos nx
= 1− −
π π n π n2 0
2(−1)n
  
2 2
= − − −
π πn2 πn2
2 2
= [1 − (−1)n ]

=
π πn2
4
[1 − (−1)n ]
ne t
π n2
2
 8
 , when ’n’ is odd;
n g .
=

π 2 n2
0, when ’n’ is even.
e ri
i n e
ng
Substituting in (1),

X 8
⇒ f (x) = cos nx

y E n=1,3,5,...
π 2 n2

. E as 8

1

w
X
⇒ f (x) = cos nx
π2 n2

w w
Deduction:
n=1,3,5,...

Here x = 0 is a point of continuity.


8 X 1
1 =
π2 n=1,3,5,...
n2

X 1 π2
=
n=1,3,5,...
n2 8
1 1 1 π2
+ + + . . . =
12 32 52 8

13. Find the Fourier series expansion of the periodic function f (x) of the period 2π defined by

Page 22

Downloaded From : www.EasyEngineering.net


Downloaded From : www.EasyEngineering.net Umar Ali


 π + x, for −π ≤ x < 0; ∞
P 1 π2
f (x) = Deduce that 2
= .
 π − x, for 0 < x ≤ π. 1 (2n − 1) 8
Solution:
Given:

φ1 (x) = π + x

φ1 (−x) = π − x

= φ2 (x)

⇒ f (x) is an even function⇒ bn = 0.

The Fourier series of f (x) is

ne t
.

a0 X
f (x) =
2
+
n=1
an cos nx.................(1)

i n g
To find a0

e e r
i n

ng
1
a0 = f (x)dx
π

y E −π

as
2
= (π − x)dx
π

w . E =
2
π
0

πx −
x2

2 0

w w =
2
π
1

2
π −
 2
π
π2
2
−0−0


=
π 2
= π

To find an


1
an = f (x) cos nxdx
π
−π

2
= (π − x) cos nxdx
π
0

Page 23

Downloaded From : www.EasyEngineering.net


Downloaded From : www.EasyEngineering.net Umar Ali

 π
2 sin nx (− cos nx)
= (π − x) − (−1)
π n n2 0
 π
2 sin nx cos nx
= (π − x) −
π n n2 0
(−1)n
  
2 1
= 0− − 0− 2
π n2 n
2
= 2
[1 − (−1)n ]
πn
 4
 , when ’n’ is odd;
= n2 π
0, when ’n’ is even.

Substituting in (1),

π X 4
⇒ f (x) = + cos nx
2 n=1,3,5,... n2 π

π 4

1
ne t
.
X
⇒ f (x) = + cos nx
2 π n=1,3,5,...
n2 π

i n g
Deduction:
Here x = 0 is a point of discontinuity at the mid point.
e e r
i n
ng

f (0−) + f (0+) π 4 X 1
= f (x) = + cos 0
2 2 π n2

s y E π+π
=
π 4
+

X
n=1,3,5,...

.E a 2
π
2 π
4

X
n=1,3,5,...

1
n2

w w 4

π−
2
=

1
π
π
n=1,3,5,...
n2

w
X
2
=
π n=1,3,5,...
n 2
1 1 1 π2
+ + + ... =
12 32 52 8


 1 − x, for −π ≤ x ≤ 0;
14. If f (x) = find the Fourier series of f (x) and hence deduce that
 1 + x, for 0 ≤ x ≤ π.
1 1 1
2
+ 2 + 2 + . . ..
1 3 5


 −1 + x, for −π ≤ x ≤ 0;
15. Obtain the Fourier series expansion of f (x) given by f (x) =
 1 + x, for 0 ≤ x ≤ π.

Page 24

Downloaded From : www.EasyEngineering.net


Downloaded From : www.EasyEngineering.net Umar Ali

1 1 1 π2
and hence deduce that + + + . . . = .
12 32 52 8
Solution:
Given:

φ1 (x) = −1 + x

φ1 (−x) = −1 − x

= −(1 + x)

= −φ2 (x)

⇒ f(x) is an odd function ⇒ a0 = an = 0.

The Fourier series of f (x) is

ne t
f (x) =

X
bn sin nx.................(1)

n g .
n=1

e ri
To find bn

i n e
ng

1
bn = f (x) sin nxdx

s y E π
−π

.E a =
2
π
0
(1 + x) sin nxdx

w w =
2
π
(1 + x)
(− cos nx)
n
− (1)
(− sin nx)


n2 0

w

2 cos nx sin nx
= −(1 + x) +
π n n2 0
(−1)n
  
2 1
= −(1 + π) +0− − +0
π n n
2
= [−(1 + π)(−1)n + 1]

Sub in (1)

X 2
⇒ f (x) = [−(1 + π)(−1)n + 1] sin nx
n=1

16. Express f (x) = x sin x as a Fourier series in (−π, π).


Solution:

Page 25

Downloaded From : www.EasyEngineering.net


Downloaded From : www.EasyEngineering.net Umar Ali

Given:

f (x) = x sin x

f (−x) = −x sin(−x)

= x sin x

= f (x)

⇒ f(x) is an even function ⇒ bn = 0.

The Fourier series of f (x) is


a0 X
f (x) = + an cos nx.................(1)
2 n=1

ne t
To find a0

n g .
a0 =
1

f (x)dx
e ri
π
−π

i n e
ng
2
= x sin xdx
π

y E 2
0

as
= [x(− cos x) − (1)(− sin x)]π0
π

w . E =

=
2
π
2
[−x cos x + sin x]π0

[−π(−1) − 0 − {−0 − 0}]

w w =
π
2

To find an


1
an = f (x) cos nxdx
π
−π

2
= x sin x cos nxdx
π
0

2
= x [sin (1 + n)x + sin (1 − n)x] dx

0

Page 26

Downloaded From : www.EasyEngineering.net


Downloaded From : www.EasyEngineering.net Umar Ali


1
= x [sin (1 + n)x − sin (n − 1)x] dx
π
0    π
1 − cos(n + 1)x − cos(n − 1)x − sin(n + 1)x sin(n − 1)x
= x − − (1) +
π n+1 n−1 (n + 1)2 (n − 1)2 0
 π
1 x cos(n + 1)x x cos(n − 1)x sin(n + 1)x sin(n − 1)x
= − + + −
π n+1 n−1 (n + 1)2 (n − 1)2 0
π(−1)n+1 π(−1)n−1
 
1
= − +
π n+1 n−1
 
1 n+1 1 1
= π(−1) − +
π n+1 n−1
 
n+1 −(n − 1) + n + 1
= (−1)
(n + 1)(n − 1)
n+1
2(−1)
= provided n 6= 1
(n2 − 1)

To find a1
ne t

n g .
a1 =
1
π
f (x) cos xdx

e ri
2
−π

i n e
ng
= x sin x cos xdx
π
0

s
2

y
π E Zπ
sin 2x
x
2
dx

.E a =
1
π
0 
x
− cos 2x
2

− (1)

− sin 2x
4
π

w w =
1
π


x cos 2x sin 2x
2
+
4
π 0

w
0
1 h π i
= −
π 2
1
= −
2

Substituting in (1)
1 P∞ 2(−1)n+1
⇒ f (x) = 1 − cos x + π sin x + 2
cos nx.
2 n=2 n − 1

17. Find theFourier series expansion of the periodic function f (x) of the period 2π defined by
 −k, for −π < x < 0; 1 1 π
f (x) = Deduce that 1 − + + . . . = .
 k, for 0 < x < π. 3 5 4
Solution:

Page 27

Downloaded From : www.EasyEngineering.net


Downloaded From : www.EasyEngineering.net Umar Ali

Given:

φ1 (x) = −k

φ1 (−x) = −k

= −φ2 (x)

⇒ f (x) is an odd function,a0 = an = 0.

The Fourier series of f (x) is


X
f (x) =
n=1
bn sin nx.................(1)

ne t
To find bn
n g .

e ri
bn =
1
π
i nf (x) sin nxdx
e
ng
−π

2

y E =
π
k sin nxdx

as
0

2k

w . E =
π
0
2k − cos nx

sin nxdx

w w =

=
π
−2k

n 0

[(−1)n − 1]
 4k
 , when ’n’ is odd;
= nπ
0, when ’n’ is even.

Substituting in (1),

X 4k
⇒ f (x) = sin nx
n=1,3,5,...

Deduction:

Page 28

Downloaded From : www.EasyEngineering.net


Downloaded From : www.EasyEngineering.net Umar Ali

π
Here x = 2
is a point of continuity.

π  ∞
4k 1 X
f = sin nx
2 π
n=1,3,5,...
n
 
4k π 1 3π 1 5π
k = sin + sin + sin ...
π 2 3 2 5 2
π 1 1
= 1 + (−1) + (1) + . . .
4 3 5
1 1 π
1 − + − ... =
3 5 4


 −1, for −π < x < 0;
18. If f (x) = and f (x + 2π) = f (x) for all x, find the Fourier series for
 1, for 0 < x < π.
f(x).

ne t
 −π, for −π < x < 0;
19. Obtain the Fourier series of the periodic function defined by f (x) =
 x,

n g .
for 0 < x < π.

Deduce that
1
+
1
+
12 32 52
1
+ . . . =
π2
8
e ri
i n
20. Find the Fourier expansion of f (x) = x in the interval (−π, π). e
ng

 0, for −π ≤ x ≤ 0;

y E
21. Find the Fourier series expansion of f (x) =
 sin x, for 0 ≤ x ≤ π.
Deduce that

. E
(i)

(ii)
1

1 as
+


1

1
+

+
1
1·3 3·5 5·7
1
− ...∞
1
+ ...∞ = .
2
π−2
.

w
Solution: w 1·3 3·5 5·7 4

wThe Fourier series of f (x) is

a0 X
∞ X∞
f (x) = + an cos nx + bn sin nx.................(1)
2 n=1 n=1

To find a0


1
a0 = f (x)dx
π
−π

1
= sin xdx
π
0

Page 29

Downloaded From : www.EasyEngineering.net


Downloaded From : www.EasyEngineering.net Umar Ali

1
= [− cos x]π0
π
−1
= [−1 − 1]
π
2
=
π

To find an


1
an = f (x) cos nxdx
π
−π

1
= sin x cos nxdx
π
0

t
1
= [sin (1 + n)x + sin (1 − n)x] dx

0

. ne
=
1

[sin (1 + n)x − sin (n − 1)x] dx

i n g
=
0
1 − cos(n + 1)x − cos(n − 1)x

e e r


n+1
1 − cos(n + 1)x cos(n − 1)x
n−1

i n π 0

ng
= +
2π n+1 n−1 0

E n+1 n−1
  
1 −(−1) (−1) −1 1
=



as y
n+1
1 −(−1)n+1 (−1)n+1
+
n−1

n+1 n−1
1
+

1


w . E =

=

1

n+1

(−1)n+1

+
n−1
+
n+1 n−1
−(n − 1) + (n + 1)


+
n − 1 − (n + 1)


w w =

1


(−1)n+1 2
2
(n + 1)(n − 1)

+ 2
−2
(n − 1) (n − 1)

(n + 1)(n − 1)

1 2
= [(−1)n+1 − 1]
2π (n2 − 1)
1
= 2
[(−1)n+1 − 1]
(n − 1)π

 0,
 when ’n’ is odd;
= −2 provided n 6= 1

 2 , when ’n’ is even.
(n − 1)π

To find a1


1
a1 = f (x) cos xdx
π
−π

Page 30

Downloaded From : www.EasyEngineering.net


Downloaded From : www.EasyEngineering.net Umar Ali


1
= sin x cos xdx
π
0

1 sin 2x
= dx
π 2
0
 π
1 − cos 2x
=
2π 2 0
−1
= [1 − 1]

= 0

To find bn


bn =
1
π
f (x) sin nxdx

ne t
.
−π

=
1
π

sin x sin nxdx

i n g
1
0

e e r
=

i n
[cos (n − 1)x − cos (n + 1)x]

ng
0 π
1 sin(n − 1)x sin(n + 1)x
= −

= 0
n−1

s y E
n+1 0
provided n 6= 1

To find b1
.E a
w w 1

w b1 =
π

1
−π

f (x) sin xdx

= sin x sin xdx


π
0

1
= sin2 xdx
π
0

1 1 − cos 2x
= dx
π 2
0
 π
1 sin 2x
= x−
2π 2 0
1
= [π]

Page 31

Downloaded From : www.EasyEngineering.net


Downloaded From : www.EasyEngineering.net Umar Ali

1
=
2

Substituting in (1)


1 2 X 1 1
⇒ f (x) = − 2
cos nx + sin x
π π n=2,4,... n − 1 2

1 2 X 1 1
f (x) = − cos nx + sin x
π π n=2,4,... (n − 1)(n + 1) 2

Deduction:
Here x = 0 is a point of continuity.

1

2 X 1 1

ne t
f (0) = −
π π n=2,4,... (n − 1)(n + 1)

n g .
(1) + (0)
2

0 =
1

2 X

e
1

ri
π π n=2,4,... (n − 1)(n + 1)
2 X

1
π n=2,4,... (n − 1)(n + 1)
=
1
π
i n e
1
+
1
+
1
1·3 3·5 5·7
E
+ ...∞ =
1
2 ng
π
as y
Here x = is a point of continuity.
2

w . E π  1 2 X

1 nπ 1

w w 1−
f

1 1

2
=

= −
2

π π n=2,4,... (n − 1)(n + 1)

1
(−1) +
1
(1) +
cos

1
2
+ (1)
2

(−1) + . . .


2 π π 1·3 3·5 5·7


 
1 1 2 1 1 1
− = − + − ...
2 π π 1·3 3·5 5·7
π−2π 1 1 1
= − + − ...
2π 2 1·3 3·5 5·7
1 1 1 π−2
− + − ...∞ =
1·3 3·5 5·7 4

Page 32

Downloaded From : www.EasyEngineering.net


Downloaded From : www.EasyEngineering.net Umar Ali

l Type

 x, for 0 < x ≤ l; 1
22. Find the Fourier series expansion of f (x) = Deduce that 2 +
 2l − x, for l ≤ x < 2l. 1
1 1 π2
+ + . . . ∞ = .
32 52 8
Solution:
The Fourier series of f (x) is

∞ ∞
a0 X nπx X nπx
f (x) = + an cos + bn sin .................(1)
2 n=1
l n=1
l

To find a0

1
Z2l

ne t
.
a0 = f (x)dx
l
0
 l
Z2l

i n g
r
Z
1
= xdx + (2l − x)dx
l
0 l

n e e
i
"  2l #
l
x2 x2

1

ng
= + 2lx −
l 2 0 2 l

E
"  #
l
l2 2
 
1 1

as y =

1
l 2 0
 2 
l
+ 4l2 − 2l2 − 2l2 −

l2

2

w . E =

=
l
1 2
[l ]
2
+ 2l2 − 2l2 +
2

w w = l
l

To find an

Z2l
1 nπx
an = f (x) cos dx
l l
0
 l 
Z Z2l
1 nπx nπx 
= x cos dx + (2l − x) cos dx
l l l
0 l
 !l !2l 
nπx nπx nπx nπx
1  sin l (− cos l ) sin (− cos l )
= x nπ − (1) 2 + (2l − x) nπ l − (−1) 
l nπ nπ 2
 
l l 0 l l l

Page 33

Downloaded From : www.EasyEngineering.net


Downloaded From : www.EasyEngineering.net Umar Ali

" l  2l #
1 l nπx l2 nπx l nπx l2 nπx
= x sin + 2 2 cos + (2l − x) sin − 2 2 cos
l nπ l nπ l 0 nπ l nπ l l
2 2 2 2
    
1 l l l l
= 0 + 2 2 (−1)n − 0 + 2 2 + 0 − 2 2 − 0 − 2 2 (−1)n
l nπ nπ nπ nπ
2
1 l
= [2(−1)n − 2]
l n2 π 2
2l
= [(−1)n − 1]
n2 π 2

 0, when ’n’ is even;
= −4l
 , when ’n’ is odd.
n2 π 2

To find bn

Z2l
bn =
1
l
f (x) sin
nπx
l
dx

ne t
1
0
 l
Z
nπx
Z2l
nπx 

n g .
=
l
0
x sin
l
dx + (2l − x) sin
l
l
dx

e ri
e
 !l !2l 

n
nπx nπx nπx nπx
1  (− cos l ) (− sin l ) (− cos l ) (− sin l )
=
l
"
x nπ
l
− (1)

l2

l
 2

ng i 
0
l
+ (2l − x)



l
− (−1)

l2
nπ 2
l



l
2l

#
=
1
l
−x
l

cos
nπx
l
E
+ 2 2 sin

y nπ
nπx
l
+ −(2l − x)
l

cos
nπx
l
− 2 2 sin

nπx
l

as
0 l
  
1 l l
= −l (−1)n + 0 − {0 + 0} − 0 − 0 − −l (−1)n − 0

=
l
0
w . E
nπ nπ

w w
Substituting in (1)


l X −4l nπx
⇒ f (x) = + 2 2
cos
2 n=odd n π l

l 4l X cos nπx
l
= − 2 2
2 π n=1,3,5,...
n

Deduction:
Here x = 0 is a point of discontinuity at the end point.


f (0) + f (2π) l 4l X 1
= − 2 2
(1)
2 2 π n=1,3,5,...
n

Page 34

Downloaded From : www.EasyEngineering.net


Downloaded From : www.EasyEngineering.net Umar Ali


0+0 l 4l 1 X
= − 2
2 2 π n=1,3,5,...
n2
 
l 4l 1 1 1
0 = − + + + ...
2 π 2 12 32 52
 
4l 1 1 1 l
2 2
+ 2 + 2 + ... =
π 1 3 5 2
1 1 1 π2
+ + + . . . =
12 32 52 8


 l − x, for 0 < x < l; ∞
P 1
23. Find the Fourier series expansion of f (x) = Deduce that 2
.
 0, for l < x < 2l. 0 (2n + 1)
Solution:
The Fourier series of f (x) is

a0 X

nπx X

nπx
ne t
f (x) =
2
+
n=1
an cos
l
+
n=1
bn sin
l
.................(1)

n g .
To find a0

e ri
1
i n
Z2l e
ng
a0 = f (x)dx
l
0

y E 1
Zl

as
= (l − x)dx
l

E
0

.
l
x2

1
= lx −

w w =
l
1 2 l2

2 0

l − − {0 − 0}


w =
l
 2
1 l
l 2
2

l
=
2

To find an

Z2l
1 nπx
an = f (x) cos dx
l l
0
Zl
1 nπx
= (l − x) cos dx
l l
0

Page 35

Downloaded From : www.EasyEngineering.net


Downloaded From : www.EasyEngineering.net Umar Ali

" #l
1 sin nπx (− cos nπx
)
= (l − x) nπ l − (−1) 2l
l l

l 0
 2
l
1 l nπx l nπx
= (l − x) sin − 2 2 cos
l nπ l nπ l 0
l2 l2
  
1
= 0 − 2 2 (−1)n − 0 − 2 2
l nπ nπ
2 2
 
1 l l
= − 2 2 (−1)n + 2 2
l nπ nπ
2
1 l
= [1 − (−1)n ]
l n2 π 2
l
= [1 − (−1)n ]
n2 π 2

 0, when ’n’ is even;
=
 2l , when ’n’ is odd.
n2 π 2

ne t
To find bn

n g .
bn =
1
Z2l
f (x) sin
nπx
dx
e ri
l
0

i n
l
e
ng
Zl
1 nπx
= (l − x) sin dx
l l

y E 0
#l

as
"
1 (− cos nπx
l
) (− sin nπx
)
= (l − x) nπ − (−1) l
nπ 2
l

w . E =
1
l

−(l − x)
l

l

cos
nπx
l
l 2
− 2 2 sin

l

nπx
l 0
l
0

w w =
1
l
l
 
l
0 − 0 − −l (1) − 0



=

Substituting in (1)

l ∞ ∞
2 2l X nπx X l nπx
⇒ f (x) = + 2 2
cos + sin
2 n=odd n π l n=1
nπ l
∞ ∞
l 2l X 1 nπx l X1 nπx
= + 2 2
cos + sin
4 π n=n=1,3,5,...
n l π n=1 n l

Deduction:

Page 36

Downloaded From : www.EasyEngineering.net


Downloaded From : www.EasyEngineering.net Umar Ali

Here x = 0 is a point of discontinuity at the end point.


f (0) + f (2π) l 2l X 1
= + 2 2
(1)
2 4 π n=1,3,5,...
n

l+0 l 2l X 1
= + 2
2 4 π n=n=1,3,5,...
n2

l l 2l X 1
− =
2 4 π2 n=n=1,3,5,...
n2

l 2l X 1
=
4 π2 n=n=1,3,5,...
n2
1 1 1 π2
+ + + . . . =
12 32 52 8

24. Find the Fourier series of periodicity 3 for f (x) = 2x − x2 in 0 < x < 3.

ne t
Solution:

n g .
i
3
Let 2l = 3 ⇒ l =

r
2

The Fourier series of f (x) is

n e e
f (x) =
a0 X
2
+
n=1

an cos
nπx X
l
+
n=1
ng i
bn sin
nπx
l

.................(1)

y E
as
To find a0

w . E a0 =
1
l
Z2l
f (x)dx

w w =
1
3
2
0
Z3
(2x − x2 )dx
0
3
2 x2 x3

= 2 −
3 2 3 0
2
= [9 − 9]
3
= 0

To find an

Z2l
1 nπx
an = f (x) cos dx
l l
0

Page 37

Downloaded From : www.EasyEngineering.net


Downloaded From : www.EasyEngineering.net Umar Ali

Z3
1 nπx
= 3 (2x − x2 ) cos 3 dx
2 2
0
Z3
2 2nπx
= (2x − x2 ) cos dx
3 3
0
" #3
2nπx 2nπx 2nπx
2 sin (− cos ) (− sin )
= (2x − x2 ) 2nπ3 − (2 − 2x)  3 + (−2) 3
3 2nπ 2 2nπ 3
3 3 3 0
"  2  3 #3
2 3 2nπx 3 2nπx 3 2nπx
= (2x − x2 ) sin + (2 − 2x) cos +2 sin
3 2nπ 3 2nπ 3 2nπ 3
0
"  2 (  2 )#
2 3 3
= 0−4 +0− 0+2 +0
3 2nπ 2nπ
 
2 9 9
= − 2 2− 2 2

=
3
2


36
− 2 2− 2 2
2n π
18


ne t
=
3
2

4n π

− 2 2
54

4n π

n g .
3
9
4n π

e ri
= − 2 2

i n e
To find bn

E ng
bn =
1
l
Z2l
f (x) sin
as
nπx
l y
dx

=
1
0
Z3

w . E (2x − x2 ) sin
nπx
dx

w
3 3
2 2
0

w =
1
3
2
Z3

0
(2x − x2 ) sin
2nπx
3
dx

" #3
2nπx
2 2 (− cos 3 ) (− sin 2nπx ) cos 2nπx
= (2x − x ) 2nπ − (2 − 2x) 3 + (−2) 2nπ 3 3
2nπ 2
3 3 3 3 0
"  2  3 #3
2 3 2nπx 3 2nπx 3 2nπx
= −(2x − x2 ) cos + (2 − 2x) sin −2 cos
3 2nπ 3 2nπ 3 2nπ 3
0
  
2 3 27 27
= −(−3) +0−2 3 3 − 0+0−2 3 3
3 2nπ 8n π 8n π
 
2 9
=
3 2nπ
3
=

Page 38

Downloaded From : www.EasyEngineering.net


Downloaded From : www.EasyEngineering.net Umar Ali

Sub in (1)

∞ ∞
X −9 nπx X 3 nπx
⇒ f (x) = 2 2
cos 3 + sin 3
n=1
nπ 2 n=1
nπ 2
∞ ∞
9 X 1 2nπx 3 X 1 2nπx
= − 2 2
cos + sin
π n=1 n 3 π n=1 n 3

25. Find the Fourier series of periodicity 3 for f (x) = 2x − x2 in 0 < x < 2.


 x, for 0 ≤ x ≤ 3;
26. Find the Fourier Series for f (x) =
 6 − x, for 3 ≤ x ≤ 6.

27. Expand f (x) = x − x2 in −l < x < l using Fouries Series.

ne t
Solution:

n g .
Given: f(x) is neither odd nor even.

e ri
The Fourier series of f (x) is

i n e
ng
∞ ∞
a0 X nπx X nπx
f (x) = + an cos + bn sin .................(1)
2

y En=1
l n=1
l

To find a0

. E as Zl

w w a0 =
1
l
f (x)dx

w
−l
Zl
1
= (x − x2 )dx
l
−l
 l 
Z Zl
1
= xdx − x2 dx
l
−l −l
 l

Z
1
= 0 − 2 x2 dx
l
0
 3 l
2 x
= −
l 3 0
2
= − [l3 ]
3l

Page 39

Downloaded From : www.EasyEngineering.net


Downloaded From : www.EasyEngineering.net Umar Ali

2l2
= −
3

To find an

Z1
1 nπx
an = f (x) cos dx
1 l
−1
Zl
1 nπx
= (x − x2 ) cos dx
l l
−l
 l 
Z Zl
1 nπx nπx 
= x cos dx − x2 cos dx
l l l
−l −l
 
Zl
=
1
l
0 − 2 x2 cos
nπx 
l
dx

ne t
.
0

g
" #l
−2 2 sin nπx (− cos nπx
) (− sin nπx
)
=
l
x nπ l − (2x)
l
nπ 2
l
 l +2

ri n l
nπ 3
l 0

=
−2 2 l
"
x sin
nπx
+ 2x
 2

n
l
e
cos e
nπx
−2
 3
l
sin
nπx
#l

i
l nπ l nπ l nπ l
0

ng
2
 
2 l
= − 0 + 2l 2 2 (−1)n − 0 − {0 + 0 − 0}
l nπ
 3
2 2l
y E n


as
= − (−1)
l n2 π 2
4l2

w . E= − 2 2 (−1)n

w w
To find bn

Zl
1 nπx
bn = f (x) sin dx
l l
−l
Zl
1 nπx
= (x − x2 ) sin dx
l l
−l
 l 
Z Zl
1 nπx nπx 
= x sin dx − x2 sin dx
l l l
−l −l
 l 
Z
1 nπx
= 2 x sin dx + 0
l l
0

Page 40

Downloaded From : www.EasyEngineering.net


Downloaded From : www.EasyEngineering.net Umar Ali

" #l
2 (− cos nπxl
) (− sin nπx
)
= x nπ − (1) 2l
l l

l 0
"  2 #l
2 l nπx l nπx
= −x cos + sin
l nπ l nπ l
0
 
2 l
= −l (−1)n − 0 − {0 − 0}
l nπ
2l
= − (−1)n

Substituting in (1),

∞ ∞
2l2 X −4l2
n nπx X −2l(−1)n nπx
⇒ f (x) = − + (−1) cos + sin
3 × 2 n=1 n2 π 2 l n=1
nπ l

t
∞ ∞
l2 4l2 X (−1)n nπx 2l X (−1)n nπx
= − − 2
3 π n=1 n 2
cos
l

π n=1 n
sin
l

. ne
28. Find the Fourier series of f (x) = e−x in (−1, 1).
i n g
Solution:

e e r
Let 2l = 2 ⇒ l = 1
i n
The Fourier series of f (x) is

E ng
f (x) =

a
a0 X
2
s
+
y
n=1

an cos
nπx X
l
+
n=1
bn sin

nπx
l
.................(1)

w
To find a0 .E
w w a0 =
1
l
Z1
f (x)dx
−1
Z1
= e−x dx
−1
1
e−x

=
−1 −1
= − e−1 − e1
 

= e1 − e−1
 

= 2 sinh 1

Page 41

Downloaded From : www.EasyEngineering.net


Downloaded From : www.EasyEngineering.net Umar Ali

To find an

Zl
1 nπx
an = f (x) cos dx
l l
−l
Z1
= e−x cos nπxdx
−1
1
e−x

= [(−1) cos nπx + nπ sin nπx]
(−1)2 + (nπ)2 −1
1  −1 n
 1 n

= e [−(−1) ] − e [−(−1) ]
1 + n2 π 2
(−1)n  1
e − e1

= 2 2
1+n π
(−1)n
= 2 sinh 1
1 + n2 π 2

ne t
To find bn

n g .
1
Zl
nπx
e ri
bn =
l
−l
f (x) sin
l

i
dx

n e
ng
Z1
= e−x sin nπxdx

y E
−1

as
1
e−x

= [(−1) sin nπx − nπ cos nπx]
(−1)2 + (nπ)2

w . E =
1
1 + n2 π 2
 −1
e [−nπ(−1)n ] − e1 [−nπ(−1)n ]

−1


w w =

=
nπ(−1)n  1
1 + n2 π 2
nπ(−1)n
e − e

2 sinh 1
1


1 + n2 π 2

Sub in (1)

∞ ∞
2 sinh 1 X (−1)n X nπ(−1)n
⇒ f (x) = + 2π2
2 sinh 1 cos nπx + 2π2
2 sinh 1 sin nπx
2 n=1
1 + n n=1
1 + n

X (−1)n
= sinh 1 + 2 sinh 1 [cos nπx + nπ sin nπx]
n=1
1 + n2 π 2

29. Find the Fourier series expansion of f (x) = 1 − x2 in the interval (−1, 1)

Page 42

Downloaded From : www.EasyEngineering.net


Downloaded From : www.EasyEngineering.net Umar Ali

Half Range

Formula:
1. Half Range Cosine Series of f(x) in (0, π) is
a0 P ∞
f (x) = + an cos nx
2 0
2 Rπ 2 Rπ
where a0 = f (x)dx, an = f (x) cos nxdx
π0 π0
2. Half Range Sine Series of f(x) in (0, l) is
a0 P ∞
f (x) = + an cos nπx l
2 0
2 Rl 2 Rl
where a0 = f (x)dx, an = f (x) cos nπx
l
dx
l 0 l 0
3. Half Range Sine Series of f(x) in (0, π) is
P∞
f (x) = bn sin nx
0

ne t
where bn =
2 Rπ
π0
f (x) sin nxdx

n g .
4. Half Range Sine Series of f(x) in (0, l) is

e ri
f (x) = bn sin nπx
e
P
l

2 Rl
i
0

n
ng
where bn = f (x) sin nπx
l
dx
l 0

E
30. Find the Fourier series expansion of f (x) = x(π −x) over the interval (0, π) as a Fourier cosine

y
series of period 0 π 0 .
Deduce 1
14
+
. E
1
24
+ 1
34
as
+ ... = π4
90
.
Solution:

w w
The Fourier series of f (x) is

w f (x) =
a0 X
+

an cos nx.................(1)
2 n=1

To find a0


2
a0 = f (x)dx
π
0

2
= (πx − x2 )dx
π
0

Page 43

Downloaded From : www.EasyEngineering.net


Downloaded From : www.EasyEngineering.net Umar Ali

 2 π
2 x x3
= π −
π 2 3 0
 3 3

2 π π
= −
π 2 3
 3
2 π
=
π 6
π2
=
3

To find an


2
an = f (x) cos nxdx
π
0

2
=
π
0
(πx − x2 ) cos nxdx

ne t
=
2
π
2 sin nx
(πx − x )
n
− (π − 2x)
(− cos nx)
n2
+ (−2)
(− sin nx)

nn3

g .
i
0

r
 π
2 sin nx cos nx sin nx
=
π
2

(πx − x2 )

(−1)
n
n
+ (π − 2x)

n
1
n
2
+2 3

n

e e 0

=
π
2 −π
0 + (−π) 2 + 0 − 0 + π 2 + 0
n
× 2 [(−1)n + 1]
ng in

π

n

y E
as
 0, when ’n’ is odd;
=
 −4 , when ’n’ is even.

w . E n2

w w
Substituting in (1)

π2
3
X∞
−4
⇒ f (x) = + cos nx
2 n=even
n2

π2 X 1
= −4 cos nx
6 n=2,4,6,...
n2

Using Parseval’s identity

Zπ ∞
1 a20 1 X 2
[f (x)]2 dx = + a
π 4 2 0 n
0
Zπ π4 ∞
1 2 2 9 1 X 16
[xπ − x ] dx = +
π 4 2 n=2,4,6,... n4
0

Page 44

Downloaded From : www.EasyEngineering.net


Downloaded From : www.EasyEngineering.net Umar Ali


π4
 
1 2 2 3 4 1 1 1
[x π − 2πx + x ] dx = + 8 4 + 4 + 4 + ...
π 9 2 4 6
0
π
1 2 x3 x 4 x5 π4
   
1 1 1 1
π − 2π + = + 8 4 4 + 4 + 4 + ...
π 3 4 5 0 9 2 1 2 3
5 4
   
π 1 1 1 π 1 1 1 1
− + = + + + + ...
π 3 2 5 9 2 14 24 34
π4 1 1
   
4 10 − 15 − 6 1 1
π = + + + + ...
30 9 2 14 24 34
π4
 
4 1 1 1 1 1
π − = + + + ...
30 9 2 14 24 34
π4
 
1 1 1 1
= + + + ...
180 2 14 24 34
1 1 1 π4
+ + + . . . =
14 24 34 90

ne t
series of period ’l’
n g .
31. Find the Fourier series expansion of f (x) = x(l − x) over the interval (0, l) as a Fourier cosine

Solution:
e ri
The Fourier series of f (x) is

i n e
E
f (x) =
a0
2
+
ng

X
an cos
nπx
l
.................(1)

as y n=1

To find a0

w . E
w w a0 =
2
l
Zl

0
f (x)dx

Zl
2
= (lx − x2 )dx
l
0
l
2 x2 (x)3

= l −
l 2 3 0
 3 3

2 l l
= −
l 2 3
 3
2 l
=
l 6
l2
=
3

Page 45

Downloaded From : www.EasyEngineering.net


Downloaded From : www.EasyEngineering.net Umar Ali

To find an

Zl
2 nπx
an = f (x) cos dx
l l
0
Zl
2 nπx
= (lx − x2 ) cos dx
l l
0
" #l
nπx nπx nπx
2 sin (− cos ) (− sin )
= (lx − x2 ) nπ l − (l − 2x)  l + (−2) l
l nπ 2 nπ 3
l l l 0
"  2  3 #l
2 2 l nπx l nπx l nπx
= (lx − x ) sin + (l − 2x) cos +2 sin
l nπ l nπ l nπ l
0
l2 2
  
2 l
= −l 2 2 (−1)n − 0 + l 2 2 + 0
l nπ nπ
2 l 3
= − 2 2 [(−1)n + 1]

ne t
lnπ
2l2
= − 2 2 [(−1)n + 1]

n g .
n π
 0,

when ’n’ is odd;
e ri
=


−4l 2

n2 π 2
, when ’n’ is even.

i n e
Substituting in (1)
E ng
as y
⇒ f (x) =
l2
+
X∞
−4l2
cos
nπx

w . E =
l2 4l2
− 2

X
2
3 × 2 n=even n π 2 l
cos nπx
l

w w 6 π n=2,4,6,...
n 2

32. Find the half range Fourier cosine series of f (x) = (π − x)2 in (0, π).Hence find the sum of
1 1 1
the series 4 + 4 + 4 + . . .
1 2 3
Solution:
The half range Fourier cosine series of f (x) is


a0 X
f (x) = + an cos nx.................(1)
2 n=1

Page 46

Downloaded From : www.EasyEngineering.net


Downloaded From : www.EasyEngineering.net Umar Ali

To find a0


2
a0 = f (x)dx
π
0

2
= (π − x)2 dx
π
0

2 (π − x)3

=
π 3(−1) 0
2
= [0 − π 3 ]
−3π
2π 2
=
3

To find an

ne t
an =
2
π

f (x) cos nxdx

n g .
2
0

e ri
=
π
0
2
(π − x) cos nxdx

i n e
g

2 sin nx (− cos nx) (− sin nx)

En
= (π − x)2 − 2(π − x)(−1) 2
+ 2(−1)(−1)
π n n n3 0

y
 π
2 sin nx cos nx sin nx
=
π
(π − x)2

a s n
− 2(π − x)
n 2
−2 3
n 0

E
  
2 1

w
=
. π
0 − 0 − 0 − 0 − 2π 2 − 0
 
2 2π
n

w w =

=
π n2
4
n2

Substituting in (1)


π2 X 4
⇒ f (x) = + 2
cos nx
3 n=1
n

By parseval’s identity,

Zπ ∞
1 a20 1 X 2
[f (x)]2 dx = + a
π 4 2 n=1 n
0

Page 47

Downloaded From : www.EasyEngineering.net


Downloaded From : www.EasyEngineering.net Umar Ali

Zπ 4π 4 ∞
1 4 9 1 X 16
(π − x) dx = +
π 4 2 n=1 n4
0
π ∞
1 (π − x)5 4π 4
 X 1
= +8
π −5 0 9 n=1
n4

1  5
 π4 X 1
− 0−π = +8
5π 9 n=1
n4

1 π5 π4 X 1
= +8
π 5 9 n=1
n4

X 1 π4 π4
8 = −
n=1
n4 5 9

X 1 4π 4
8 =
n=1
n4 45
1
+
1
+
14 24 34
1
+ . . . =
π4
90

ne t
n g .
33. Find the half range Fourier cosine series of f (x) = (x − 2)2 in [0, 2].Hence find the sum of the
1 1 1
series 2 + 2 + 2 + . . .
e ri
1
Solution:
3 5

i n e
ng
Here l = 2

E
The half range Fourier cosine series of f (x) is

y
. E as f (x) =
a0 X
2
+
n=1

an cos
nπx
l
.................(1)

w
To find a0 w
w a0 =
2
l
Zl
f (x)dx
0
Z2
2
= (x − 2)2 dx
2
0
2
(x − 2)3

=
3 0
1
= [0 − (−8)]
3
8
=
3

Page 48

Downloaded From : www.EasyEngineering.net


Downloaded From : www.EasyEngineering.net Umar Ali

To find an

Zl
2 nπx
an = f (x) cos dx
l l
0
Z2
2 nπx
= (x − 2)2 cos dx
2 2
0
"  #2
nπx
− cos nπx nπx

2 sin 2 − sin
= (x − 2) nπ − 2(x − 2) 2 +2
nπ 2
2
nπ 3
2 2 2 0
"  2  3 #2
2 nπx 2 nπx 2 nπx
= (x − 2)2 sin + 2(x − 2) cos −2 sin
nπ 2 nπ 2 nπ 2
0
  
16
= 0+0−0− − 2 2

=
16
n2 π 2
ne t
n g .
Substituting in (1)
8
∞ 16 nπx
e ri
⇒ f (x) = 3 +
e
P
cos

n
2
2 n=1 n π 2 2
4 16 P
⇒ f (x) = + 2
Deduction:
∞ 1

3 π n=1 n2
cos
nπx
2
.....(2)

ng i
y E
as
Here x = 0 is a point of continuity.

w . E f (0) = +

4 16 X 1
3 π 2 n=1 n2

w w 4−
4
3
=

8
16 X 1

π 2 n=1 n2
16 X 1

=
3 π 2 n=1 n2
π2 1 1 1
= 2 + 2 + 2 + . . . .....(3)
6 1 2 3

Here x = 2 is a point of continuity.


4 16 X (−1)n
f (2) = +
3 π 2 n=1 n2

4 16 X (−1)n
0− =
3 π 2 n=1 n2

Page 49

Downloaded From : www.EasyEngineering.net


Downloaded From : www.EasyEngineering.net Umar Ali

 
4 16 (−1) 1 (−1)
− = + 2 + 2 + ...
3 π2 12 2 3
2
 
4π 1 1 1
− = − 2 − 2 + 2 − ...
3 16 1 2 3
2
π 1 1 1
= 2 − 2 + 2 − . . . .....(4)
12 1 2 3

(3) + (4)

π2 π2
 
1 1 1
2 2 + 2 + 2 + ... = +
1 3 5 6 12
2

=
12
π2
=
4
1
12
1 1
+ 2 + 2 + ... =
3 5
π2
8
ne t
l 4l

1 1


n g .
2 π
πx

4
3
3πx
5
5πx

e ri
34. Prove that, if 0 < x < l, x = − 2 cos l + 2 cos l + 2 cos l + . . . .

e
1 1 1 π
Also deduce that 4 + 4 + 4 + . . . =
1 3 5 96

i n
ng
Solution:
The half range Fourier cosine series of f (x) is

y E
as

a0 X nπx
f (x) = + an cos .................(1)

To find a0
w . E 2 n=1
l

w w a0 =
2
Zl
f (x)dx
l
0
Zl
2
= xdx
l
0
l
2 x2

=
l 2 0
2 l2
=
l 2
= l

Page 50

Downloaded From : www.EasyEngineering.net


Downloaded From : www.EasyEngineering.net Umar Ali

To find an

Zl
2 nπx
an = f (x) cos dx
l l
0
Zl
2 nπx
= x cos dx
l l
0
"
nπx
 #l
2 sin nπx − cos
= x nπ l − (1) l
nπ 2
l l l 0
 2
l
2 l nπx l nπx
= x sin + 2 2 cos
l nπ l nπ l 0
2 2
  
2 l n l
= 0 + 2 2 (−1) − 0 + 2 2 (1)
l nπ nπ

=
2 l 2

l n2 π 2
[(−1)n − 1]

ne t
=

 0,
−4l
when ’n’ is even;

n g .
Sub in (1)

n2 π 2
, when ’n’ is odd.

e ri
⇒ f (x) =
l
+
X∞
−4l
cos
nπx

i n e
ng
2
2 n=1,3,5,... n π 2 l

E
l 4l 1 nπx
X
= − 2 cos

as
x =
y
l

4l
cos
2 π
n=1,3,5,...

πx
n 2

1
+ 2 cos
l
3πx 1
+ 2 cos
5πx
+ ...


w . E 2 π2 l 3 l 5 l

w
Using Parseval’s identity

w 1
Zl
a2 1 X 2

[f (x)] dx = 0 +
2
a
l 4 2 0 n
0
Zl ∞
1 2 l2 1 X 16l2
[x] dx = +
l 4 2 n=1,3,5,... n4 π 4
0
 l ∞
1 x3 l2 8l2 X 1
= + 4
l 3 0 4 π n=1,3,5,... n4

1 l3 l2 8l2 X 1
 
= + 4
l 3 4 π n=1,3,5,... n4

l2 l2 8l2 X 1
− =
3 4 π4 n=1,3,5,...
n4

Page 51

Downloaded From : www.EasyEngineering.net


Downloaded From : www.EasyEngineering.net Umar Ali


l2 8l2 X 1
=
12 π4 n=1,3,5,...
n4
π4 1 1 1
= 4 + 4 + 4 + ...
96 1 3 5

35. Expand f (x) = sin x, 0 < x < π in a Fourier cosine series.

1 π2
36. Obtain the half range cosine series for f (x) = x in (0, π) and show that 12
+ 312 + 512 + . . . = 8

37. Obtain the Fourier Cosine series Expansion of x sin x in (0, π) and hence find the value of
2 2 2 2
1+ − + −
1·3 3·5 5·7 7·9

t

 x, for 0 ≤ x < π2 ;
38. Find the half range cosine series for the function f(x)is defined as f (x) =

. ne
 π − x, for π ≤ x < π.
2

i n g
P∞ (−1)n
=
π3 1 1 1
or deduce that the value of 3 − 3 + 3 − . . ..
e r
39. Find the half range sine series for a function f (x) = x(π − x), 0 < x < π. Hence deduce

e
n=1 (2n − 1)
3 32 1

i n3 5

ng
Solution:
The Half range Fourier sine series of f (x) is

y E
as

X
f (x) = bn sin nx.................(1)

To find bn
w . E n=1

w w bn =
2

f (x) sin nxdx
π
0

2
= (xπ − x2 ) sin nxdx
π
0 π
2 (− cos nx) (− sin nx) (cos nx)
= (xπ − x2 ) − (π − 2x) 2
+ (−2)
π n n n3 0
 π
2 cos nx sin nx cos nx
= −(xπ − x2 ) + (π − 2x) 2 − 2
π n n n3 0
(−1)n
  
2 1
= 0+0−2 3 − 0−0−2 3
π n n
2 2
= [1 − (−1)n ]
π n3

Page 52

Downloaded From : www.EasyEngineering.net


Downloaded From : www.EasyEngineering.net Umar Ali


 0, when ’n’ is even;
= 8
 , when ’n’ is odd.
n3 π
Sub in (1)

X 8
⇒ f (x) = 3π
sin nx
n=1,3,5,...
n

Deduction:
π
Here x = is a point of continuity.
2

π  ∞
8 X nπ
f = 3
sin
2 n=1,3,5,...
nπ 2
 
π π 8 1 π 1 3π 1 5π
2
π−
2
π2 π
=
π 13
1 1
sin + 3 sin
2 3
1
2
+ 3 sin
5 2
+ ...

ne t
1 1 1
4 8
= 3 − 3 + 3 − ...
1
π 3
3 5

n g .
13
− 3 + 3 − ... =
3 5 32

e ri
i n e 
 kx, for 0 < x < 2l ;

ng
40. Find the half range sine series for the function f(x)is defined as f (x) =
 k(l − x), for l < x < l.
2
Solution:

y E
as
The Fourier series of f (x) is

w . E f (x) =

X
bn sin
nπx
l
.................(1)

w w
To find bn
n=1

Zl
2 nπx
bn = f (x) sin dx
l l
0
 l 
Z 2 Zl 
2 nπx nπx 
= kx sin dx + k(l − x) sin dx
l  2 2 

0 l
2
" # 2l " #l 
nπx nπx nπx nπx
2k  (− cos l ) (− sin l ) (− cos l ) (− sin l ) 
= x nπ − (1) n2 π 2
+ (l − x) nπ − (−1) n2 π 2
l  l l2 l l2 l

0 2
( l  l )
2k l nπx l2 nπx 2 l nπx l2 nπx
= −x cos + 2 2 sin + −(l − x) cos − 2 2 sin
l nπ l nπ l 0 nπ l nπ l l
2

Page 53

Downloaded From : www.EasyEngineering.net


Downloaded From : www.EasyEngineering.net Umar Ali

l2 l2
   nπ 
2k l l  nπ   nπ  l l  nπ 
= − cos + 2 2 sin +0− − cos − 2 2 sin
l 2 nπ 2 nπ 2 2 nπ 2 nπ 2
2
2k 2l  nπ 
= 2 2
sin
l nπ 2
4lk  nπ 
= sin
n2 π 2 2

Substituting in (1)


X 4lk  nπ  nπx
⇒ f (x) = 2 2
sin sin
n=1
nπ 2 l

4lk X 1  nπ  nπx
= sin sin
π 2 n=1 n2 2 l


 x, for 0 < x < 2l ;

ne t
.
41. Find the half range sine series for the function f(x)is defined as f (x) =
 l − x, for l < x < l.

i n
2

g
r

 sin x, for 0 < x < π ;
42. Expand f (x) = 4
 cos x, for π < x < π .
in aa series of sine.

n e e
i
4 2

ng

 x − 1, for 0 ≤ x ≤ 1;
43. Find the half range sine series for the function f(x)is defined as f (x) =

y E  1 − x, for 1 ≤ x ≤ 2.

as
Solution: Here l = 2

w . E
The Fourier series of f (x) is

w nπx
X
f (x) = bn sin .................(1)

wTo find bn
n=1
l

Zl
2 nπx
bn = f (x) sin dx
l l
0
Z2
2 nπx
= f (x) sin dx
2 2
0
Z1 Z2
nπx nπx
= (x − 1) sin dx + (1 − x) sin dx
2 2
0 1
"
nπx
 #1 "  #2
− sin nπx − cos nπx − sin nπx
 
− cos 2
= (x − 1) nπ − (1) 2
nπ 2
+ (1 − x) 2
nπ − (−1) 2
nπ 2
2 2 0 2 2 1

Page 54

Downloaded From : www.EasyEngineering.net


Downloaded From : www.EasyEngineering.net Umar Ali

"  2 #1 "  2 #2
2 nπx 2 nπx 2 nπx 2 nπx
= −(x − 1) cos + sin + −(1 − x) cos − sin
nπ 2 nπ 2 nπ 2 nπ 2
0 1
     
4 nπ 2 2 4 nπ
= 0 + 2 2 sin − +0 + (−1)n − 0 − 0 − 2 2 sin
nπ 2 nπ nπ nπ 2
8 nπ 2
= sin + [(−1)n − 1]
n2 π 2 2 nπ

 0, when ’n’ is even;
=
 8 sin nπ − 4 , when ’n’ is odd.
2
n2 π 2 nπ

Substituting in (1)

∞  
X 8 nπ 4 nπx
⇒ f (x) = sin − sin
n=odd
n2 π 2 2 nπ 2

ne t
.
 
4 πx 1 3πx 1 5πx

g
44. check that for 0 < x < l, 1 = sin l + sin l + sin l + . . .
π 3 5

45. find the half range sine series for f (x) = 1 − x in (0, l)
ri n
46. Show that for 0 < x < l, x =
2l

πx 1

n 2πx 1 3πx
e

e
sin l − sin l + sin l + . . . using root mean square
π
1 1

ng i1
2
value of x, deduce the value of 12 + 22 + 32 + . . .
3

y E
HARMONIC ANALYSIS

. E as
w w The process of finding the Fourier series for a function given by numerical value
is known as Harmonic analysis.

w Formula:f (x) =
a0
2
+ (a1 cos x + b1 sinx) + (a2 cos 2x + b2 sin2x) + . . .

a0 = 2
P 
y
N

P  P 
y cos x y sin x
a1 = 2 b1 = 2
N N

P  P 
y cos 2x y sin 2x
a2 = 2 b2 = 2
N N

.. ..
. .

Page 55

Downloaded From : www.EasyEngineering.net


Downloaded From : www.EasyEngineering.net Umar Ali

Note:1
First Harmonic : a1 cos x + b1 sinx
Second Harmonic : a2 cos 2x + b2 sin2x
Third Harmonic :a3 cos 3x + b3 sin3x.
Note:2
p
Amplitude of First Harmonic A1 = a21 + b21
p
Amplitude of Second Harmonic A2 = a22 + b22

Type : Degree

47. Find the Fourier Series expansion of period 2π for the function y = f (x) which is defined in

π 2π 4π 5π

ne
(0, 2π) b means of the table of values given below. Find the series upto the third harmonic.
t
x 0
3 3
π
3
y = f (x) 1.0 1.4 1.9 1.7 1.5 1.2 1.0
3

n g .
Solution:

e ri
The Fourier Series is given by
i n e
Since the first and the last value of y is same omit the last one.

a0
ng
f (x) = +(a1 cos x+b1 sin x)+(a2 cos 2x+b2 sin 2x)+(a3 cos 3x+b3 sin 3x).....(1)
2
E
x
0 1 0
as y
y y cos x y sin x y cos 2x y sin 2x y cos 3y y sin 3x
1 1 0 1 0
π
3

1.4

w .
0.7E 1.212 −0.7 1.212 −1.4 0

w w 3
π

1.9 −0.95

1.7 −1.7
1.65

0
−0.95

1.7
−1.645

0 −1.7
1.9 0

0
1.5 0.75 −1.299 −0.75 1.299 1.5 0
3

1.2 0.6 −1.039 −0.6 −1.039 −1.2 0
3
P
8.7 −1.1 0.524 −0.3 −0.178 0.1 0

P  P  P  P 
y y cos x y sin x y cos 2x
a0 = 2 a1 = 2 b1 = 2 a2 = 2
N N N N
       
8.7 −1.1 0.5196 −0.3
= 2 = 2 = 2 = 2
6 6 6 6
a0 = 2.9 a1 = −0.367 b1 = 0.175 a2 = −0.1

Page 56

Downloaded From : www.EasyEngineering.net


Downloaded From : www.EasyEngineering.net Umar Ali

P  P  P 
y sin 2x y cos 3x y sin 3x
b2 = 2 a3 = 2 b3 = 2
N N N
   
−0.1732 0.1 = 0
= 2 = 2
6 6
b2 = −0.0593 a3 = 0.033

Sub in (1)
f (x) = 1.45 + (−0.367 cos x + 0.175 sin x) + (−0.1 cos 2x − 0.0593 sin 2x) + 0.033 cos 3x

48. Determine the first two harmonics of the Fourier series for the following data:
π 2π 4π 5π
x 0 π
3 3 3 3
y = f (x) 1.98 1.30 1.05 1.30 -0.88 -0.25

ne t
Ans: f (x) = 0.75 + (0.37 cos x + 1.004 sin x) + (0.877 cos 2x − 0.109 sin 2x)

n g .
Type 2: l

e ri
i n e
49. Find the Fourier series as far as the second harmonic to represent the function given in the
following data.
x 0 1
E 2 3 4 ng5
f(x) 9

as
18 y 24 28 26 20
Solution:

w . E
Here the length of the interval is 6

w w i.e., 2l = 6 ⇒ l = 3
w.k.t The Fourier Series is given by
a0
f (x) = 2
+ (a1 cos πx
l
+ b1 sin πx
l
) + (a2 cos 2πx
l
+ b2 sin 2πx
l
)
a0 πx
f (x) = 2
+ (a1 cos θ + b1 sin θ) + (a2 cos 2θ + b2 sin 2θ), θ = 3
.....(1)

Page 57

Downloaded From : www.EasyEngineering.net


Downloaded From : www.EasyEngineering.net Umar Ali

πx
x θ= 3
y y cos θ y cos 2θ y sin θ y sin 2θ
0 0 9 9 9 0 9
1 60 18 9 −9 15.588 15.588
2 120 24 −12 −12 20.784 −20.784
3 180 28 −28 28 0 0
4 240 26 −13 −13 −22.516 22.516
5 300 20 10 −10 −17.32 −17.32
P
125 −25 −7 −3.464 −.01

a0 = 2
P 
y
a1 = 2
P
y cos πx
3

b1 = 2
ne
P
ty sin πx
3


= 2

N
125

= 2

−25
N


n g .
= 2

N
−3.464


a0 = 41.66
6
a1 = −8.33
6

e ri b1 = −1.15
6

i n e
y cos 2πx
P
3

E

ng P
y sin 2πx
3

a2 = 2

= 2
a
 
−7
N

s y b2 = 2

= 2

N
−0.01


w a2 .E 6
= −2.33 b2
6
= −0.003

w w
Sub in (1)
f (x) = 41.67
+ (−8.33 cos θ − 1.15 sin θ) + (−2.33 cos 2θ − 0.003 sin 2θ), θ = πx
2 3

f (x) = 20.83 − 8.33 cos πx


3
− 1.15 sin πx
3
− 2.33 cos 2πx
3
− 0.003 sin 2πx
3

50. Compute the first harmonic of the Fourier series of f(x)


x 0 1 2 3 4 5
f(x) 4 8 15 7 6 2
Ans : f (x) = 20.83 + (−2.83 cos πx
3
+ 4.33 sin πx
3
)

51. The following table gives the variations of a periodic function over a period T
T T T 2T 5T
x 0 2
T
6 3 3 6
f (x) 1.98 1.3 1.05 1.3 -0.88 -0.25 1.98

Page 58

Downloaded From : www.EasyEngineering.net


Downloaded From : www.EasyEngineering.net Umar Ali

2πx
Show that f (x) = 0.75 + 0.37 cos θ + 1.004 sin θ where θ = .
T
Solution:
Since the first and the last value of y is same omit the last one.
T
i.e., 2l = T ⇒ l =
2
w.k.t The Fourier Series is given by
a0 πx πx
f (x) = + (a1 cos + b1 sin )
2 l l
a0 πx 2πx
f (x) = + (a1 cos θ + b1 sin θ), θ = T = .....(1)
2 2
T
2πx
x θ= T
y y cos θ y sin θ
0 0 1.98 1.98 0
T π
1.30 0.65 1.1258
6 3
T
3

3
1.05 −0.525 0.9093

ne t
T
2
2T
π

1.30 −1.30 0

n g .
3
5T
3

−0.88 0.44

−0.25 −0.125 0.2165


0.762

e ri
P
6 3
4.6
i
1.12
n e
3.013

n g
sy E
a0 = 2

.E a
P 

N
y
a1 = 2
P
y cos θ
N

b1 = 2
P
y sin θ
N


w
     
4.6 1.12 3.013
= 2 = 2 = 2

w wa0 = 1.5
6
a1 = 0.37
6
b1 = 1.004
6

Sub in (1)
f (x) = 0.75 + 0.37 cos θ + 1.004 sin θ

Complex Form of Fourier Series

52. Find the complex form of the Fourier series of f (x) = eax (−π < x < π) in the form.

sin haπ P a + in inx π ∞ (−1)n
eax = (−1)n 2
P
e . And hence prove that =
π −∞ a + n2 a sin haπ −∞ a2 + n2
Solution:

Page 59

Downloaded From : www.EasyEngineering.net


Downloaded From : www.EasyEngineering.net Umar Ali

The complex form of Fourier series is


X
f (x) = cn einx ......(1)
−∞

To find cn

Z π
1
cn = f (x)e−inx dx
2π −π
Z π
1
= eax e−inx dx
2π −π
Z π
1
= e(a−in)x dx
2π −π

1 e(a−in)x

=

=
2π a − in −π
1  (a−in)π
e
ne t
− e−(a−in)π


=
2π(a − in)
1

n g .
a + in  aπ −inπ
e e − e−aπ einπ


e r
a + ini
2π(a − in) a + in
e (−1)n − e−aπ (−1)n
 aπ 

i n
=
e
2
2π[a − (in) ] 2

(a + in)(−1)  aπ

ng
e − e−aπ

= 2 2
2π(a + n )
(a + in)(−1)n

y E =
2π(a2 + n2 )
2 sin haπ

as
a + in
= (−1)n sin haπ

w . E Sub in (1)
π(a2 + n2 )


sin haπ X a + in inx

w w f (x) =

ax
e =
π −∞
(−1)n 2


sin haπ X
a + n 2
e

a + in inx
(−1)n 2 e ...(2)
π −∞
a + n2
put x = 0

sin haπ X a + in
f (0) = (−1)n 2
π −∞
a + n2

sin haπ X a + in
1 = (−1)n 2
π −∞
a + n2

π X a + in
= (−1)n 2
sin haπ −∞
a + n2
Equating the real part

π X a
= (−1)n 2
sin haπ −∞
a + n2

Page 60

Downloaded From : www.EasyEngineering.net


Downloaded From : www.EasyEngineering.net Umar Ali


π X (−1)n
=
a sin haπ −∞
a2 + n 2

53. Find the complex form of the Fourier series e−x in −1 < x < 1 and hence prove that
1 ∞ 1 − inπ
(−1)n
P
= .
sin h1 −∞ 1 + n2 π 2
Solution:
Here 2l = 2 ⇒ l = 1
The complex form of Fourier series is


inπx
X
f (x) = cn e l , ......(1)
−∞

t
To find cn

1 l
Z
−inπx

. ne
cn =
2l −l
f (x)e l dx

1 1 −x −inπx
i n g
r
Z
= e e dx
2 −1
1 1 −(1+inπ)x
Z

n e e
=
2 −1
e

ng
1 e−(1+inπ)x
 i dx
1

E
=
2 −(1 + inπ) −1

as y =
−1
2(1 + inπ)
 −(1+inπ)
e − e(1+inπ)


w . E =
−1 1 − inπ  −1 −inπ
2(1 + inπ) 1 − inπ
−(1 − inπ)  −1
e e − e1 einπ


w e (−1)n − e1 (−1)n

= 2 2
2(1 − (inπ) )

w =
(1 − inπ)
2
2(1 + n π )
(1 − inπ)
2
(−1)n e1 − e−1
 

= (−1)n 2 sin h1
2(1 + n2 π 2 )
1 − inπ
= sin h1(−1)n
1 + n2 π 2

Sub in (1)


X 1 − inπ inπx
f (x) = sin h1 (−1)n 2π2
e
−∞
1 + n

−x
X 1 − inπ inπx
e = sin h1 (−1)n e .....(2)
−∞
1 + n2 π 2

Page 61

Downloaded From : www.EasyEngineering.net


Downloaded From : www.EasyEngineering.net Umar Ali

put x = 0

X 1 − inπ
f (0) = sin h1 (−1)n
−∞
1 + n2 π 2

X 1 − inπ
1 = sin h1 (−1)n
−∞
1 + n2 π 2

X 1 − inπ 1
(−1)n 2 2
=
−∞
1+n π sin h1

Two Marks

 x, for 0 < x < 1;
1. Find the Sum of the Fourier series for f (x) = at x = 1
 2, for 1 < x < 2.
Solution:
Here x = 1 is a point of discontinuity(mid point)

ne t
n
f (1−) + f (1+)
g .
Sum of the Fourier series =
1+2
e ri
2

i n
=
3 e2

ng
=
2

y E
2. Find the Sum of the Fourier series for f (x) = x + x2 in −π < x < π at x = π
Solution:

. E as
Here x = π is a point of discontinuity(end point)

w w f (−π) + f (π)

w Sum of the Fourier series =

=
2
−π + π 2 + π + π 2
2
2

=
2
2
= π

3. Find the Constant term in the expansion of cos2 x as a Fourier Series in the interval (−π, π).
Solution:
Sincef (x) is an even function,bn = 0

Page 62

Downloaded From : www.EasyEngineering.net


Downloaded From : www.EasyEngineering.net Umar Ali

The Fourier series of f (x) is


a0 X
f (x) = + an cos nx.................(1)
2 n=1

To find a0


1
a0 = f (x)dx
π
−π

1
= cos2 dx
π
−π

2 1 + cos 2x
= dx

1
π
0
2

sin 2x

ne t
=
π
1
x+
2

n
0
dx

g .
=
π
(π)

e ri
∴ the constant term =
i n
= 1
a0
= e
1

ng
2 2

y E
4. write down a0 , an in the expansion of x + x3 as Fourier series in (−π, π).
Solution:

. E as
w w f (x) = x + x3

w f (−x) = (−x) + (−x)3

= −x − x3

= −(x + x3 )

f (−x) = f (x)

∴ f (x) is an odd function,⇒ a0 = an = 0.

π2
5. If the Fourier Series of the function f (x) = x + x2 in the interval −π < x < π is 3
+

 
4 2 1 1 1
(−1)n 2 cos nx − sin nx then find the value of the Series 2 + 2 + 2 + · · ·
P
n=1 n n 1 2 3
Solution:

Page 63

Downloaded From : www.EasyEngineering.net


Downloaded From : www.EasyEngineering.net Umar Ali

π2 P ∞
 
n 4 2
Given f (x) = + (−1) cos nx − sin nx
3 n=1 n2 n
Put x = π is a point of discontinuity(end point)


π2 X
 
f (−π) + f (π) n 4 n
= + (−1) 2
(−1) − 0
2 3 n=1
n

−π + π 2 + π + π 2 π2 X 4
= + 2
(−1)2n
2 3 n=1
n

2π 2 π 2 X 1
− = 4
2 3 n=1
n2

2π 2 X 1
= 4
3 n=1
n2
2π 2 1 1 1
= 2 + 2 + 2 + ···
12 1 2 3
1
2
1 1
+ 2 + 2 + ··· =
π 2

ne t
1 2 3 6

n g .
ri
6. Define Root Mean Square value of a function f(x) over the interval (a, b).

e
Solution:

i n
The Root Mean Square value of a function f(x) over e
v the interval (a, b) is

ng
u Rb
u [f (x)]2 dx
u

y E ȳ = a
t
b−a

. E as ȳ 2 =
(or)
1 Rb
b−a a
[f (x)]2 dx

w w
7. Find the Root mean square value of f (x) = π − x in 0 < x < 2π.

wSolution:

1
Z2π
ȳ 2 = [f (x)]2 dx

0
Z2π
1
= [π − x]2 dx

0
2π
(π − x)3

1
=
2π −3 0
(−π) − π 3
3
 
1
=
2π −3
−2π 3
 
1
=
2π −3

Page 64

Downloaded From : www.EasyEngineering.net


Downloaded From : www.EasyEngineering.net Umar Ali

1 2π 3
 
=
2π 3
2
π
=
3
π
ȳ = √
3

8. Find the R.M.S. value of f (x) = 1 − x in 0 < x < 1.


Solution:
The R.M.S value of f(x) in (0, l) is

Zl
2 1
ȳ = [f (x)]2 dx
l
0

=
Z1
[1 − x]2 dx

ne t
0

(1 − x)3
1

n g .
=
−3 0

e ri
e
 
0−1
=

i n −3

ng
 
1
=
3

s y E =
1
3

.E a ȳ = √
1
3

w w
w

Page 65

Downloaded From : www.EasyEngineering.net


Panimalar Engineering College Downloaded From : www.EasyEngineering.net Umar Ali

Applications of PDE

WAVE EQUATION

WITHOUT VELOCITY

1. A string is stretched and fastened to two points l apart. Motion is started by displacing the string into the
form y = k(lx − x2 ) and then released it from this position at time t = 0. Find the displacement of the point
of the string at a distance of x from one end at time t.
Solution:
The wave equation is
∂2y 2
2∂ y
= a
∂t2 ∂x2

The Boundary conditions are:

ne t
(i)y(0, t) = 0 ∀t > 0

n g .
i
(ii)y(l, t) = 0 ∀t > 0

(iii)
∂y(x, 0)
∂t
=0

e e r
0<x<l

i n
(iv)y(x, 0) = k(lx − x2 ) 0<x<l

The Suitable solution is

E ng
Apply condition (i)

as y
y(x, t) = (c1 cos px + c2 sin px)(c3 cos pat + c4 sin pat)........(1)

w . E y(0, t) = c1 (c3 cos pat + c4 sin pat) = 0


⇒ Either c1 = 0 or c3 cos pat + c4 sin pat = 0

w w
Since c3 cos pat + c4 sin pat 6= 0

Sub in (1)
⇒ c1 = 0

y(x, t) = c2 sin px(c3 cos pat + c4 sin pat)........(2)


Apply condition (ii)
y(l, t) = c2 sin pl(c3 cos pat + c4 sin pat) = 0
⇒ Either c2 = 0 or sin pl = 0 or c3 cos pat + c4 sin pat = 0
Since c3 cos pat + c4 sin pat 6= 0 and c2 6= 0[if c2 = 0 we get a trivial solution]
⇒ sin pl = 0 But sin nπ = 0
⇒ pl = nπ

⇒ p=
l
Sub in (2)
 
nπx nπat nπat
y(x, t) = c2 sin c3 cos + c4 sin ........(3)
l l l

Page 1

Downloaded From : www.EasyEngineering.net


Panimalar Engineering College Downloaded From : www.EasyEngineering.net Umar Ali

Diff par w.r.t ’t’


 
∂y(x, t) nπx nπat  nπa  nπat  nπa 
= c2 sin −c3 sin + c4 cos
∂t l l l l l
Apply condition (iii)
∂y(x, 0) nπx h  nπa i
= c2 sin c4 =0
∂t l l
nπx nπa
⇒ Either c2 = 0 or sin = 0 or c4 = 0 or =0
l l
nπx nπa
Since c2 6= 0 , sin 6= 0 and 6= 0
l l
⇒ c4 = 0
Sub in (3)
nπx nπat
y(x, t) = c2 sin c3 cos
l l
The most general form is

P nπx nπat
y(x, t) = cn sin cos ........(4)
n=1 l l
Apply condition (iv)
∞ nπx
= k(lx − x2 )
P
y(x, 0) = cn sin 0<x<l
which is a half range Fourier sine series
n=1 l

ne t
To find cn

n g .
cn =
2
l
Zl
f (x) sin
nπx
l
dx

e ri
e
0

=
2k
Zl
(lx − x2 ) sin
nπx
dx
i n
ng
l l
0
nπx  nπx  nπx l
 

E
 
2k  − cos − sin (cos )

y
= (lx − x )
2 l − (l − 2x)  l2 + (−2)   l 

as
l nπ nπ 3 
l l l 0
3
l3

E
  
2k l n

w . =

=
l
2k
−2 3 3 (−1) − −2 3 3
n π
2l 3
× 3 3 [−(−1)n + 1]
n π

w w =
n

l
4kl2
3 π3
n π
[1 − (−1)n ]
2
 8kl , when ’n’ is odd;

= n π3
3
 0,

when ’n’ is even.

Substituting in (4)
8kl2∞
P nπx nπat
y(x, t) = 3 3
sin cos
n=odd n π l l
8kl2 P∞ 1 nπx nπat
y(x, t) = 3 sin cos
π n=1,3,5,... n3 l l

2. A string is stretched and fastened to two points l apart. Motion is started by displacing the string into the
form y = 3x(l − x) and then released it from this position at time t = 0. Find the displacement of the point of
the string at a distance of x from one end at time t.
Ans: Form the Previous question put k = 3

Page 2

Downloaded From : www.EasyEngineering.net


Panimalar Engineering College Downloaded From : www.EasyEngineering.net Umar Ali

3. A tightly stretched string of length 2l is fastened at both ends. The midpoint of the string is displaced by
a distance ’b’ transversely and the string is released from rest in this position. Find an expression for the
transverse displacement of the string at any time during the subsequent motion.

Solution:
Let 2l = L
First we find the equation of the String.
Equation between two points(x1 , y1 )(x2 , y2 ) is
y − y1 x − x1
=
y2 − y1 x2 − x1

Equantion along AB

y−0 x−0
= L
b−0 2 −0

y
2
= b x
L

ne t
=
2bx
L
0<x<

n
L
2
g .
Equantion along BC

e ri
y−b
=
x−
L
2
i n e
ng
0−b L
L−
2

y E
y−b =

s
−b
L
x−
L
2

.E a y = b−
2b
L
2
x−
L
2


w w = b−
2bx
L
2bx
+b

w =

=
2b −
2b
L
L
(L − x)
L
2
<x<L


 2bx ,
 L
for 0 < x < ;
∴ y(x, 0) = L 2
2b L

 (L − x), for < x < L.
L 2

The wave equation is


∂2y ∂2y
2
= a2 2
∂t ∂x

Page 3

Downloaded From : www.EasyEngineering.net


Panimalar Engineering College Downloaded From : www.EasyEngineering.net Umar Ali

The Boundary conditions are:

(i)y(0, t) = 0 ∀t > 0

(ii)y(L, t) = 0 ∀t > 0
∂y(x, 0)
(iii) =0 0<x<L
∂t 
 2bx ,
 L
for 0 < x < ;
(iv)y(x, 0) = L 2
2b L

 (L − x), for < x < L.
L 2

The Suitable solution is


y(x, t) = (c1 cos px + c2 sin px)(c3 cos pat + c4 sin pat)........(1)
Apply condition (i)
y(0, t) = c1 (c3 cos pat + c4 sin pat) = 0
⇒ Either c1 = 0 or c3 cos pat + c4 sin pat = 0
Since c3 cos pat + c4 sin pat 6= 0

ne t
⇒ c1 = 0

n g .
Sub in (1)

e ri
y(x, t) = c2 sin px(c3 cos pat + c4 sin pat)........(2)

i n e
ng
Apply condition (ii)
y(L, t) = c2 sin pL(c3 cos pat + c4 sin pat) = 0

y E
⇒ Either c2 = 0 or sin pL = 0 or c3 cos pat + c4 sin pat = 0

. ⇒ as
Since c3 cos pat + c4 sin pat 6= 0 and c2 6= 0[if c2 = 0 we get a trivial solution]

E sin pL = 0 But sin nπ = 0

w w ⇒

pL = nπ
p=

L

wSub in (2)
y(x, t) = c2 sin
nπx
L
(c3 cos
nπat
L
+ c4 sin
nπat
L
)........(3)

Diff par w.r.t ’t’


 
∂y(x, t) nπx nπat  nπa  nπat  nπa 
= c2 sin −c3 sin + c4 cos
∂t L L L L L
Apply condition (iii)
∂y(x, 0) nπx h  nπa i
= c2 sin c4 =0
∂t L L
nπx nπa
⇒ Either c2 = 0 or sin = 0 or c4 = 0 or =0
L L
nπx nπa
Since c2 6= 0 , sin 6= 0 and 6= 0
L L
⇒ c4 = 0
Sub in (3)
nπx nπat
y(x, t) = c2 sin c3 cos
L L
The most general form is

Page 4

Downloaded From : www.EasyEngineering.net


Panimalar Engineering College Downloaded From : www.EasyEngineering.net Umar Ali


P nπx nπat
y(x, t) = cn sin cos ........(4)
n=1 L L
Apply condition (iv)
∞ nπx
= k(lx − x2 )
P
y(x, 0) = cn sin 0<x<l
n=1 L
which is a half range Fourier sine series
To find cn

ZL
2 nπx
cn = f (x) sin dx
L L
0
 L 
Z2
 ZL 
2 2bx nπx 2b nπx 
= sin dx + (L − x) sin dx
L L 2 L 2 

0 L
2
 L   
 nπx nπx 2 nπx nπx L 
2 2b  (− cos L ) (− sin ) (− cos ) (− sin )
 

= x − (1) L 
+

(L − x) L − (−1) L 
LL  nπ n2 π 2
  nπ n2 π 2

t
 
 L L 2 0
L L 2 L

e
2

L2 L2
   nπ 
4b L L  nπ   nπ  L L  nπ 
=
L2

4b 2L

2 nπ
2
cos
 nπ 
2

+
n2 π 2
sin
2
+ 0 − −
2 nπ
cos
2

n2 π 2
sin

g . n 2

=
L2 n2 π 2
8b
sin


2

ri n
e
 
= sin

Sub in (4)
n2 π 2 2

i n e
ng

P8b  nπ  nπx nπat
y(x, t) = 2 π2
sin sin cos
n 2 L L

E
n=1
8b P∞ 1  nπ  nπx nπat

as y
y(x, t) = 2
π n=1 n2
sin
2
sin
2l
cos
2l

4. A tightly stretched string with fixed end points x = 0 and x = l initially in a position given by y(x, 0) =
y0 sin3 πx


. E
l . It is released from rest from this position, find the displacement y at any time and at any distance

w
from the end x = 0. Solution:

w w
The wave equation is
∂2y
∂t2
= a
2
2∂ y
∂x2

The Boundary conditions are:

(i)y(0, t) = 0 ∀t > 0

(ii)y(l, t) = 0 ∀t > 0
∂y(x, 0)
(iii) =0 0<x<l
∂t
 πx 
(iv)y(x, 0) = y0 sin3 0<x<l
l

The Suitable solution is


y(x, t) = (c1 cos px + c2 sin px)(c3 cos pat + c4 sin pat)........(1)
Apply condition (i)

Page 5

Downloaded From : www.EasyEngineering.net


Panimalar Engineering College Downloaded From : www.EasyEngineering.net Umar Ali

y(0, t) = c1 (c3 cos pat + c4 sin pat) = 0


⇒ Either c1 = 0 or c3 cos pat + c4 sin pat = 0
Since c3 cos pat + c4 sin pat 6= 0
⇒ c1 = 0
Sub in (1)
y(x, t) = c2 sin px(c3 cos pat + c4 sin pat)........(2)
Apply condition (ii)
y(l, t) = c2 sin pl(c3 cos pat + c4 sin pat) = 0
⇒ Either c2 = 0 or sin pl = 0 or c3 cos pat + c4 sin pat = 0
Since c3 cos pat + c4 sin pat 6= 0 and c2 6= 0[if c2 = 0 we get a trivial solution]
⇒ sin pl = 0 But sin nπ = 0
⇒ pl = nπ

⇒ p=
Sub in (2)
l

ne t
.
nπx nπat nπat
y(x, t) = c2 sin (c3 cos + c4 sin )........(3)

g
l l l
Diff par w.r.t ’t’
∂y(x, t)
= c2 sin
nπx

−c3 sin
nπat  nπa 
+ c4 cos
ri n
nπat  nπa 


Apply condition (iii)


∂t

∂y(x, 0)
l

nπx h  nπa i
l

n
l

e e l l

∂t
= c2 sin
⇒ Either c2 = 0 or sin
nπx nπa
l
nπx
l
c4
l

ng i
=0
= 0 or c4 = 0 or
nπa
l
=0
Since c2 6= 0 , sin
l
6= 0 and
l

y E
6= 0

as
⇒ c4 = 0
Sub in (3)

w . E
The most general form is
y(x, t) = c2 sin
nπx
l
c3 cos
nπat
l

w w
Apply condition (iv)
y(x, t) =

P
n=1


cn sin

nπx
nπx
l
cos
nπat
l
........(4)

 πx 
= y0 sin3
P
y(x, 0) = cn sin
n=1 l l  
πx 2πx nπx 3 πx 1 3πx
c1 sin + c2 sin + c3 sin + . . . = y0 sin − sin
l l l 4 l 4 l
Comparing like coefficients we get,
3y0 −y0
c1 = , c2 = 0, c3 = , c4 = c5 = c6 = · · · = 0
4 4
Sub in (1)
πx πat 2πx 2πat nπx 3πat
y(x, 0) = c1 sin cos + c2 sin cos + c3 sin cos + ...
l l l l l l
3y0 πx πat y0 nπx 3πat
y(x, 0) = sin cos − sin cos .
4 l l 4 l l

5. A tightly stretched string with fixed end points x = 0 and x = l initially in a position given by y(x, 0) =
   
3πx 2πx
k sin cos . It is released from rest from this position, Determine the displacement y(x, t).
l l

Page 6

Downloaded From : www.EasyEngineering.net


Panimalar Engineering College Downloaded From : www.EasyEngineering.net Umar Ali

WITH VELOCITY

6. A tightly stretched string with fixed end points x = 0 and x = l is initially at rest in its equilibrium position. If it
8λl3 ∞ 1
sin nπx nπat
P
is set vibrating giving each point a velocity λx(l − x),then show that y(x, t) = 4 l sin l .
π a n=1,3,5,.. n4
Solution:
The wave equation is
∂2y 2
2∂ y
= a
∂t2 ∂x2

The Boundary conditions are:

(i)y(0, t) = 0 ∀t > 0

(ii)y(l, t) = 0 ∀t > 0

(iii)y(x, 0) = 0 0<x<l
∂y(x, 0)
(iv)
∂t
= λx(l − x) 0<x<l

ne t
The Suitable solution is

n g .
Apply condition (i)
e ri
y(x, t) = (c1 cos px + c2 sin px)(c3 cos pat + c4 sin pat)........(1)

n
y(0, t) = c1 (c3 cos pat + c4 sin pat) = 0

i e
ng
⇒ Either c1 = 0 or c3 cos pat + c4 sin pat = 0
Since c3 cos pat + c4 sin pat 6= 0

y Ec1 = 0
Sub in (1)

. E as
y(x, t) = c2 sin px(c3 cos pat + c4 sin pat)........(2)

w w
Apply condition (ii)

w y(l, t) = c2 sin pl(c3 cos pat + c4 sin pat) = 0


⇒ Either c2 = 0 or sin pl = 0 or c3 cos pat + c4 sin pat = 0
Since c3 cos pat + c4 sin pat 6= 0 and c2 6= 0[if c2 = 0 we get a trivial solution]
⇒ sin pl = 0 But sin nπ = 0
⇒ pl = nπ

⇒ p=
l
Sub in (2)
y(x, t) = c2 sin nπx nπat
l (c3 cos l + c4 sin nπat
l )........(3)

Apply condition (iii)


y(x, 0) = c2 sin nπx
l c3 = 0

⇒ Either c2 = 0 or sin nπx


l or c3 = 0

Since c2 = 0 6= 0 and sin nπx


l 6= 0

Page 7

Downloaded From : www.EasyEngineering.net


Panimalar Engineering College Downloaded From : www.EasyEngineering.net Umar Ali

⇒ c3 = 0
Sub in (3)
y(x, t) = c2 sin nπx nπat
l c4 sin l

The most general form is



cn sin nπx nπat
P
y(x, t) = l sin l ........(4)
n=1

Diff par w.r.t ’t’


∂y(x, t) ∞
cn sin nπx nπat nπa
P 
= l cos l l
∂t n=1
Apply condition (iv)
∂y(x, 0) ∞
cn sin nπx nπa
P 
= l l = λx(l − x)
∂t n=1
∞ ∞
cn sin nπx nπa
bn sin nπx nπa
P  P
⇒ l l = l = λx(l − x) where bn = cn l
n=1 n=1
which is a half range Fourier sine series
To find bn

ne t
bn =
2
l
Zl
f (x) sin
nπx
l
dx

n g .
i
0

=

l
Zl
(lx − x2 ) sin
nπx
l
dx

e e r
n
0

=

l
"
(lx − x ) 2 − cos l

l
nπx


ng i
− (l − 2x)
− sin nπx
nπ 2
l

l

+ (−2)
(cos nπx
nπ 3
l

l )
#l

E l3 3
  
2λ l
−2 3 3 (−1)n − −2 3 3

y
=
l n π n π

.
cn
E nπa
=

=

l
as
4λl2
2l3
× 3 3 [−(−1)n + 1]
n π
[1 − (−1)n ]

w w l
cn =
n3 π 3
l 4λl2
nπa n3 π 3
[1 − (−1)n ]

w =

=
n


4λl3
4 π4 a
[1 − (−1)n ]
3
 8λl , when ’n’ is odd;
n4 π 4 a
 0,

when ’n’ is even.

Substituting in (4)
8λl3

P nπx nπat
y(x, t) = 4 π4 a
sin cos
n=odd n l l
8λl3 P∞ 1 nπx nπat
y(x, t) = 4 4
sin cos
π a n=1,3,5,... n l l

7. A tightly stretched string of length ’l’ is initially at rest in its equilibrium position and each of its point given
a velocity v0 sin3 πx

l .Determine the displacement y(x, t). Solution:

The wave equation is


∂2y 2
2∂ y
= a
∂t2 ∂x2

Page 8

Downloaded From : www.EasyEngineering.net


Panimalar Engineering College Downloaded From : www.EasyEngineering.net Umar Ali

The Boundary conditions are:

(i)y(0, t) = 0 ∀t > 0

(ii)y(l, t) = 0 ∀t > 0

(iii)y(x, 0) = 0 0<x<l
∂y(x, 0)
(iv) = λx(l − x) 0<x<l
∂t

The Suitable solution is


y(x, t) = (c1 cos px + c2 sin px)(c3 cos pat + c4 sin pat)........(1)

Apply condition (i)


y(0, t) = c1 (c3 cos pat + c4 sin pat) = 0
⇒ Either c1 = 0 or c3 cos pat + c4 sin pat = 0
Since c3 cos pat + c4 sin pat 6= 0
⇒ c1 = 0

ne t
Sub in (1)

n g .
e ri
y(x, t) = c2 sin px(c3 cos pat + c4 sin pat)........(2)

Apply condition (ii)

i n e
ng
y(l, t) = c2 sin pl(c3 cos pat + c4 sin pat) = 0
⇒ Either c2 = 0 or sin pl = 0 or c3 cos pat + c4 sin pat = 0

y E
Since c3 cos pat + c4 sin pat 6= 0 and c2 6= 0[if c2 = 0 we get a trivial solution]

. E

⇒ as sin pl = 0
pl = nπ

But sin nπ = 0

w
Sub in (2) w ⇒ p=
l

wApply condition (iii)


y(x, t) = c2 sin nπx nπat
l (c3 cos l + c4 sin nπat
l )........(3)

y(x, 0) = c2 sin nπx


l c3 = 0

⇒ Either c2 = 0 or sin nπx


l or c3 = 0

Since c2 = 0 6= 0 and sin nπx


l 6= 0

⇒ c3 = 0
Sub in (3)
y(x, t) = c2 sin nπx nπat
l c4 sin l

The most general form is



cn sin nπx nπat
P
y(x, t) = l sin l ........(4)
n=1

Diff par w.r.t ’t’

Page 9

Downloaded From : www.EasyEngineering.net


Panimalar Engineering College Downloaded From : www.EasyEngineering.net Umar Ali

∂y(x, t) ∞
cn sin nπx nπat nπa
P 
= l cos l l
∂t n=1
Apply condition (iv)
∂y(x, 0) ∞
cn sin nπx nπa
= v0 sin3 πx
P  
= l l l
∂t n=1  
πx πx 2πx 3πx 3 3
c1 sin l + c2 sin 2πx
l + c 3 sin nπx
l + . . . = v 0 sin πx
l − sin 3πx
l
l l l 4 4
Comparing like coefficients we get,
πx 3y0 3πx −y0
c1 = , c2 = 0, c3 = , c4 = c5 = c6 = · · · = 0
l 4 l 4 
l 3y0 l −y0
c1 = , c2 = 0, c3 = , c4 = c5 = c6 = · · · = 0
πx 4 3πx 4
Sub in (1)
y(x, 0) = c1 sin πx 2πx nπx
l + c2 sin l + c3 sin l + . . .
3v0 l v0 l
y(x, 0) = sin πx nπx
l − 12πa sin l .
4πa

8. A string of length ’l’ isinitially at rest in its equilibrium position and motion is started by giving each of its
 cx, for 0 < x ≤ 2l ;
points a velocity V =
 c(l − x), for l < x ≤ l.
2
Find the displacement y(x, t).

ne t
1D HEAT FLOW EQUATION

n g .
e ri
1. A metal bar 30 cm long has its ends A and B kept at 20◦ C and 80◦ C respectively, until steady state conditions

n e
prevail. The temperature at each end is then suddenly reduced to 0◦ C and kept so. Find the resulting

i
ng
temperature distribution function u(x, t) taking x = 0 at A.

Solution:

y E
as
The 1D heat equation is
∂u ∂2u
= α2 x

w . E ∂t ∂

w w
When steady state condition prevails,
we get,
∂u
∂t
=0

θ2 − θ1
u(x) = x + θ1
l
80 − 20
= x + 20
30
= 2x + 20 0 ≤ x ≤ 30

The Boundary conditions are:

(i)u(0, t) = 0 ∀t > 0

(ii)u(30, t) = 0 ∀t > 0

(iii)u(x, 0) = 2x + 20 0 ≤ x ≤ 30

Page 10

Downloaded From : www.EasyEngineering.net


Panimalar Engineering College Downloaded From : www.EasyEngineering.net Umar Ali

The Suitable solution is


2 2
u(x, t) = (c1 cos px + c2 sin px)e−α p t
........(1)

Apply condition (i)


2 2
u(0, t) = c1 e−α p t
=0
−α2 p2 t
⇒ Either c1 = 0 or e =0
2 2
Since e−α p t
6= 0
⇒ c1 = 0
Sub in (1)
2 2
u(x, t) = c2 sin pxe−α p t
........(2)

Apply condition (ii)


2 2
u(30, t) = c2 sin 30pe−α p t
=0
2 2
⇒ Either c2 = 0 or sin 30p = 0 or e−α p t
=0

t
2 2
Since e−α p t
6= 0 and c2 6= 0[if c2 = 0 we get a trivial solution]
⇒ sin 30p = 0

n
But sin nπ = 0

. e


30p = nπ
p=

i n g
Sub in (2)
30

u(x, t) = c2 sin nπx −α2 ( nπ


30 )
2
t

e e r
30 e

i n
ng
The most general form is
∞ 2 2 2t
nπx −α 900
n π
P
u(x, t) = cn sin 30 e ........(3)

Apply condition (iii)


y E n=1

as

cn sin nπx
P
u(x, 0) = 30 = 2x + 20

E
n=1

To find cn
w .
which is a half range Fourier sine series

w w cn =
2
l
Zl

0
f (x) sin
nπx
l
dx

Z30
2 nπx
= (2x + 20) sin dx
30 30
0
"  #30
− cos nπx − sin nπx

1 30 30
= (2x + 20) nπ −2
15 nπ 2

30 30 0
  
1 30 30
= −80 (−1)n − −20 ·1
15 nπ nπ
1 30
= · 20 [−4(−1)n + 1]
15 nπ
40
= [1 − 4(−1)n ]

Substituting in (3)

Page 11

Downloaded From : www.EasyEngineering.net


Panimalar Engineering College Downloaded From : www.EasyEngineering.net Umar Ali

∞ 40 −α2 n2 π 2 t
[1 − 4(−1)n ] sin nπx
P
u(x, t) = 30 e 900
n=1 nπ
40 P∞ 1 −α2 n2 π 2 t
u(x, t) = [1 − 4(−1)n ] sin nπx
30 e 900
π n=1 n

2. A bar,10 cm long, with insulated sides, has its ends A and B kept at 20◦ C and 40◦ C respectively until steady
state condition prevail. The temperature at A is suddenly raised to 50◦ C and at the same instant at B is
lowered to 10◦ C. Find the subsequent temperature at any point of the bar at any time.

Solution:
The 1D heat equation is
∂u ∂2u
= α2 x
∂t ∂

Here there are two steady states,


The solution may be, u(x, t) = us (x) + uT (x, t).......(I)

ne t
n g .
u(x, 0) =
θ2 − θ1
l
x + θ1

e ri
us (x) =
θ2 − θ1
l
x + θ1

=
40 − 20
10
x + 20

in e =
10 − 50
10
x + 50

= 2x + 20

n g = −4x + 50

(I) ⇒ u(x, t) = −4x + 50 + uT (x, t)

s y E
The Boundary conditions are:

. E a
w w (i)u(0, t) = 50

(ii)u(10, t) = 10
∀t > 0

∀t > 0

wThe Suitable solution is


(iii)u(x, 0) = 2x + 20 0 ≤ x ≤ 10

2 2
u(x, t) = −4x + 50 + (c1 cos px + c2 sin px)e−α p t
........(1)

Apply condition (i)


2 2
u(0, t) = 50 + c1 e−α p t
= 50
2 2
⇒ c1 e−α p t
=0
2 2
⇒ Either c1 = 0 or e−α p t
=0
−α2 p2 t
Since e 6= 0
⇒ c1 = 0
Sub in (1)
2 2
u(x, t) = −4x + 50 + c2 sin pxe−α p t
........(2)

Page 12

Downloaded From : www.EasyEngineering.net


Panimalar Engineering College Downloaded From : www.EasyEngineering.net Umar Ali

Apply condition (ii)


2 2
u(10, t) = 10 + c2 sin 10pe−α p t
= 10
2 2
⇒ c2 sin 10pe−α p t
=0
2 2
⇒ Either c2 = 0 or sin 10p = 0 or e−α p t
=0
−α2 p2 t
Since e 6= 0 and c2 6= 0[if c2 = 0 we get a trivial solution]
⇒ sin 10p = 0 But sin nπ = 0
⇒ 10p = nπ

⇒ p=
10
Sub in (2)
2
−α2 ( nπ
10 ) t
u(x, t) = −4x + 50 + c2 sin nπx
10 e

The most general form is


∞ −α2 n2 π 2 t
cn sin nπx
P
u(x, t) = −4x + 50 + 10 e
100 ........(3)
n=1

Apply condition (iii)

ne t
u(x, 0) = −4x + 50 +

P
n=1
cn sin nπx
10 = 2x + 20

n g .
i

cn sin nπx
P
10 = 6x − 30
n=1
which is a half range Fourier sine series

e e r
To find cn

i n
ng
Zl
2 nπx
cn = f (x) sin dx

y E l
0
l

as
Z10
2 nπx
= (6x − 30) sin dx

E
10 10

w . =
1
5
"
0

(6x − 30)
− cos nπx

10

−6
− sin nπx

nπ 2
10
 #10

w

10
   10 0

w
1 10 10
= −30 (−1)n − 30 ·1
5 nπ nπ
1 30
= · 30 [−(−1)n − 1]
15 nπ
−60
= [1 + (−1)n ]


 0, when ’n’ is odd;
=
 −120 , when ’n’ is even.

Substituting in (3)
−120 ∞ −α2 n2 π 2 t
sin nπx
P
u(x, t) = −4x + 50 + 10 e
100
n=2,4,6,··· nπ
120 ∞ 1 −α2 n2 π 2 t
sin nπx
P
u(x, t) = −4x + 50 − 30 e 100
π n=2,4,6,··· n

A metal bar 20 cm long has its ends A and B kept at 30◦ C and 90◦ C respectively, until steady state condi-

Page 13

Downloaded From : www.EasyEngineering.net


Panimalar Engineering College Downloaded From : www.EasyEngineering.net Umar Ali

tions prevail. The temperature at each end is then suddenly reduced to 0◦ C and kept so. Find the resulting
temperature distribution function u(x, t) at a distance x from A at time t.
∞ [1 − 3(−1)n ]
60 P −α2 n2 π 2 t
Ans:u(x, t) = sin nπx
20 e
400
π n=1 l

A bar,l cm long, with insulated sides, has its ends A and B kept at 30◦ C and 80◦ C respectively until steady
state condition prevail. The temperature of the end B is suddenly reduced to 60◦ C and that of A increased to
40◦ C . Find the temperature distribution of the rod after time t.
20 20 ∞ [1 + 2(−1)n ] −α2 n2 π 2 t
sin nπx
P
Ans:u(x, t) = x + 40 − l e l2
l π n=2,4,6,··· n

2D HEAT FLOW EQUATION

3. A square plate is bounded by the lines x = 0, y = 0, x = 20 and y = 20. Its faces are insulated. The temperatue

e t
along the upper horizontal edge is given by u(x, 20) = x(20 − x), 0 < x < 20 while other three edges are kept

n
at 0◦ C. Find the steady state temperature in the plate.

n g .
Solution:
The 2D heat flow equation is

e ri
2
∂ u ∂ u
∂x2
+ 2 =0
∂y
2

i n e
ng
The Boundary conditions are:

(i)u(0, y) = 0

y E0 ≤ y ≤ 20

as
(ii)u(20, y) = 0 0 ≤ y ≤ 20

. E
(iii)u(x, 0) = 0

w
(iv)u(x, 20) = x(20 − x)
0 ≤ x ≤ 20

0 ≤ x ≤ 20

w w
The Suitable solution is
u(x, y) = (c1 cos px + c2 sin px)(c3 epy + c4 e−py )........(1)
Apply condition (i)
u(0, y) = c1 (c3 epy + c4 epy ) = 0
⇒ Either c1 = 0 or c3 epy + c4 e−py
Since c3 epy + c4 epy 6= 0
⇒ c1 = 0
Sub in (1)
u(x, y) = c2 sin px(c3 epy + c4 e−py )........(2)
Apply condition (ii)
u(20, y) = c2 sin 20p(c3 epy + c4 e−py ) = 0
⇒ Either c2 = 0 or sin 20p = 0 or c3 epy + c4 e−py = 0

Page 14

Downloaded From : www.EasyEngineering.net


Panimalar Engineering College Downloaded From : www.EasyEngineering.net Umar Ali

Since c3 epy + c4 e−py 6= 0 and c2 6= 0[if c2 = 0 we get a trivial solution]


⇒ sin 20p = 0 But sin nπ = 0
⇒ 20p = nπ

⇒ p=
20
Sub in (2)
nπx  nπy −nπy

u(x, y) = c2 sin c3 e 20 + c4 e 20 ........(3)
20
Apply condition (iii)
nπx
u(x, 0) = c2 sin (c3 + c4 ) = 0
20
nπx
⇒ Either c2 = 0 or sin or c3 + c4 = 0
20
nπx
6 0 and sin
Since c2 = 6= 0
20
⇒ c3 + c4 = 0
⇒ c4 = −c3
Sub in (3)
u(x, y) = c2 sin
nπx  nπy
20
−nπy
c3 e 20 − c3 e 20


ne t
.
nπx nπy −nπy
u(x, y) = c2 sin c3 (e 20 − e 20 )

g
20
nπx
c3 (2 sinh nπy
The most general form is
u(x, y) = c2 sin
20 20 )

ri n
u(x, y) =

P
n=1
cn sin
nπx
20
sinh nπy

n e e
20 ........(4)

Apply condition (iv)


u(x, 20) =

P
n=1
cn sin

ng
nπx
20isinh nπ = x(20 − x) 0<x<l

E

P nπx
= x(20 − x)
bn sin where bn = cn sinh nπ
n=1

as y
20
which is a half range Fourier sine series

E
To find bn

w . 2
Zl
nπx

w
bn = f (x) sin dx
l l
0

w =
2
20
Z20

0
(20x − x2 ) sin
nπx
20
dx

nπx  nπx  nπx 20


   
1  − cos − sin (cos )
= (20x − x )
2 20 − (20 − 2x) 20 + (−2) 20 
10 nπ  nπ 2  nπ 3 
20 20 20 0
3 3
  
1 20 20
= −2 3 3 (−1)n − −2 3 3
10 n π n π
3
1 2 · 20
= × 3 3 [−(−1)n + 1]
10 n π
2
4 · 20
bn = [1 − (−1)n ]
n3 π 3
4 · 202
cn sinh nπ = [1 − (−1)n ]
n3 π 3
1 4 · 202
cn = [1 − (−1)n ]
sinh nπ n3 π 3

Page 15

Downloaded From : www.EasyEngineering.net


Panimalar Engineering College Downloaded From : www.EasyEngineering.net Umar Ali


2
 8 · 20
 1
, when ’n’ is odd;
n 3 π 3 sinh nπ
=
 0,

when ’n’ is even.

Substituting in (4)
∞ 3200 nπx
sinh nπy
P
u(x, y) = sin 20
n=odd n3 π 3
sinh nπ 20
3200 P∞ 1 nπx
u(x, y) = 3 sin sinh nπy
20
π n=odd n3 sinh nπ 20

4. A rectangular plate with insulated surfaces is 20 cm wide and so long compared to its width that it may be
considered infinite in 
length without introducing an appreciable error. If the temperature of the short edge
 10y, 0 ≤ y ≤ 10;
x = 0 is given by u =
 10(20 − y), 10 ≤ y ≤ 20.
and the two long edges as well as the other short edge are kept at 0◦ C, find the steady state temperature

t
distribution in the plate. Solution:
The 2D heat flow equation is
∂2u ∂2u
+ 2 =0
. ne
∂x2 ∂y

i n g
r
The Boundary conditions are:

(i)u(x, 0) = 0

n e e
(ii)u(x, 20) = 0

ng i
E
(iii)u(∞, y) = 0 0 ≤ y ≤ 20

y

as
 10y, 0 ≤ y ≤ 10;
(iv)u(0, y) =
 10(20 − y), 10 ≤ y ≤ 20.

w . E
The Suitable solution is

w w
Apply condition (i)
u(x, y) = (c1 epx + c2 e−px )(c3 cos py + c4 sin py)........(1)

u(x, 0) == (c1 epx + c2 e−px )c3 = 0


⇒ Either c1 epx + c2 e−px or c1 = 0
Since c1 epx + c2 e−px or 6= 0
⇒ c3 = 0
Sub in (1)
u(x, y) = (c1 epx + c2 e−px )c4 sin py........(2)
Apply condition (ii)
u(x, 20) = (c1 epx + c2 e−px )c4 sin 20p = 0
⇒ Either c1 epx + c2 e−px = 0 or c4 = 0 or sin 20p = 0
Since c1 epx + c2 e−px 6= 0 and c4 6= 0[if c4 = 0 we get a trivial solution]
⇒ sin 20p = 0 But sin nπ = 0

Page 16

Downloaded From : www.EasyEngineering.net


Panimalar Engineering College Downloaded From : www.EasyEngineering.net Umar Ali

⇒ 20p = nπ

⇒ p=
20
Sub in (2)
nπx −nπx nπy
u(x, y) = (c1 e 20 + c2 e 20 )c4 sin ........(3)
20
Apply condition (iii)
nπy
=0
u(∞, y) = c1 c4 sin
20
nπy
⇒ Either c1 = 0 or c4 = 0 or sin
20
nπy
Since c4 6= 0 and sin 6= 0
20
⇒ c2 = 0 for if c2 6= 0,then eλx → ∞ ⇒ u → ∞
which is a contradiction for u = 0.
Sub in (3)
nπx nπy
u(x, y) = c2 e 20 c4 sin
20

The most general form is

ne t
.

P nπy nπx
u(x, y) = cn sin e 20 ........(4)
Apply condition (iv)
n=1 20

i n g
r

∞ nπy  10y, 0 ≤ y ≤ 10;

e
P
u(0, y) = cn sin =
20

e
n=1  10(20 − y), 10 ≤ y ≤ 20.
which is a half range Fourier sine series

i n
ng
To find cn

2
Zl
nπx

y E
as
cn = f (x) sin dy
l l
0

E
 10 
Z20

.
Z
2  nπy nπy 
= 10y sin dy + 10(20 − y) sin dy
20 20 20

w 1
0


w 
− cos
nπy
10

20  − (1) 

− sin
nπy 10 
20 





− cos
nπy 

20  − (−1) 
− sin
 
nπy 20
20 
 

w = 10  y nπ + (20 − y) nπ
  
10  n2 π 2 n2 π 2
    
 
 
 

20 20 2 0
20 20 2 10
202 202
  
20 nπ nπ 20 nπ nπ
= −10 cos + 2 2 sin − −10 cos − 2 2 sin
nπ 2 n π 2 nπ 2 n π 2
202 nπ
= 2 2 2 sin
n π 2
800 nπ
cn = sin
n2 π 2 2
 800 nπ

2 2
sin , when ’n’ is odd;
= n π 2
 0, when ’n’ is even.

Substituting in (4)

P 800 nπ nπx nπy
u(x, y) = 2 π2
sin e 20 sin
n=odd n 2 20
3200 P ∞ 1 nπx nπy
u(x, y) = 2 e 20 sin
π n=odd n2 20

Page 17

Downloaded From : www.EasyEngineering.net


Panimalar Engineering College Downloaded From : www.EasyEngineering.net Umar Ali

Find the steady state temperature distribution in a rectangular plate of sides a and b insulated at the lateral
surfaces and satisfying the boundary conditions
u(0, y) = 0, u(a, y) = 0, 0 ≤ y ≤ b
u(x, b) = 0, u(x, 0) = x(a − x), 0 ≤ x ≤ a
A rectangular plate with insulated surface is 10cm wide and so long compared to its width that it may be
considered infinite in length without introducing appreciable error. The temperature at short edge y = 0 is
given by 
 20x, 0 ≤ x ≤ 5;
u=
 20(10 − x), 5 ≤ x ≤ 10.

and all the other three edges are kept at 0 ◦ C. Find the steady state temperature at any point in the plate.
nπy
800 P ∞ 1 nπx
Ans:u(x, y) = 2 2
e 10 sin
π n=odd n 10
An infinite long rectangular plate with insulated surface in 10cm wide. The two long edges and one short edge

t
are kept at zero temperature. While the other short edge x = 0 is kept at temperature given by


 20y, 0 ≤ y ≤ 5;

. ne
g
u=
 20(10 − y), 5 ≤ y ≤ 10.

ri n
Find the steady state temperature in the plate.
800 P
Ans:u(x, y) = 2
∞ 1
e
nπx
10 sin
nπy

n e e
π n=odd n2 10

ng i
Two Marks

y E
Note:

.
In 2nd order PDE,
E as
w w
1.If B 2 − 4AC = 0 then it is said to Prabolic
2.If B 2 − 4AC < 0 then it is said to Elliptic

w 3.If B 2 − 4AC > 0 then it is said to Hyperbolic.

1. Find the nature of the PDE


x2 uxx + 2xyyxy + (1 + y 2 )uyy − 2ux = 0.
Solution:
Given x2 uxx + 2xyyxy + (1 + y 2 )uyy − 2ux = 0.
Here A = x2 , B = 2xy, C = 1 + y 2
∴ B 2 − 4AC = 4x2 y 2 − 4x2 (1 + y 2 )
= −4x2 If x = 0 then the equation is Prabolic
If x < 0 or x > 0 then the equation is Elliptic.

Classify the PDE,


(a) y 2 uxx − 2xyyxy + x2 uyy + 2ux − 3uy = 0.[Ans: Parabolic]

Page 18

Downloaded From : www.EasyEngineering.net


Panimalar Engineering College Downloaded From : www.EasyEngineering.net Umar Ali

∂2u ∂2u ∂2u ∂u


(b) 3 2
+ 43 +6 2 −2 − u = 0.[Ans: Elliptic]
∂x ∂x∂x ∂y ∂y

2. Write down the PDE governing the transverse vibrations of an elastic string.(or)
∂2y ∂2y
In the wave equation 2 = c2 2 ,what is c2 stands for?
∂t ∂x
Solution:
The PDE governing the transverse vibrations of an elastic string is

∂2y ∂2y
2
= c2 2
∂t ∂x

Tension(T)
where c2 = .
Mass(M) per unit length of the string
∂u ∂2u
3. State the governing equation for 1D heat equation.(or) In the diffusion equation = α2 2 ,what is α2 stands
∂t ∂x
for?

t
Solution:
The governing equation is
∂u ∂2u
= α2 2
. ne
∂t ∂x

i n g
r
k
where α2 =
ρc
k-thermal conductivity

n e e
ρ-density of material

ng i
c-specific heat of the material

y E
4. Write down the 2D heat flow equation in steady state.

as
Solution:

w E
The 2D heat flow equation in steady state is the Laplace equation

. ∂2u ∂2u
+ 2 =0

w w
5. What are the various solutions of 1D wave equation:
∂x2 ∂y

Solution:
The possible solutions are

(i)y(x, t) = (A cos px + B sin px)(C cos pat + D sin pat)

(ii)y(x, t) = (Aepx + Be−px )(Cepat + De−pat )

(iii)y(x, t) = (Ax + B)(Ct + D)

∂u ∂2u
6. State the three possible solutions of the heat equation = α2 2
∂t ∂x
Solution:

Page 19

Downloaded From : www.EasyEngineering.net


Panimalar Engineering College Downloaded From : www.EasyEngineering.net Umar Ali

The possible solutions are

2
α2 t
(i)u(x, t) = (A cos px + B sin px)e−p
2
α2 t
(ii)u(x, t) = (Aepx + Be−px )ep

(iii)u(x, t) = Ax + B

7. Write down the possible solutions of the 2D heat equation in steady state.
Solution:
The possible solutions of the 2D heat equation in steady state will be:

(i)u(x, y) = (A cos px + B sin px)(Cepy + De−py )

(ii)u(x, y) = (Aepx + Be−px )(C cos py + D sin py)

(iii)u(x, y) = (Ax + B)(Cy + D)

ne t
8. What is the basic difference between the solution of 1D wave and 1D heat equations?

n g .
Solution:
Solution of 1D wave equation
∂2y
∂t2
∂2y
∂x
e ri
= c2 2 is y(x, t) = (A cos px + B sin px)(C cos pat + D sin pat),
which is periodic w.r.to t.

i n
∂u ∂2u e
ng
2 2
But the Solution of 1D heat equation = α2 2 is u(x, t) = (A cos px + B sin px)e−p α t , which
∂t ∂x
is non-periodic w.r.to t.

y E
as
9. in steady state conditions derive the solution of 1D heat flow equation.
Solution:

w . E
The 1D heat equation is
∂u
∂t
In steady state,
∂u
∂2u
= α2 2 .......(1)
∂x
=0

w w then (1)⇒
2
∂ u
∂x2
=0


∂t

∂u
∂x
=a
⇒ u = ax + b, where a and b are arbitrary constants.

10. A rod 30 cm long has its ends A and B kept at 20 ◦ C and 80 ◦ C respectively until steady state condition
prevail. Find the steady state temperature in the rod.
Solution:
The steady state temperature is

θ2 − θ1
u(x) = x + θ1 , Here θ1 = 20,θ2 = 80,l = 30
l
80 − 20
= x + 20
30
⇒ = 2x + 20, 0 ≤ x ≤ 30

Page 20

Downloaded From : www.EasyEngineering.net


Panimalar Engineering College Downloaded From : www.EasyEngineering.net Umar Ali

The ends A and B of a rod l cm long have temperature 40 ◦ C and 90 ◦ C until steady state prevails. Find the
temperature in the rod that state.
50
Ans:u(x) = x + 40, 0≤x≤l
l

A rod 40cm long with insulated sides has its ends A and B kept at 20 ◦ C and 60 ◦ C respectively. Find the
steady state temperature at a location 15cm for A.
x
Hint: u(x) = + 20, 0 ≤ x ≤ 40
2
Sub x = 15, Ans:27.5 ◦ C

11. What conditions are assumed in deriving the 1D wave equation?


Solution:
Assumptions:
(i) The string is homogenous.

ne t
(ii) The string is perfectly elastic and so it does not offer any resistance to bending.
(iii) The tension T caused by stretching the string is so large so that the action of the gravitational
force on the string can be neglected.

n g .
the slope
∂y

e i
(iv) The string performs small transverse motions in a vertical plane so that the direction y and

r
are small in absolute value and hence, their higher powers may be neglected.
∂x

i n
12. What are the laws assumed to derive the 1D heat equation? e
ng
Solution:
Assumptions:

y E
as
(i) Heat flows from higher to lower temperature
(ii)The amount of heat required to produce a given temperature change in a body is proportional

. E
to the mass of the body and to the temperature change.

w (iii) Fourier Law of Heat Conduction:

w w
The rate at which heat flows through an area is proportional to the area and to the temperature gradient
normal to the area.

Note:
In 2D they will ask the boundary conditions from the given question.

Page 21

Downloaded From : www.EasyEngineering.net


Downloaded From : www.EasyEngineering.net Umar Ali

Z Transform

Definition:1
Lef {f (n)} be a sequence defined for n = 0, ±1, ±2, · · · .Then the Z-transform is defined as


X
Z[f (n)] = f (n)z −n , [z → a complex number]
n=−∞
= F (z)

This is called two sided or bilateral Z-transform.


Definition:2


X
Z[f (n)] = f (n)z −n , [z → a complex number]
n=0
= F (z)

ne t
This is called one sided or unilateral Z-transform.

n g .
i
1. Find

(a) Z[an ]

e e r
Solution:

i n
ng
Given f (n) = an

E

X
f (n)z −n

y
Z[f (n)] =

as
n=0
X∞
Z[an ] an z −n

E
=

w . =
n=0
X∞
(az −1 )n

w w =

=
n=0
1 + az −1 + (az −1 )2 + (az −1 )3 + · · ·

(1 − az −1 )−1 if |az −1 | < 1


1 a
= a if | | < 1
1− z
z
z
= if |z| > |a|
z−a

Note:

z
If a = 1, Z[1] = if |z| > 1
z−1
z
If a = −1, Z[−1] = if |z| > 1
z+1

i. Find Z[1]

ii. Find Z[−1]

Page 1

Downloaded From : www.EasyEngineering.net


Downloaded From : www.EasyEngineering.net Umar Ali

 n
−1
iii. Find
3
(b) Z[n]
Solution:
Given f (n) = n


X
Z[f (n)] = f (n)z −n
n=0
X∞
Z[n] = nz −n
n=0
= 0 + 1 · z −1 + 2 · z −2 + 3 · z −3 + · · ·
1 1 1
= + 2 + 3 3 + ···
z  z2 z 
1 1 1
= 1 + 2 + 3 + ···
z z z
 −2
1 1
=
z

1−

1 z−1
z
−2
if |z| > 1

ne t
=
z
1

z
z2


n g .
=

=
z (z − 1)2
z

e ri if |z| > 1
(z − 1)2

i n e
ng
 
1
(c) Z
n
Solution:

y E 1

as
Given f (n) =
n

w . E Z[f (n)] =

X

n=0

f (n)z −n

w
 
1 X 1 −n
Z = z
n n

w =
n=1

= z −1 +
z −2
2
+

− log 1 − z −1
z −3


3
+ ···

if |z| > 1
 
1
= − log 1 −
z
 
z−1
= − log
z
z
= loge if |z| > 1
z−1

 
1
(d) Z
n!
Solution:

Page 2

Downloaded From : www.EasyEngineering.net


Downloaded From : www.EasyEngineering.net Umar Ali

1
Given f (n) =
n!


X
Z[f (n)] = f (n)z −n
n=0
  ∞
1 X 1 −n
Z = z
n n=0
n!
z −1 z −2 z −3
= 1+ + + + ···
1! 2! 3!
−1
= ez
1
= ez

 
1
(e) Z
(n + 1)!
Solution:
1
Given f (n) =
(n + 1)!

ne t
.
X
Z[f (n)] = f (n)z −n
 
1
n=0

1

i n g
r
X
Z = z −n
n (n + 1)!

=
1
n=0

n
+
z
+
−1
z
+
z
e
−2
e −3
+ ···

ng
= z i
1!
z −1
1!
2!
+
z −2
2!
3!
+
z −3
3!
4!
+ ···


E
h −1 i
z
= z e −1

as y h 1
= z ez − 1
i

(f) Z
an


n!


w . E
w
Solution:
an

w Given f (n) =
n!

Z[f (n)] =

X
f (n)z −n
n=0

an an −n
  X
Z = z
n n=0
n!

X (az −1 )n
=
n=0
n!
az −1 (az −1 )2 (az −1 )3
= 1+ + + + ···
1! 2! 3!
−1
= eaz
a
= ez

i. Find Z[nan ]

Page 3

Downloaded From : www.EasyEngineering.net


Downloaded From : www.EasyEngineering.net Umar Ali

Property:Differentiation in Z-Domain

d d
2. Prove that Z[nf (n)] = −z Z[f (n)] = −z F (z).
dz dz
Solution:


X
W.K.T F (z) = Z[f (n)] = f (n)z −n
n=0

d d X
F (z) = f (n)z −n
dz dz n=0

X
= f (n)(−n)z −n−1
n=0

X 1
= − nf (n)z −n
n=0
z

d X
−z nf (n)z −n

t
F (z) =
dz

e
n=0

n
d

.
Z[nf (n)] = −z F (z)
dz

(a) Find Z[n2 ]


i n g
Solution:

e e r
Z[n2 ]
i n
Z[n × n]

ng
=
d
= −z Z[n]

s y E = −z
dz
d z
dz (z − 1)2

.E a = −z



(z − 1)2 (1) − z2(z − 1)(1)
(z − 1)4
(z − 1)(z − 1 − 2z



w w =

=
−z

−z

(z − 1)4
−z − 1


w =
2
z +z
(z − 1)3

(z − 1)3

z 3 + 4z 2 + z
Show that Z[n3 ] =
(z − 1)4
(b) Find Z[nan ]
Solution:

d
Z[nan ] = −z Z[an ]
dz
d z
= −z
dz z − a
(z − a)(1) − z(1)
= −z
(z − a)2
az
=
(z − a)2

Page 4

Downloaded From : www.EasyEngineering.net


Downloaded From : www.EasyEngineering.net Umar Ali

(c) Find Z[(n + 1)(n + 2)]


Solution:

Z[(n + 1)(n + 2)] = Z[n2 + 3n + 2]

= Z[n2 ] + 3Z[n] + 2Z[1]


z2 + z z z
= +3 +2
(z − 1)3 (z − 1)2 z−1
z 2 + z + 3z(z − 1) + 2z(z − 1)2
=
(z − 1)3
z + z + 3z − 3z + 2z(z 2 − 2z + 1)
2 2
=
(z − 1)3
z + z + 3z − 3z + 2z 3 − 4z 2 + 2z
2 2
=
(z − 1)3
3
2z
=
(z − 1)3

(d) Find Z[n(n − 1)(n − 2)]

ne t
Solution:

n g .
Z[n(n − 1)(n − 2)] = Z[n3 − 3n2 + 2n]

e ri
= Z[n3 ] − 3Z[n2 ] + 2Z[n]

=
i n
z 3 + 4z 2 + z
− 3
z2 + z
e +2
z

ng
(z − 1) 4 (z − 1) 3 (z − 1)2
z + 4z + z − 3(z + z)(z − 1) + 2z(z − 1)2
3 2 2
=

y E (z − 1)4
z + 4z + z + (−3z 2 − 3z)(z − 1) + 2z(z 2 − 2z + 1)
3 2

as
=
(z − 1)4
z + 4z + z − 3z − 3z 2 + 3z 2 + 3z + 2z 3 − 4z 2 + 2z
3 2 3

w . E =

=
6z
(z − 1)4
(z − 1)4

w w
3. Find Z[an cos nθ] and Z[an sin nθ]
Solution:

W.K.T eiθ = cos θ + i sin θ

and einθ = cos nθ + i sin nθ


z
Also Z[an ] =
z−a
z
⇒ Z[(aeiθ )n ] =
z − aeiθ
z
⇒ Z[an cos nθ + ian sin nθ] =
z − a(cos θ + i sin θ)
z
⇒ Z[an cos nθ] + iZ[an sin nθ] =
(z − a cos θ) − ia sin θ
z (z − a cos θ) + ia sin θ
=
(z − a cos θ) − ia sin θ (z − a cos θ) + ia sin θ

Page 5

Downloaded From : www.EasyEngineering.net


Downloaded From : www.EasyEngineering.net Umar Ali

z[(z − a cos θ) + ia sin θ]


=
(z − a cos θ)2 + a2 sin2 θ
z(z − a cos θ) + iza sin θ
=
z 2 − 2za cos θ + a2 cos2 θ + a2 sin2 θ
z(z − a cos θ) + iza sin θ
=
z 2 − 2za cos θ + a2
z(z − a cos θ) za sin θ
= +i 2 ......(1)
z 2 − 2za cos θ + a2 z − 2za cos θ + a2
z(z − a cos θ) za sin θ
Similarly Z[an cos nθ] − iZ[an sin nθ] = 2 2
−i 2 ......(2)
z − 2za cos θ + a z − 2za cos θ + a2

2z(z − a cos θ)
(1) + (2) ⇒ 2Z[an cos nθ] =
z 2 − 2za cos θ + a2
z(z − a cos θ)
⇒ Z[an cos nθ] = 2
z − 2za cos θ + a2
2za sin θ
(1) − (2) ⇒ 2Z[an sin nθ] = 2
z − 2za cos θ + a2
⇒ Z[an sin nθ] = 2
za sin θ
z − 2za cos θ + a2

ne t
Note:

n g .
Find Z[2n cos

2
]

e ri
W.K.T Z[an cos nθ]

i n =
e
z(z − a cos θ)
z 2 − 2za cos θ + a2

ng
π
put a = 2 and θ =
2
π

s y E n π
Z[2 cos n ]
2
=
z(z − 2 cos
2
π
)

z 2 − 2z(2) cos + 4
2

.E a Z[2n cos

2
] =
z2
z2 + 4

w w
Find Z[2n sin

2
]

w
4. Find Z[cos nθ] and Z[sin nθ]
Hint:Put a = 1

z(z − cos θ)
Ans : Z[cos nθ] =
z 2 − 2z cos θ + 1
z sin θ
Z[sin nθ] =
z 2 − 2z cos θ + 1

nπ nπ
Find Z[cos ] and Z[sin ]
2 2
Note:1

1. Z[e−at f (t)] = Z[f (t)]z→zeaT

2. Z[an f (t)] = Z[f (t)]z→ az

Page 6

Downloaded From : www.EasyEngineering.net


Downloaded From : www.EasyEngineering.net Umar Ali

Note:2
Find Z[nan ]
Solution:

Z[nan ] = Z[n]z→ az
z
= 2
z
(z − 1) z→ a
z
= a
z 2
−1
a
z
= a
 2
z−a
a
z a2
=
a (z − a)2
=
az
(z − a)2

ne t
Try Z[an cos nθ] and Z[an sin nθ]

n g .
e ri
Find Z[e−at sin bt]

i n e
ng
W.K.T Z[e−at f (t)] = Z[f (t)]z→zeaT

y E Z[e−at sin bt] = Z[sin bt]z→zeaT

as
= Z[sin bnT ]z→zeaT
z sin bT

E
=

w . =
z 2 − 2z cos bT + 1 z→zeaT

2
z e 2aT
zeaT sin bT
− 2zeaT cos bT + 1

w w
5. Find the Z-transform of
Solution:
1
(n + 1)(n + 2)

1 A B
Let = +
(n + 1)(n + 2) n+1 n+2
1 = A(n + 2) + B(n + 1)

Put z = −1 Put z = −2
⇒1=A ⇒ 1 = −B ⇒ B = −1

1 1 1
= −
(n + 1)(n + 2) n+1 n+2

Page 7

Downloaded From : www.EasyEngineering.net


Downloaded From : www.EasyEngineering.net Umar Ali

     
1 1 1
Z = Z −Z ......(1)
(n + 1)(n + 2) n+1 n+2
  ∞
1 X 1
Now Z = z −n
n+1 n=0
n + 1
z −1 z −2 z −3
1+ = + + + ···
 −12 −2
3
−3
4
z −4

z z z
= z + + + + ···
1 2 3 4
−1

= −z log 1 − z if |z| > 1
 
1
= −z log 1 −
z
 
z−1
= −z log
z
z
= z loge if |z| > 1
z−1
  ∞
1 X 1
Also Z = z −n
n+2 n=0
n + 2

=
1 z −1
+
2  3
+
z −2
4
+
z −3
5
+ ···

ne t
−2
z −3 z −4

.

2 z
+ ···

g
= z + +
2 3 4

= 2

z − log 1 − z −1
 z −1

1


ri n if |z| > 1

=

−z 2 log 1 −
1

+z
e e
in
z
 
z−1
=

n g
−z 2 log

z

+z

E
z
= z 2 loge +z if |z| > 1

Sub in (1)
a s y z−1

w . E Z

1
(n + 1)(n + 2)

= z loge
z−1
z
+ z 2 loge

z
z−1

+z

w w  
= (z − z 2 ) loge

z
z−1

+z

2n + 3
Find Z
(n + 1)(n + 2)  
z
Ans: (z + z 2 ) loge −z
z−1

6. Prove that Z[f (n + 1)] = z[F (z) − f (0)][Second Shifting Theorem]


Solution:


X
F (z) = Z[f (n)] = f (n)z −n
n=0
X∞
Z[f (n + 1)] = f (n + 1)z −n
n=0
X∞
= f (n + 1)z −n z −1 z
n=0

Page 8

Downloaded From : www.EasyEngineering.net


Downloaded From : www.EasyEngineering.net Umar Ali


X
= z f (n + 1)z −(n+1)
n=0

Put n + 1 = m,
if n = 0 ⇒ m = 1
if n = ∞ ⇒ m = ∞


X
we get, Z[f (n + 1)] = z f (m)z m
m=1
X∞
= z f (m)z m + zf (0)z −0 − zf (0)z −0
m=1
" ∞ #
X
m −0
= z f (m)z + f (0)z − zf (0)
m=1

X
= z f (m)z m − zf (0)
m=0
= zF (z) − zf (0)

ne t
Note:

n g .
If Z[f (n)] = f (z) then find Z[f (n − k)] and Z[f (n + k)]

Initial and Final value Theorem:


e ri
7. State and prove the Initial and final value theorem.

i n e
ng
Solution:
Initial value theorem:

y E
as
If Z[f (n)] = f (z) then f (0) = lim F (z)
z→∞

w . EW.K.T F (z) = Z[f (n)] =



X

n=0
f (n)z −n

w w = f (0) + f (1)z −1 + f (2)z −2 + f (3)z −3 + · · ·

= f (0) +
f (1) f (2) f (3)
 z
lim F (z) = lim f (0) +
+ 2 + 3 + ···
z z
f (1) f (2) f (3)
+ 2 + 3 + ···


z→∞ z→∞ z z z
= f (0)

Final value theorem:


If Z[f (n)] = f (z) then lim f (n) = lim (z − 1)F (z)
n→∞ z→1

W.K.T Z[f (n + 1)] = z[F (z) − f (0)]

= zF (z) − zf (0)

Z[f (n + 1)] − F (z) = zF (z) − zf (0) − F (z)

Z[f (n + 1)] − Z[f (n)] = (z − 1)F (z) − zf (0)

Page 9

Downloaded From : www.EasyEngineering.net


Downloaded From : www.EasyEngineering.net Umar Ali

Z[f (n + 1) − f (n)] = (z − 1)F (z) − zf (0)



X
[f (n + 1) − f (n)]z−n = (z − 1)F (z) − zf (0)
n=0
lim (z − 1)F (z) − f (0) = lim [f (1) − f (0) + f (2) − f (1) + f (3) − f (2) + · · · + f (n + 1) − f (n)]
z→1 n→∞

= lim [f (n) − f (0)]


n→∞

lim (z − 1)F (z) − f (0) = lim [f (n) − f (0)]


z→1 n→∞

lim (z − 1)F (z) = lim f (n)


z→1 n→∞

Convolution Theorem

Note:
an+1 − 1
1. 1 + a + a2 + a3 + . . . + an = ifa > 1
a−1
n+1
1−a
2. 1 + a + a2 + a3 + . . . + an = ifa < 1

t
1−a
z2
e
 

n
−1
8. Using Convolution theorem, find the Z
Solution:
(z − a)(z − b)

n g .
Z −1

z2
(z − a)(z − b)

= Z −1

z
z−a

Z −1


e
z
r
z−bi

= an ∗ bn
n

i n e
ng
X
= ar bn−r
r=0

E
n  r
X a
= bn

a s y =
r=0

bn 1 + +
b
a  a 2  a 3
+ + ... +
 a n 

.E
b b b b
  n+1
a

−1

w w = bn  b a

b
−1

w
 n+1
a − bn+1

= bn 
 bn+1 
a−b 

 n+1 b n+1 
n a −b b
= b
bn+1 a−b
an+1 − bn+1
=
a−b

z2
 
Using Convolution theorem, find the Z −1
(z − 1)(z − 3)
8z 2
 
−1
9. Using Convolution theorem, find the Z
(2z − 1)(4z + 1)

Page 10

Downloaded From : www.EasyEngineering.net


Downloaded From : www.EasyEngineering.net Umar Ali

Solution:
     
2
8z z z
Z −1  = Z −1 
    −1  
       Z   
1 1  1   −1 
2 z− 4 z+ z− z−
2 4 2 4
 n  n
1 −1
= ∗
2 4
n
X −1  1 n−r
 r
=
r=0
4 2
 n X n  r  −r
1 −1 1
=
2 r=0
4 2
 n X n  r
1 −1
= 2r
2 r=0
4
 n X n  r
1 −1
= 2̇
2 r=0
4
n 

t
 n X r
1 −1
=
2
 n "
r=0
2
2  3
. ne n #

g
 
1 −1 −1 −1 −1
= 1+ + + + ... +
2
 
2
−1
2
n+1 

ri n2 2

=
 n  1 −
1
2

n

2
−1
 


e e
i
1−
 
2

ng
  n+1 
−1
 n  1 −

E
1 2 
=
 

y 2

 3 

as
2
 n    n 
2 1 −1 −1

w . E =

=
3 2
 n 
2 1
3 2
1−

1+
2

1 −1
2

2
2
n 

w w =
 n
2 1
3 2
+
3

1 −1
4
n

8z 2
 
−1
Using Convolution theorem, find the Z
(2z − 1)(4z − 1)

14z 2
 
−1
Using Convolution theorem, find the Z
(7z − 1)(2z − 1)

z2
 
10. Find Z −1 using convolution theorem.
(z − a)2
Solution:

z2
     
−1 −1 z −1 z
Z = Z Z
(z − a)2 z−a z−a
= an ∗ an

Page 11

Downloaded From : www.EasyEngineering.net


Downloaded From : www.EasyEngineering.net Umar Ali

n
X
= ar an−r
r=0
n
X
= an
r=0
n
X
= an 1
r=0
= an [1 + 1 + 1 + · · · + 1]

= (n + 1)an

z3
 
−1
11. Find Z using convolution theorem.
(z − 4)3
Solution:

z3 z2
     
z
Z −1 = Z −1 Z −1
(z − 4)3 (z − 4)2 z−4
(n + 1)4n ∗ 4n

t
=

e
Xn
(r + 1)4r 4n−r

n
=

=
r=0
Xn
(r + 1)4n

n g .
r=0

4n
n
X

e ri
e
= (r + 1)

n
r=0

= i
= an [1 + 2 + 3 + · · · + (n + 1)]

ng
(n + 1)(n + 2) n
2
4

y E
as
z3
 
−1
12. Find Z
(z − 2)2 (z − 3)
Solution:

w
−1
. E

z3

−1

z2

−1

z


w
Z = Z Z
(z − 2)2 (z − 3) (z − 2)2 z−3

w =

=
(n + 1)2n ∗ 3n
Xn
(r + 1)2r 3n−r
r=0
n  r
n
X 2
= 3 (r + 1)
r=0
3
"    2  3  n #
n 2 2 2 2
= 3 1+2 +3 +4 + . . . + (n + 1)
3 3 3 3
2
Let S = 1 + 2x + 3x2 + 4x3 + . . . + nxn−1 + (n + 1)xn , where x =
3
xS = x + 2x2 + 3x3 + 4x4 + . . . + nxn + (n + 1)xn+1

(1 − x)S = (1 + x + x2 + x3 + . . . + xn ) − (n + 1)xn+1
1 − xn+1
= − (n + 1)xn+1
1−x
1 − xn+1 (n + 1)xn+1
∴S = −
(1 − x)2 1−x

Page 12

Downloaded From : www.EasyEngineering.net


Downloaded From : www.EasyEngineering.net Umar Ali

2 1
Put x = ⇒1−x=
3 3
 n+1  n+1
2 2
1− (n + 1)
3 3
∴S =  2 −  
2 2
3 3
"  n+1 #  n+1
2 2
= 9 1− − 3(n + 1)
3 3
 n+1  n+1
2 2
= 9− − 3(n + 1)
3 3
 n+1
2
= 9− [9 + 3n + 3]
3
 n+1
2
= 9− [12 + 3n]
3
 n+1
2
= 9 − 3(n + 4)
3

ne t
.
"  n+1 #
z3
 
−1 n 2
Z = 3 9 − 3(n + 4)
(z − 2)2 (z − 3) 3

i n
 n+1
2 g
r
n n+1
= 9·3 −3 (n + 4)

n
3 n+2

e e3
− (n + 4)2 n+1

Method of Partial fraction


ng i
Note:
y E
. E T ype1 :as f (x)
(x − a)(x − b)
=
A
+
x−a x−b
B

w w T ype2 :
f (x)
(x − a)2 (x − b)
=
A
+
B
x − a (x − a)2
+
C
x−b

w
f (x) A Bx + C
T ype3 : = +
(x − a)(x2 − bx − c) x − a x2 − bx − c

 
−1 10z
13. Find Z
z 2 − 3z + 2
Solution:

10z
Let F (z) =
z 2 − 3z + 2
F (z) 10
⇒ =
z (z − 1)(z − 2)
10 A B
Let = +
(z − 1)(z − 2) z−1 z−2
10 = A(z − 2) + B(z − 1)

Page 13

Downloaded From : www.EasyEngineering.net


Downloaded From : www.EasyEngineering.net Umar Ali

Put z = 1 Put z = 2
⇒ 10 = −A ⇒ 10 = B
A = −10

10 −10 10
= +
(z − 1)(z − 2) z−1 z−2
F (z) −10 10
= +
z z−1 z−2
z z
F (z) = −10 + 10
z−1 z−2
   
z z
f (n) = −10Z −1 + 10Z −1
z−1 z−2
= −10(1)n + 10 · 2n

t
 
z

e
−1
Evaluate Z
z 2 + 7z + 10

2z 2 + 3z
g . n
Find the inverse Z-transform of
(z + 2)(z − 4)

ri n
14. Find Z −1

z 3


n e e
i
(z − 1)2 (z − 2)

ng
Solution:

y E Let F (z) =
z3
(z − 1)2 (z − 2)

as
F (z) z2
⇒ =
z (z − 1)2 (z − 2)

w . E Let
z2
(z − 1)2 (z − 2)
z2
=
A
+
B
z − 1 (z − 1)2
+
C
z−2
= A(z − 1)(z − 2) + B(z − 2) + C(z − 1)2

w w
Put z = 1 Put z = 2 Eq.coeff of z 2
⇒ 1 = −B ⇒4=C 1=A+C ⇒A=1−C
B = −1 ⇒A=1−4
⇒ A = −3

z2 −3 −1 4
= + +
(z − 1)2 (z − 2) z − 1 (z − 1)2 z−2
F (z) −3 −1 4
= + +
z z − 1 (z − 1)2 z−2
z z z
F (z) = −3 − +4
z − 1 (z − 1)2 z−2

Page 14

Downloaded From : www.EasyEngineering.net


Downloaded From : www.EasyEngineering.net Umar Ali

     
z z z
f (n) = −3Z −1 − Z −1 + 4Z −1
z−1 (z − 1)2 z−2
= −3(1)n − n + 4 · 2n

 
−1 z
Find Z
(z − 1)2 (z + 1)

z(z 2 − z + 2)
 
Find Z −1
(z − 1)2 (z + 1)

z2
 
−1
15. Find Z by the method of partial fraction.
(z + 2)(z 2 + 4)
Solution:

z2
Let F (z) =
(z + 2)(z 2 + 4)
F (z) z

z
z
=
(z + 2)(z 2 + 4)
A Bz + C

ne t
.
Let = + 2
(z + 2)(z 2 + 4) z+2 z +4
z = A(z 2 + 4) + (Bz + C)(z + 2)

i n g
Put z = −2 Put z = 0

e e r Eq.coeff of z 2
⇒ −2 = 8A
1
⇒ 0 = 4A + 2C
−1
i n 0 = A + B ⇒ B = −A
1

ng
A=− ⇒ 2C = −4 B=
4 4 4
1

E
⇒C=
2

as y
w . E z
(z + 2)(z 2 + 4)
=
−1
4 + 4
z+2
1
z+
z2 + 4
1
2

w w F (z)
z
F (z)
=

=


1 1
+
1 z
+
1 1
4 z + 2 4 z2 + 4 2 z2 + 4
1 z
+
1 z2
+
1 z
4 z − (−2) 4 z + 4 2 z 2 + 4
2

z2
     
1 z 1 1 −1 2z
f (n) = − Z −1 + Z −1 2 + Z
4 z − (−2) 4 z + 22 4 z 2 + 22
1 1 nπ 1 n nπ
= − (−2)n + 2n cos + 2 sin
4 4 2 4 2

z 3 + 3z
 
Find Z −1
(z − 1)2 (z 2 + 1)

Residue Method

Note:
Simple Pole:

Page 15

Downloaded From : www.EasyEngineering.net


Downloaded From : www.EasyEngineering.net Umar Ali

{ResF (z)z n−1 }z=a = lim (z − a)F (z)z n−1


z→a
Pole of order n:
1 ∂ n−1
{ResF (z)z n−1 }z=a = lim (z − a)n F (z)z n−1
(n − 1)! z→a ∂z n−1
z 2 − 3z
 
16. Find Z −1 using residue method.
(z + 2)(z − 5)
Solution

z 2 − 3z
F (z) =
(z + 2)(z − 5)
z(z − 3)
z n−1 F (z) = z n−1
(z + 2)(z − 5)
z n (z − 3)
=
(z + 2)(z − 5)
Eq the domininator to zero

(z + 2)(z − 5) = 0

z = 5, −2

ne t
The poles are simple,z = 5, −2

W.K.T {ResF (z)z n−1 }z=a =


n
lim (z − a)F (z)z n−1
g .
z→a

e ri
When z = 5

i n e
{ResF (z)z n−1 }z=5 =
g lim (z − 5)
z n (z − 3)

En
z→5 (z + 2)(z − 5)
5n (2)
=

a s y =
2 n
7
5
7

When z = −2

w . E
w w {ResF (z)z n−1 }z=−2 =

=
lim (z + 2)
z→5

(−2)n (−5)
z n (z − 3)
(z + 2)(z − 5)

−7
5
= (−2)n
7

∴ f (n) = Sum of the Residues


2 n 5
= 5 + (−2)n
7 7

2z 2 + 3z + 12
17. If U (z) = ,find the values of u2 and u3
(z − 1)4

Page 16

Downloaded From : www.EasyEngineering.net


Downloaded From : www.EasyEngineering.net Umar Ali

Solution

2z 2 + 3z + 12
U (z) =
(z − 1)4
2z 2 + 3z + 12
z n−1 U (z) = z n−1
(z − 1)4
Eq the domininator to zero

(z − 1)4 = 0

z = 1, 1, 1, 1

The poles are order 4,z = 1


1 ∂ n−1
W.K.T {ResF (z)z n−1 }z=a = lim (z − a)n F (z)z n−1
(n − 1)! z→a ∂z n−1

When z = 1

{ResF (z)z n−1 }z=1 =


1 ∂3z
lim 3 (z − 1)4 z n−1
3! z→1 ∂z
2z 2 + 3z + 12
(z − 1)4

ne t
=
1
6 z→1 ∂z
1
∂3z
lim 3 [2z n+1 + 3z n + 12z n−1 ]
∂2z
n g .
=
6 z→1 ∂z
1 ∂z

e ri
lim 2 [2(n + 1)z n + 3nz n−1 + 12(n − 1)z n−2 ]

[2(n + 1)nz n−1 + 3n(n − 1)z n−2 + 12(n − 1)(n − 2)z n−3 ]
=

=
6
1
lim
z→1 ∂z

i n e
lim [2(n + 1)n(n − 1)z n−2 + 3n(n − 1)(n − 2)z n−3 + 12(n − 1)(n − 2)(n − 3)z n−4 ]

ng
6 z→1
1
= [2(n + 1)n(n − 1) + 3n(n − 1)(n − 2) + 12(n − 1)(n − 2)(n − 3)]

E
6

∴ f (n) =
as y
Sum of the Residues

w . E =
1
6
[2(n + 1)n(n − 1) + 3n(n − 1)(n − 2) + 12(n − 1)(n − 2)(n − 3)]

w w Put n=2, u2 =
1
6
1
[2 · 3 · 2 · 1] = 2

Put n=3, u3 = [2 · 4 · 3 · 2 + 3 · 3 · 2 · 1] = 11
6

2z 2 + 5z + 14
If U (z) = ,find the values of u2 and u3
(z − 1)4

 
−1 z(z + 1)
Find Z
(z − 1)3

Page 17

Downloaded From : www.EasyEngineering.net


Downloaded From : www.EasyEngineering.net Umar Ali

Formation of Differential Equation

18. Form a differential equation by eliminating arbitrary constants yn = a + b3n


Solution:

yn = a + b3n

yn+1 = a + b3n+1

= a + 3b3n

yn+2 = a + b3n+2

= a + 9b3n

Eliminating a and b3n ,

t

yn yn+1 yn+2



1

1 1



= 0

. ne
g

1 3 9
yn [9 − 3] − yn+1 [9 − 1] + yn+2 [3 − 1]

ri n = 0

6yn − 8yn+1 + 2yn+2

n
yn+2 − 4yn+1 + 3yn
e e =

=
0

ng i
19. Form a differential equation by eliminating arbitrary constants yn = (A + Bn)2n
Solution:
y E
. E as yn = A2n + Bn2n

w w yn+1 = A2n+1 + B(n + 1)2n+1

= 2A2n + 2B(n + 1)2n

w yn+2 = A2n+2 + B(n + 2)2n+2

= 4A2n + 4B(n + 2)2n

Eliminating a and b3n ,


yn yn+1 yn+2


= 0

1 2 4


n 2(n + 1) 4(n + 2)
yn [8(n + 2) − 8(n + 1)] − yn+1 [4(n + 2) − 4n] + yn+2 [2(n + 1) − 2n] = 0

8yn − 8yn+1 + 2yn+2 = 0

yn+2 − 4yn+1 + 4yn = 0

Page 18

Downloaded From : www.EasyEngineering.net


Downloaded From : www.EasyEngineering.net Umar Ali

Form a differential equation by eliminating arbitrary constants yn = a − b3n

Form a differential equation by eliminating arbitrary constants yn = an + b2n

Solving Linear Differential equation

Formula:

Z[yn ] = F (z)

Z[yn+1 ] = zF (z) − zy(0)

Z[yn+2 ] = z 2 F (z) − z 2 y(0) − zy(1)

= z 3 F (z) − z 3 y(0) − z 2 y(1) − zy(2)

t
Z[yn+3 ]

20. Using Z-transform solve un+2 − 5un+1 + 6un = 4n given that u0 = 0, u1 = 1.


. ne
Solution:

i n g
Given un+2 − 5un+1 + 6un = 4n
Applying Z-transforms on both side,
e e r
i n
ng
Z[un+2 ] − 5Z[un+1 ] + 6Z[un ] = Z[4n ]

E
z
z 2 F (z) − z 2 u(0) − zu(1) − 5[zF (z) − zu(0)] + 6F (z) =

as y
Given u0 = 0, u1 = 1 F (z)[z 2 − 5z + 6] − z =
z−4
z
z−4
z

w . E F (z)(z − 2)(z − 3) =

=
z−4
+z
z + z 2 − 4z
z−4

w w F (z) =

=
z 2 − 3z
(z − 4)(z − 2)(z − 3)
z(z − 3)
(z − 4)(z − 2)(z − 3)
z
=
(z − 4)(z − 2)
By Residue Method,
z
z n−1 F (z) = z n−1
(z − 4)(z − 2)
z n (z − 3)
=
(z − 4)(z − 2)
Eq the domininator to zero

(z − 4)(z − 2) = 0

z = 2, 4

The poles are simple,z = 2, 4

Page 19

Downloaded From : www.EasyEngineering.net


Downloaded From : www.EasyEngineering.net Umar Ali

W.K.T {ResF (z)z n−1 }z=a = lim (z − a)F (z)z n−1


z→a

When z = 2

z n (z − 2)
{ResF (z)z n−1 }z=2 = lim (z − 2)
z→2 (z − 4)(z − 2)
n
2
=
−2
= −(2n−1 )

When z = 4

z n (z − 4)
{ResF (z)z n−1 }z=4 = lim (z − 4)
z→4 (z − 4)(z − 2)
n
4
=
2
= 22n−1

ne t
∴ un = Sum of the Residues

n g .
= −(2n−1 ) + 22n−1

e ri
i n e
Solve the differential equation y(n + 3) − 3y(n + 1) + 2y(n) = 0 given that y(0) = 4, y(1) = 0 and y(2) = 8.

ng
8 4
Ans:y(n) = n + · (−2)n
3 3

y E
as
Solve the differential equation un+2 + 3un+1 + 2un = 0 given that u0 = 1, u1 = 2

w . E
Ans:un = 4(−1)n − 3(−2)n

w w
Solve the differential equation yn+2 + 4yn+1 + 3yn = 2n given that y0 = 0, y1 = 1
2 1
Ans:yn = − (−3)n + · (−1)n +
5 3
1
15
· (2)n

Solve the differential equation y(n) + 3y(n − 1) − 4y(n − 2) = 0, n ≥ 2 given that y(0) = 3, y(1) = −2
Hint:Changing n to n + 2
Ans:y(n) = (−4)n + 2

Solve the differential equation y(n + 3) − 3y(n + 1) + 2y(n) = 0 with y(0) = 4, y(1) = 0 and y(2) = 8
8 4
Ans:y(n) = + (−2)n
3 3

21. Solve yn+2 + 6yn+1 + 9yn = 2n given that y0 = y1 = 0 using Z-transform


Solution:

Page 20

Downloaded From : www.EasyEngineering.net


Downloaded From : www.EasyEngineering.net Umar Ali

Given yn+2 + 6yn+1 + 9yn = 2n


Applying Z-transforms on both side,

Z[yn+2 ] + 6Z[yn+1 ] + 9Z[yn ] = Z[2n ]


z
z 2 F (z) − z 2 y(0) − zy(1) + 6[zF (z) − zy(0)] + 9F (z) =
z−2
z
Given y0 = y1 = 0 F (z)[z 2 + 6z + 9] =
z−2
z
F (z)(z + 3)2 =
z−2
z
F (z) =
(z − 2)(z + 3)2
By Residue Method,
z
z n−1 F (z) = z n−1
(z − 2)(z + 3)2
zn
=
(z − 2)(z + 3)2
Eq the domininator to zero

(z − 2)(z + 3)2 = 0

ne t
z

n
= 2, −3, −3
g .
The poles are simple,z = 2

e ri
The poles are order 2,z = −3, 4

i n
W.K.T {ResF (z)z n−1 }z=a e = lim (z − a)F (z)z n−1

ng
z→a

When z = 2

y E
as
z n (z − 2)
{ResF (z)z n−1 }z=2 = lim (z − 2)
z→2 (z − 2)(z + 3)2

w . E =
n
2
25

w w
{ResF (z)z n−1 }z=a =
1
lim
∂ n−1
(n − 1)! z→a ∂z n−1
(z − a)n F (z)z n−1 When z = −3

1 ∂z zn
{ResF (z)z n−1 }z=−3 = lim (z + 3)2
(1)! z→−3 ∂z (z − 2)(z + 3)2
n
∂z z
= lim [ ]
z→−3 ∂z z − 2
(z − 2)nz n−1 − z n (1)
= lim [ ]
z→−3 (z − 2)2
−5n(−3)n−1 − (−3)n
=
25
(−3)n
−5n − (−3)n
= −3
25
1 1
= n(−3)n − (−3)n
15 25

Page 21

Downloaded From : www.EasyEngineering.net


Downloaded From : www.EasyEngineering.net Umar Ali

∴ un = Sum of the Residues


2n 1 1
= + n(−3)n − (−3)n
25 15 25

Solve by Z-transform un+2 − 2un+1 + un = 2n with u0 = 2 and u1 = 1.


Ans:un = 2n + 1 − 2n

Solve the equation yn+2 − 3yn+1 + 2yn = 2n with y0 = y1 = 0


Ans:yn = 1 + n2n−1 − 2n

Solve the differential equation y(k + 2) − 4y(k + 1) + 4y(k) = 0 with y(0) = 1, y(1) = 0
Ans:y(n) = 2k − k · 2k

Solve the equation yn+2 + 4yn+1 − 5yn = 24n − 8 with y0 = 3, y1 = −5

ne t
Ans:yn = (−5)n + 2n2 − 4n + 2[use partial fraction method]

22. Solve the differential equation y(k + 2) + y(k) = 1 with y(0) = y(1) = 0
n g .
Solution:

e ri
Given y(n + 2) + y(n) = 0
Applying Z-transforms on both side,
i n e
E ng
Z[yn+2 ] + Z[yn ] = Z[1]

a s y
z 2 F (z) − z 2 y(0) − zy(1) + F (z) =
z
z−1
z

.E
Given y0 = y1 = 0 F (z)[z 2 + 1] =
z−1
z

w w By Partial fraction Method,


F (z) =
(z − 1)(z 2 + 1)

w Let
1

F (z)
z

(z − 1)(z 2 + 1)
=

=
A
z−1
1
(z − 1)(z 2 + 1)

+ 2
Bz + C
z +1
1 = A(z 2 + 1) + (Bz + C)(z − 1)

Put z = 1 Put z = 0 Eq.coeff of z 2


⇒ 1 = 2A 1=A−C ⇒C =A−1 0 = A + B ⇒ B = −A
1 1 1
A= ⇒C = −1 B=−
2 2 2
1
⇒C=−
2

Page 22

Downloaded From : www.EasyEngineering.net


Downloaded From : www.EasyEngineering.net Umar Ali

1 −1 −1
1 z+
∴ = 2 + 2 2
(z − 1)(z 2 + 1) z−1 z2 + 1
F (z) 1 1 1 z 1 1
= − +
z 2 z − 1 2 z2 + 1 2 z2 + 1
   2   
1 −1 z 1 −1 z 1 −1 z
F (z) = Z − Z − Z
2 z−1 2 z2 + 1 2 z2 + 1
2
1 z 1 z 1 z
y(n) = − −
2 z − 1 2 z + 1 2 z2 + 1
2

1h nπ nπ i
= 1 − cos − sin
2 2 2 
1 kπ kπ
∴ y(k) = 1 − cos − sin
2 2 2

ne t
n g .
e ri
i n e
E ng
as y
w . E
w w

Page 23

Downloaded From : www.EasyEngineering.net


Downloaded From : www.EasyEngineering.net Umar Ali

Unit-4
Fourier Transforms

The Fourier transform pair for f (x) is:


The Fourier transform of f (x) is

Z∞
1
F (s) = F [f (x)] = √ f (x)eisx dx

−∞

The Inverse Fourier transform is

Z∞
1
f (x) = √ F (s)e−isx ds

−∞

Note:

ne t
.
Parseval’s identity is
Z∞ Z∞
|F (s)|2 ds = |f (x)|2 dx

i n g
r
−∞ −∞

1. Show that the Fourier transform of f (x) =



 a2 − x2 , |x| < a;

n e e r 
sin as − as cos as
is 2
2

. Hence

deduce that
R∞ sin t − t cos t
t3
π
 0,

ng i
|x| > a > 0

dt = . Using parseval’s identity, show that


4 t 3
s3
R∞ sin t − t cos t 2
 
π

dt =
π
15
.
Solution:
0

y E 0

as
The Fourier transform of f (x) is

w . E
F (s) = F [f (x)] = √
1

Z∞

−∞
f (x)eisx dx

w w = √
1

Za

−a

(a2 − x2 )(cos sx + i sin sx)dx


Za Za
1
= √  (a2 − x2 ) cos sxdx + i (a2 − x2 ) sin sxdx

−a −a
 a 
Z
1 
= √ 2 (a2 − x2 ) cos sxdx + 0

0
    a
2 sin sx − cos sx − sin sx
= √ (a2 − x2 ) − (−2x) + (−2)
2π s s2 s3 0
   a
2 2 2 sin sx
 cos sx  sin sx
= √ (a − x ) − 2x +2
2π s s2 s3 0
r  
2 cos sa sin sa
= −2a 2 + 2 3
π s s
r  
2 −as cos as + sin as
= 2
π s3

Page 1

Downloaded From : www.EasyEngineering.net


Downloaded From : www.EasyEngineering.net Umar Ali

r  
2 sin as − as cos as
= 2
π s3

The Inverse Fourier transform is

Z∞
1
f (x) = √ F (s)e−isx ds

−∞
Z∞
r  
1 2 sin as − as cos as
= √ 2 (cos sx − i sin sx)ds
2π π s3
−∞
 ∞
Z∞ 

Z   
2 sin as − as cos as sin as − as cos as
= cos sxds − i sin sxds
π s3 s3
−∞ −∞
 ∞ 
Z  
2 sin as − as cos as
= 2 cos sxds + 0
π s3
0
Z∞  
4 sin as − as cos as
f (x) =
π
0
s3
cos sxds

ne t
n g .
i
Z∞  

r
sin as − as cos as π
cos sxds = f (x)
0
s3

n
Put x = 0 and a = 1
e e 4

Z∞ 
ng i
E

sin s − s cos s π
ds = f (0)

as y 0
s3

=
4
π
4
(1)

w . E ⇒
Z∞ 

0
sin s − s cos s
s3

ds =
π
4

w w
Put s = t ⇒ ds = dt

Z∞  
sin t − t cos t π
dt =
t3 4
0

Using Parseval’s identity

Z∞ Z∞
2
|F (s)| ds = |f (x)|2 dx
−∞ −∞
Z∞  2 Za
2 sin as − as cos as 2
4 ds = a 2 − x2 dx
π s3
−∞ −a
Z∞  2 Za
8 sin as − as cos as
a4 − 2a2 x2 + x4 dx

2 ds = 2
π s3
0 0

Page 2

Downloaded From : www.EasyEngineering.net


Downloaded From : www.EasyEngineering.net Umar Ali

Z∞  2 3
a
x5

8 sin as − as cos as 4 2x
ds = a x − 2a +
π s3 3 5 0
0
Z∞  2
2a5 a5
 
sin as − as cos as π
ds = a5 − +
s3 8 3 5
0
a5 π
 
2 1
= 1− +
8 3 5
a5 π
 
15 − 10 + 3
=
8 15
a5 π
=
15

Put a = 1

Z∞  2
sin s − s cos s π
ds =
s3 15
0
Put s = t
Z∞ 
⇒ ds = dt
sin t − t cos t
2
π

ne t
.
dt =
t3 15
0

i n g
r

 a2 − x2 , |x| ≤ a;

e
R∞ sin s − s cos s
2. Find the Fourier transform of f (x) = Hence prove that cos 2s ds =


.
 0, |x| > a > 0

i n e 0 s3

ng
16
Solution:
The Fourier transform of f (x) is

y E
. E
F (s)
as
= F [f (x)] = √
1

Z∞

−∞
f (x)eisx dx

w w = √
1

Za
(a2 − x2 )(cos sx + i sin sx)dx

w
−a
 
Za Za
1
= √  (a2 − x2 ) cos sxdx + i (a2 − x2 ) sin sxdx

−a −a
 a 
Z
1 
= √ 2 (a2 − x2 ) cos sxdx + 0

0
    a
2 sin sx − cos sx − sin sx
= √ (a2 − x2 ) − (−2x) + (−2)
2π s s2 s3 0
   a
2 2 2 sin sx
 cos sx  sin sx
= √ (a − x ) − 2x +2
2π s s2 s3 0
r  
2 cos sa sin sa
= −2a 2 + 2 3
π s s
r  
2 −as cos as + sin as
= 2
π s3
r  
2 sin as − as cos as
= 2
π s3

Page 3

Downloaded From : www.EasyEngineering.net


Downloaded From : www.EasyEngineering.net Umar Ali

The Inverse Fourier transform is

Z∞
1
f (x) = √ F (s)e−isx ds

−∞
Z∞
r  
1 2 sin as − as cos as
= √ 2 (cos sx − i sin sx)ds
2π π s3
−∞
 ∞
Z∞ 

Z   
2 sin as − as cos as sin as − as cos as
= cos sxds − i sin sxds
π s3 s3
−∞ −∞
 ∞ 
Z  
2 sin as − as cos as
= 2 cos sxds + 0
π s3
0
Z∞  
4 sin as − as cos as
f (x) = cos sxds
π s3
0

Z∞ 
sin as − as cos as

π

ne t
0
s3
cos sxds =

n
4
f (x)

g .
Put x = 1
2 and a = 1

e ri
Z∞ 
sin s − s cos s

i n

s
cos ds = e π
f
 
1

ng
s3 2 4 2
0
π3

E
=
44

a s

y Z∞ 
sin s − s cos s
s3

s
cos ds =
2

16

.E
0

w

 1 − x2 , |x| ≤ 1; r 
2 sin s − s cos s


w 0
w
3. Show that the Fourier transform of f (x) =

R∞ sin t − t cos t
t 3
π
 0, |x| > 1

dt = . Using parseval’s identity, show that


4 0
is 2

t 3
π
R∞ sin t − t cos t 2
 
s3

dt =
π
15
.
. Hence deduce that

Solution:
The Fourier transform of f (x) is

Z∞
1
F (s) = F [f (x)] = √ f (x)eisx dx

−∞
Z1
1
= √ (1 − x2 )(cos sx + i sin sx)dx

−1
 1 
Z Z1
1 
= √ (1 − x2 ) cos sxdx + i (1 − x2 ) sin sxdx

−1 −1

Page 4

Downloaded From : www.EasyEngineering.net


Downloaded From : www.EasyEngineering.net Umar Ali

 1 
Z
1 
= √ 2 (1 − x2 ) cos sxdx + 0

0
    1
2 2 sin sx − cos sx − sin sx
= √ (1 − x ) − (−2x) + (−2)
2π s s2 s3 0
   1
2 sin sx  cos sx  sin sx
= √ (1 − x2 ) − 2x 2
+2
2π s s s3 0
r  
2 cos s sin s
= −2 2 + 2 3
π s s
r  
2 −s cos s + sin s
= 2
π s3
r  
2 sin s − s cos s
= 2
π s3

The Inverse Fourier transform is

Z∞

t
1
f (x) = √ F (s)e−isx ds

1
−∞
Z∞
r 
2 sin s − s cos s


. ne
= √

−∞
2
π s3
(cos sx − i sin sx)ds

i n g
r
 ∞
Z∞ 

e
Z   
2 sin s − s cos s sin as − as cos as
= cos sxds − i sin sxds
π
−∞
 ∞
s3
−∞

i

n
s3
e
ng
Z  
2 sin s − s cos s
= 2 cos sxds + 0
π s3
0

f (x) =
4
π
Z∞ 

s y E
sin s − s cos s
s3

cos sxds

.E a 0

w w Z∞ 
sin s − s cos s
s3

cos sxds =
π
4
f (x)

w
0

Put x = 0

Z∞  
sin s − s cos s π
ds = f (0)
s3 4
0
π
= (1)
4
Z∞  
sin s − s cos s π
⇒ ds =
s3 4
0

Put s = t ⇒ ds = dt

Z∞  
sin t − t cos t π
dt =
t3 4
0

Page 5

Downloaded From : www.EasyEngineering.net


Downloaded From : www.EasyEngineering.net Umar Ali

Using Parseval’s identity

Z∞ Z∞
2
|F (s)| ds = |f (x)|2 dx
−∞ −∞
Z∞  2 Z1
2 sin s − s cos s 2
4 ds = 1 − x2 dx
π s3
−∞ −1
Z∞  2 Z1
8 sin s − s cos s
1 − 2x2 + x4 dx

2 ds = 2
π s3
0 0
Z∞  2 1
x3 x5

8 sin s − s cos s
ds = x−2 +
π s3 3 5 0
0
Z∞  2  
sin s − s cos s π 2 1
ds = 1− +
s3 8 3 5
0
 
π 15 − 10 + 3

t
=
8 15
=
π
15

. ne
Z∞ 
sin s − s cos s
s3
2
ds =
π
15

i n g
Put s = t
0

Z∞ 
⇒ ds = dt
sin t − t cos t
2

e
π e r
t3
i n dt =
15

ng
0

E

 1 − x2 , |x| ≤ 1; R∞ sin s − s cos s

y

4. Find the Fourier transform of f (x) = Hence prove that cos 2s ds = .

as
s3 16
 0, |x| > 1 0

Solution:

. E
The Fourier transform of f (x) is

w
w
Z∞
1
F (s) = F [f (x)] = √ f (x)eisx dx

w = √
1

Z1

−∞

(1 − x2 )(cos sx + i sin sx)dx


−1
 1 
Z Z1
1 
= √ (1 − x2 ) cos sxdx + i (1 − x2 ) sin sxdx

−1 −1
 1 
Z
1
= √ 2 (1 − x2 ) cos sxdx + 0

0
    1
2 sin sx − cos sx − sin sx
= √ (1 − x2 ) − (−2x) + (−2)
2π s s2 s3 0
   1
2 sin sx  cos sx  sin sx
= √ (1 − x2 ) − 2x +2
2π s s2 s3 0
r  
2 cos s sin s
= −2 2 + 2 3
π s s

Page 6

Downloaded From : www.EasyEngineering.net


Downloaded From : www.EasyEngineering.net Umar Ali

 r 
2 −s cos s + sin s
= 2
π s3
r  
2 sin s − s cos s
= 2
π s3

The Inverse Fourier transform is

Z∞
1
f (x) = √ F (s)e−isx ds

−∞
Z∞
r  
1 2 sin s − s cos s
= √ 2 (cos sx − i sin sx)ds
2π π s3
−∞
 ∞
Z∞ 

Z   
2 sin s − s cos s sin s − s cos s
= cos sxds − i sin sxds
π s3 s3
−∞ −∞
 ∞ 
Z  
2 sin s − s cos s
= 2 cos sxds + 0
π
Z∞ 
0
s3

ne t
.

4 sin s − s cos s
f (x) = cos sxds

Z∞ 
π
0
s3

i n g
r

sin s − s cos s π
cos sxds = f (x)
0
s3

Put x = 21
4

n e e
Z∞ 
sin s − s cos s
s3

s
cos ds
2
π
= f
4
 
1
2
ng i
E
0
π3

Z∞ 
sin s − s cos s


as
s y =
44

E
cos ds =
0

w .
s3 2 16

w

 a − |x|, |x| < a; R∞ sin t 2
 

w
5. Find the Fourier transform of f (x) = and deduce the value of dt and
 0, |x| > a > 0 0 t

  4
R sin t
dt.
0 t
Solution:
The Fourier transform of f (x) is

Z∞
1
F (s) = F [f (x)] = √ f (x)eisx dx

−∞
Za
1
= √ (a − |x|)(cos sx + i sin sx)dx

−a
 
Za Za
1
= √  (a − |x|) cos sxdx + i (a − |x|) sin sxdx

−a −a

Page 7

Downloaded From : www.EasyEngineering.net


Downloaded From : www.EasyEngineering.net Umar Ali

 a 
Z
1 
= √ 2 (a − x) cos sxdx + 0

0
  a
2 sin sx − cos sx
= √ (a − x) − (−1)
2π s s2 0
  a
2 sin sx  cos sx 
= √ (a − x) −
2π s s2 0
r   
2 cos sa 1
= − 2 − − 2
π s s
r  
2 1 − cos as
=
π s2
!
2 2 sin2 as
r
2
=
π s2
!
2 sin2 as
r
2
= 2
π s2

The Inverse Fourier transform is

Z∞

ne t
f (x) = √
1

−∞
F (s)e−isx ds

n g .
= √
1

2
Z∞
r

π
2 sin2 as
s2
2
!
(cos sx − i sin sx)ds

e ri
2
 ∞
−∞
Z 2 as
sin 2
!

i n Z∞
e
2 as
sin 2
! 

ng
= cos sxds − i sin sxds
π s2 s2
−∞ −∞

E
 ∞ ! 
sin2 as
Z
2

y
2
= 2 cos sxds + 0
s2

as
π
0
Z∞ !
sin2 as

E
4

.
2
f (x) = cos sxds
π s2

w
0

w w Z∞
sin2 as
s2
2
!
cos sxds =
π
4
f (x)
0

Put x = 0 and a = 1

Z∞ !
sin2 s
2 π
ds = f (0)
s2 4
0
π
= (1)
4
Z∞ !
sin2 s
2 π
⇒ ds =
s2 4
0

Page 8

Downloaded From : www.EasyEngineering.net


Downloaded From : www.EasyEngineering.net Umar Ali

s
Put =t ⇒ s = 2t ⇒ ds = 2dt
2

Z∞ 
sin2 t

π
2dt =
4t2 4
0
Z∞ 
sin2 t

1 π
dt =
2 t2 4
0
Z∞  2
sin t π
dt =
t 2
0

Using Parseval’s identity

Z∞ Z∞
|F (s)|2 ds = |f (x)|2 dx
−∞ −∞
Z∞ !2 Za
sin2 as

t
2
4 2
ds = (a − |x|)2 dx
−∞
π
Z∞
s2

sin2 as
!2
−a
Za
. ne
g
8
2 2
ds = 2 (a − x)2 dx
π
0
Z∞
s2
!2

ri
Za n 0

8
π
0
sin2 as

n
s2
2
ds =

e e 0
(a2 − 2ax + x2 )dx

8
π
Z∞

ng isin2 as
s2
2
!2
ds =

2 x2
a x − 2a +
2
x3
3
a

s y E Z∞
sin2 as
s2
2
!2
ds =
π
8

a −a +
a3
3
3 3


.E a 0

=
 3
π a
8 3

w w =
a3 π
24

w
Put a = 1
Z∞ !
sin4 s
2 π
ds =
s4 24
0
s
Put = t ⇒ s = 2t ⇒ ds = 2dt
2
Z∞ 
sin4 t

π
2dt =
16t4 24
0
Z∞ 
sin4 t

1 π
dt =
8 t4 24
0
Z∞ 
sin4 t

π
dt =
t4 3
0

Page 9

Downloaded From : www.EasyEngineering.net


Downloaded From : www.EasyEngineering.net Umar Ali


 1 − |x|, |x| < 1; R∞ sin t 2
 
6. Find the Fourier transform of f (x) = and deduce the value of dt and
 0, |x| > 1 0 t

  4
R sin t
dt.
0 t
Solution:
The Fourier transform of f (x) is

Z∞
1
F (s) = F [f (x)] = √ f (x)eisx dx

−∞
Z1
1
= √ (1 − |x|)(cos sx + i sin sx)dx

−1
 1 
Z Z1
1 
= √ (1 − |x|) cos sxdx + i (1 − |x|) sin sxdx

−1 −1

t
 1 
Z
1 
= √

2 (1 − x) cos sxdx + 0
0

. ne
g
  1
2 sin sx − cos sx
= √ (1 − x) − (−1)

2

s
sin sx  cos sx 

s2
1
0

ri n
= √

r 
2
(1 − x)

cos s

s

1

s2

n
0

e e
=

=
r 
π
− 2 − − 2
s
2 1 − cos s

s

ng i
y E
r
π s2
2 2 sin2 2s
!

as
=
π s2

E
!
2 sin2 2s
r

w . = 2
π s2

w w
The Inverse Fourier transform is

f (x) = √
1
Z∞
F (s)e−isx ds

−∞
Z∞ !
2 sin2 2s
r
1
= √ 2 (cos sx − i sin sx)ds
2π π s2
−∞
 ∞
Z∞
! ! 
2 s 2 s
sin 2 sin 2
Z
2
= cos sxds − i sin sxds
π s2 s2
−∞ −∞
 ∞ ! 
sin2 2s
Z
2
= 2 cos sxds + 0
π s2
0
Z∞ !
4 sin2 s
2
f (x) = cos sxds
π s2
0

Page 10

Downloaded From : www.EasyEngineering.net


Downloaded From : www.EasyEngineering.net Umar Ali

Z∞ !
sin2 s
2 π
cos sxds = f (x)
s2 4
0

Put x = 0

Z∞ !
sin2 s
2 π
ds = f (0)
s2 4
0
π
= (1)
4
Z∞ !
sin2 s
2 π
⇒ ds =
s2 4
0

s
Put =t ⇒ s = 2t ⇒ ds = 2dt
2

Z∞ 
sin2 t

π

t
2dt =
4t2 4

1
0
Z∞ 
sin2 t

dt =
π

. ne
2
0
Z∞ 
t2 4

i n g
r
2
sin t π

n
t
dt =

e e
2

Using Parseval’s identity

Z∞
ng i Z∞

y E |F (s)| ds = 2
|f (x)|2 dx

as
−∞ −∞
Z∞ !2 Z1
sin2 s

E
2 2
(1 − |x|)2 dx

.
4 ds =
π s2
−∞ −1

w w 2
8
π
Z∞
sin2
s2
s
2
!2
ds = 2
Z1
(1 − x)2 dx

w 8
π
0
Z∞

0
sin2
s2
s
2
!2
ds =
Z1

0
0

(1 − 2x + x2 )dx

Z∞ !2 1
sin2 s
x2 x3

8 2
ds = a2 x − 2a +
π s2 2 3 0
0
Z∞ !2
sin2 s  
2 π 1
ds = 1−1+
s2 8 3
0
 
π 1
=
8 3
π
=
24
Z∞ !
sin4 s
2 π
ds =
s4 24
0

Page 11

Downloaded From : www.EasyEngineering.net


Downloaded From : www.EasyEngineering.net Umar Ali

s
Put =t ⇒ s = 2t ⇒ ds = 2dt
2

Z∞ 
sin4 t

π
2dt =
16t4 24
0
Z∞ 
sin4 t

1 π
dt =
8 t4 24
0
Z∞ 
sin4 t

π
dt =
t4 3
0


 1, |x| < a; R∞ sin t R∞ sin t 2
 
7. Find the Fourier transform of f (x) = and deduce the value of dt and dt.
 0, |x| > a 0 t 0 t
Solution:
The Fourier transform of f (x) is

1
Z∞

ne t
F (s) = F [f (x)] = √

Za

−∞
f (x)eisx dx

n g .
= √
1

−a
r
(1)(cos sx + i sin sx)dx

e i
=
1

i
Za

n
√  cos sxdx + i sin sxdx eZa

ng

−a −a
 a 

E
Z
1 
= √ 2 cos sxdx + 0

as y = √
2


0
sin sx
a

w . E =

r 
2 sin sa
π s
s

0

w w
The Inverse Fourier transform is

1
Z∞
f (x) = √ F (s)e−isx ds

−∞
Z∞
r  
1 2 sin sa
= √ (cos sx − i sin sx)ds
2π π s
−∞
 ∞
Z∞ 

Z   
1 sin sa sin sa
= cos sxds − i sin sxds
π s s
−∞ −∞
 ∞ 
Z  
1 sin sa
= 2 cos sxds − 0
π s
0
Z∞  
2 sin sa
f (x) = cos sxds
π s
0

Page 12

Downloaded From : www.EasyEngineering.net


Downloaded From : www.EasyEngineering.net Umar Ali

Z∞  
sin sa π
cos sxds = f (x)
s 2
0

Put x = 0 and a = 1

Z∞  
sin s π
ds = f (0)
s 2
0

Put s = x ⇒ ds = dx

Z∞  
sin x π
⇒ dx =
x 2
0

Using Parseval’s identity

Z∞ Z∞

−∞
|F (s)| ds =2

−∞
|f (x)|2 dx

ne t
Z∞
2
π

sin as
s
2
ds =
Za
(1)2 dx

n g .
−∞

2
2
Z∞ 
sin as
2
ds =
−a

2
Za

e
dx
ri
π
0
Z∞ 
s

i
2
n
0
e
ng
2 sin as
ds = [x]a0
π s
0

s y E Z∞ 
sin as
2
ds =
= a
π
a

Put a = 1
.E a 0
s 2

w w Z∞ 
sin s
2
ds =
π

w
Put s = x ⇒ ds = dx
0
s 2

Z∞  2
sin x π
dx =
x 2
0


 1, |x| < 2; R∞ sin x R∞ sin x 2
 
8. Find the Fourier transform of f (x) = and hence evaluate dx and dx.
 0, |x| > 2 0 x 0 x

Page 13

Downloaded From : www.EasyEngineering.net


Downloaded From : www.EasyEngineering.net Umar Ali

Note:

eax
Z
1. eax cos bxdx = (a cos bx + b sin bx)
a2 + b2
eax
Z
2. eax sin bxdx = (a sin bx − b cos bx)
a + b2
2

Z∞ ∞
e−ax

3. e−ax cos sxdx = (−a cos sx + s sin sx)
(−a)2 + s2 0
0
 
1
= 0− 2 (−a)
a + s2
a
=
s + a2
2
Z∞ ∞
e−ax

4. e−ax sin sxdx = (−a sin sx − s cos sx)
(−a)2 + s2 0
0
 
1
= 0− 2 (−s)
a + s2
=
s
s2 + a2

ne t
9. Find the Fourier Transform of e−a|x| , a > 0.Show that
R∞ cos sx
(a2 + s2 )
π
ds = e−a|x| .

n
2
g .
Hence deduce that F [xe −a|x|
]=i
r
2 2as
π (a + s2 )2
2
.
0

e ri
Solution:
The Fourier transform of f (x) is
i n e
F (s)
E = F [f (x)] = √
1
ng Z∞
f (x)eisx dx

as y 1
Z∞

−∞

w . E =

1 


 ∞
Z
−∞
e−a|x| (cos sx + i sin sx)dx

Z∞

w w = √

1

−∞
 ∞
Z
e−a|x| cos sxdx + i

= √ 2 e−ax cos sxdx + 0


−∞

e−a|x| sin sxdx


0
r
2 a
=
π (a2 + s2 )

The Inverse Fourier transform is

Z∞
1
f (x) = √ F (s)e−isx ds

−∞
Z∞
r
1 2 a
= √ 2 + s2 )
(cos sx − i sin sx)ds
2π π (a
−∞
 ∞
Z∞

Z
1 a a
= cos sxds − i sin sxds
π (a2 + s2 ) (a2 + s2 )
−∞ −∞

Page 14

Downloaded From : www.EasyEngineering.net


Downloaded From : www.EasyEngineering.net Umar Ali

 ∞ 
Z
a 1
= 2 cos sxds + 0
π (a2 + s2 )
0
Z∞
2a cos sx
f (x) = ds
π (a2 + s2 )
0
Z∞
cos sx π
ds = f (x)
(a2 + s2 ) 2a
0
Z∞
cos sx π −a|x|
ds = e
(a2 + s2 ) 2a
0

d
W.K.T F [xf (x)] = −i F [f (x)]
ds
r
−a|x| d 2 a
F [xe ] = −i
ds π (a + s2 )
2

t
r  
2 −1

e
= −ia 2s
π (a2 + s2 )2

= i
r
2 2as
π (a + s2 )2
2

g . n
ri n
10. Find the Fourier Transform of e−|x| .Find the value of F [e−|x| cos 2x].
Solution:

n e e
The Fourier transform of f (x) is

ng
Z∞
i
y E
F (s) = F [f (x)] = √
1

f (x)eisx dx

as
−∞
Z∞
1

w . E = √

1 

 ∞
Z
−∞
e−|x| (cos sx + i sin sx)dx

Z∞

w w = √

1

−∞
 ∞
Z
e−|x| cos sxdx + i

= √ 2 e−x cos sxdx + 0


−∞

e−|x| sin sxdx


0
r
2 1
=
π (1 + s2 )

W.K.T F [f (x) cos ax] = 12 [F (s + a) + F (s − a)], a = 2

r
2 1
F (s + 2) =
π (1 + (s + 2)2 )
r
2 1
F (s − 2) =
π (1 + (s − 2)2 )
"r r #
1 2 1 2 1
F [e−|x| cos 2x] = +
2 π (1 + (s + 2)2 ) π (1 + (s − 2)2 )

Page 15

Downloaded From : www.EasyEngineering.net


Downloaded From : www.EasyEngineering.net Umar Ali

r  
1 2 1 1
= +
2 π s2 + 4s + 5 s2 − 4s + 5
 2
s − 4s + 5 + s2 + 4s + 5

1
= √
2π (s2 + 4s + 5)(s2 − 4s + 5)
2s2 + 10
 
1
= √
2π (s2 + 5)2 − (4s)2
s2 + 5
 
2
= √
2π s4 + 10s2 + 25 − 16s2
r 
s2 + 5

2
=
π s4 − 6s2 + 25


 x, |x| < a;
11. Find the Fourier transform of f (x) =
 0, |x| > a
Solution:
The Fourier transform of f (x) is

F (s) = F [f (x)] = √
1
Z∞
f (x)eisx dx

ne t
1
Za

−∞

n g .
= √

−a
x(cos sx + i sin sx)dx

e ri
e
 
Za Za
1
=

i n
√  x cos sxdx + i x sin sxdx

ng
−a −a
 
Za
1 
= √ 0 + 2i x sin sxdx

y E 2π
0

as
    a
2i − cos sx − sin sx
= √ x − (1)
2π s s2 0

w . E = i
r 

r 
π
2

x cos sx sin sx
s
+
s2 0
a

w

2 a cos as sin as
= i − +
π s s2

w = i
r 

π
2 sin as − as cos as
s2


Page 16

Downloaded From : www.EasyEngineering.net


Downloaded From : www.EasyEngineering.net Umar Ali

Fourier cosine and sine transforms

The Fourier Cosine transform of f (x) is


r Z∞
2
Fc [s] = Fc [f (x)] = f (x) cos sxdx
π
0
The Inverse Fourier Cosine transform is
r Z∞
2
f (x) = F [s] cos sxds
π
0
The Fourier Sine transform of f (x) is
r Z∞
2
Fs [s] = Fs [f (x)] = f (x) sin sxdx
π
0
The Inverse Fourier Sine transform is
r Z∞
2
f (x) = F [s] sin sxds
π
0

ne t
Self Reciprocal

n g .
i
2 x2
x2
12. Find the Fourier Cosine transform of e−a .Hence show that e− is self reciprocal under Fourier Cosine

r
2

e
h x2
i
transform and find Fs xe− 2 .
Solution:

i n e
ng
The Fourier Cosine transform of f (x) is

Fc (s) = Fc [f (x)] =

s y E r
2
π
Z∞
f (x) cos sxdx

a
0
Z∞

.E
r
2 x2 1 2
Fc [e−a ] = f (x) R.p of eisx dx
2 π

w
−∞
Z∞

w w =

=
R.p of √

R.p of √
1

1

−∞
Z∞
f (x)eisx dx

e−a
2 x2
eisx dx

−∞
Z∞ s2 s2
1 2 x2
= R.p of √ e−a eisx e− 4a2 e 4a2 dx

−∞

2 Z∞ 
s2

− s2 1 − a2 x2 −isx−
= R.p of e 4a √ e 4a2 dx

−∞

2 Z∞  2 2

− s2 1 − a2 x2 −isx+ i2 s 2
= R.p of e 4a √ e 2 a dx

−∞

2 Z∞  2 2

− s2 1 − a2 x2 − 2axis + i2s 2
= R.p of e 4a √ e 2a 2 a dx

−∞

Page 17

Downloaded From : www.EasyEngineering.net


Downloaded From : www.EasyEngineering.net Umar Ali

s2
Z∞ 2
1 is
= R.p of e −
4a2 √ e−(ax− 2a ) dx

−∞

is
Put u = ax − ⇒ du = adx
2a
if x = −∞ ⇒ u = −∞
if x = ∞ ⇒u=∞

s2
Z∞
−a2 x2 − 1 2 du
Fc [e ] = R.p of e 4a2 √ e−u
2π a
−∞

2 Z∞
− s2 1 2
= R.p of e 4a √ e−u du
a 2π
−∞
s2 1 √

= R.p of e 4a2 √ π
a 2π
s2
e− 4a2

t
−a2 x2
Fc [e ] = √

1
a 2

. ne
Now,put a = √
2

i n g
Fc [e
−x2
2 ] =
e

s2
4( 12 )

e e r
Fc [e
−x2
2

g i n
] = e− 2
√1
2
s2
2

d
E n
sy
W.k.t Fs [xf (x)] = − [Fc (f (x))]
ds

.E a Fs [xe ] =
x2
2 −
d
ds

e − s2
2


w 2
 
− s2 −2s
= −e

w w = se− 2
s2
2

2 x2
x2
13. Find the Fourier transform of e−a .Hence show that e− 2 is self reciprocal under Fourier transform and find
h x2
i
F xe− 2 .
Solution:
The Fourier transform of f (x) is

Z∞
1
F (s) = √ f (x)eisx dx

−∞
Z∞
1 2 x2
= √ e−a eisx dx

−∞

Page 18

Downloaded From : www.EasyEngineering.net


Downloaded From : www.EasyEngineering.net Umar Ali

Z∞ s2 s2
1 2 x2
= √ e−a eisx e− 4a2 e 4a2 dx

−∞

s2
Z∞ 
s2

− 1 − a2 x2 −isx−
= e 4a2 √ e 4a2 dx

−∞

s2
Z∞  2 2

− 1 − a2 x2 −isx+ i2 s 2
= e 4a2 √ e 2 a

−∞

s2
Z∞  2 2

− 1 − a2 x2 − 2axis + i2s 2
= e 4a2 √ e 2a 2 a dx

−∞

s2
Z∞ 2
1 is
= e− 4a2 √ e−(ax− 2a ) dx

−∞

is
Put u = ax − ⇒ du = adx
2a
if x = −∞
if x = ∞
⇒ u = −∞
⇒u=∞

ne t
F [f (x)] = e− s2
4a2 √
1
Z∞

n
e−u
2 du
g .

−∞
Z∞
e ri a

e
s2 1
− −u2
= e 4a2 √ e du

i n a 2π
−∞

ng
s2 1 √
= e− 4a2 √ π
a 2π

s y E
F [e −a2 x2
] =
e− 4a2

a 2
s2

Now,put a = √
1
2
.E a
w w −x2 e

s2
4( 12 )

w F [e

F [e
2

−x2
2
] =

] = e− 2
√1
2

s2
2

d
W.k.t F [xf (x)] = −i [F (f (x))]
ds

x2 s2
 
d
F [xe 2 ] = −i e− 2
ds
s2
 
−2s
= −ie− 2
2
s2
= ise− 2

x2
14. Show that e− 2 is self reciprocal under Fourier Cosine transform.
Solution:

Page 19

Downloaded From : www.EasyEngineering.net


Downloaded From : www.EasyEngineering.net Umar Ali

The Fourier Cosine transform of f (x) is

r Z∞
2
Fc (s) = Fc [f (x)] = f (x) cos sxdx
π
0
2
r Z∞
− x2 1 2
Fc [e ] = f (x) R.p of eisx dx
2 π
−∞
Z∞
1
= R.p of √ f (x)eisx dx

−∞
Z∞
1 x2
= R.p of √ e− 2 eisx dx

−∞
Z∞
1 x2 s2 s2
= R.p of √ e− 2 eisx e− 2 e 2 dx

−∞

2 Z∞
1 1
e− 2 (x ) dx

t
− s2 2 −2isx−s2
= R.p of e √

− s2
2
1

−∞
Z∞
− 21 (x2 −2isx+i2 s2 )
. ne
= R.p of e √

−∞
e

i n
dx

g
=
s2
R.p of e− 2 √
1

Z∞

e e r1
e− 2 (x−is) dx
2

n
−∞

= R.p of

ng i e− s2
2

1

Z∞

−∞
e


x−is

2
2
dx

y E
as
x − is dx
Put u = √ ⇒ du = √
2 2

w . E
if x = −∞
if x = ∞
⇒ u = −∞
⇒u=∞

w w F [f (x)] = R.p of e− s2
2

1

−∞
Z∞
e−u
2 √
2du

Z∞
s2 1 2
= R.p of e− 2 √ e−u du
π
−∞
2
− s2 1 √
= R.p of e √ π
π
x2 s2
Fc [e− 2 ] = e− 2

x2
15. Show that e− 2 is self reciprocal under Fourier transform.
Solution:

Page 20

Downloaded From : www.EasyEngineering.net


Downloaded From : www.EasyEngineering.net Umar Ali

The Fourier transform of f (x) is

Z∞
1
F (s) = F [f (x)] = √ f (x)eisx dx

−∞
Z∞ 2
1 − x2
= √ e eisx dx

−∞
Z∞
1 x2 s2 s2
= √ e− 2 eisx e− 2 e 2 dx

−∞

2 Z∞
1 1
− s2
e− 2 (x ) dx
2 −2isx−s2
= e √

−∞

2 Z∞
1 1
− s2
e− 2 (x ) dx
2 −2isx+i2 s2
= e √

−∞
Z∞
s2 1

t
1 2
= e− 2 √ e− 2 (x−is) dx

2

1
−∞
Z∞


x−is
2

. ne
g
− s2 √
= e √ e 2 dx

n

i
−∞

x − is dx
e e r
Put u = √
2
⇒ du = √
2

i n
ng
if x = −∞ ⇒ u = −∞
if x = ∞ ⇒u=∞

s y E e − s2
2

1
Z∞
e−u
2 √

a
F [f (x)] = 2du

.E
−∞
Z∞
s2 1 2
e− 2 √ e−u du

w w =

2
− s2 1 √
π
−∞

w x2
F [e− 2 ] = e− 2
= e
s2

π
π

2
16. Find the Fourier Cosine transform of e−x .

1
17. Find the Fourier Cosine and Sine transform of xn−1 .Prove that √ is self reciprocal under Fourier Cosine and
x
Sine transforms.
Solution:
The Fourier Cosine transform of f (x) is

r Z∞
2
Fc (s) = Fc [f (x)] = f (x) cos sxdx
π
0

Page 21

Downloaded From : www.EasyEngineering.net


Downloaded From : www.EasyEngineering.net Umar Ali

r Z∞
n−1 2
Fc [x ] = xn−1 cos sxdx
π
0
r
2 Γn nπ
= cos
π sn 2
1
Put n =
2
2 Γ 21
r
1 π
Fc [x 2 −1 ] = cos
π s 12 4
r √
1 2 π 1
Fc [x− 2 ] = √ √
π s 2
 
1 1
Fc √ = √
x s

The Fourier Cosine transform of f (x) is

r Z∞
2
Fs (s) = Fs [f (x)] = f (x) sin sxdx

r Z∞
π
0

ne t
.
2
Fs [xn−1 ] = xn−1 sin sxdx

g
π
0

=
r
2 Γn
n
sin

ri n
e
πs 2

e
1
Put n =
2

i
r
n 2 Γ 12 π

ng
1
Fs [x 2 −1 ] = sin
π s 12 4
r √
2 π 1

E
1
Fs [x− 2 ] = √ √

y
π s 2

as
 
1 1
Fs √ = √
x s

w . E 

 x, 0 < x < 1;

w


18. Find the Fourier Cosine Transform of f (x) = 2 − x, 1<x<2

w



0, x>2

Solution:
The Fourier Cosine transform of f (x) is

r Z∞
2
Fc (s) = Fc [f (x)] = f (x) cos sxdx
π
0
 1 
r Z Z2
2
= x cos sxdx + (2 − x) cos sxdx
π
0 1
r "    1     2 #
2 sin sx − cos sx sin sx − cos sx
= x − (1) + (2 − x) − (−1)
π s s2 0 s s2 1
r "    1    2 #
2 sin sx cos sx sin sx cos sx
= x + + (2 − x) −
π s s2 0 s s2 1

Page 22

Downloaded From : www.EasyEngineering.net


Downloaded From : www.EasyEngineering.net Umar Ali

r       
2 sin s  cos s  1 cos 2s sin s cos s
= + − 0+ 2 + 0− − − 2
π s s2 s s2 s s
r  
2 2 cos s 1 cos 2s
= − 2−
π s2 s s2
r
2 [2 cos s − (1 + cos 2s)]
=
π s2
r
2 [2 cos s − 2 cos2 s]
=
π s2
r
2 2 cos s
= [1 − cos s]
π s2




 x, 0 < x < 1;

19. Find the Fourier Sine Transform of f (x) = 2 − x, 1<x<2



0, x>2

20. Find the Fourier Cosine and Sine transform of e−ax and Find its inversion.Also find Fc [xe−ax ] and Fs [xe−ax ]
Solution:

ne t
The Fourier Cosine transform of f (x) is

Z∞

n g .
i
r
2
Fc (s) = Fc [f (x)] =

Z∞
π
0

e e r
f (x) cos sxdx

n
r
2
=

ng i
π

2
0

a
e−ax cos sxdx

E
=
π s + a2
2

as y
The Inverse Fourier Cosine transform is

w . E f (x) =
r
2
π
Z∞
Fc [s] cos sxds

w
0
r Z∞ r
2 2 a

w =

=
2a
π
Z∞
0

1
π s2 + a2

cos sxds
cos sxds

π s2 + a2
0
Z∞
cos sx π
ds = f (x)
s2 + a2 2a
0
Z∞
cos sx π −ax
ds = e
s2 + a2 2a
0

The Fourier Sine transform of f (x) is

r Z∞
2
Fs (s) = Fs [f (x)] = f (x) sin sxdx
π
0

Page 23

Downloaded From : www.EasyEngineering.net


Downloaded From : www.EasyEngineering.net Umar Ali

r Z∞
2
= e−ax sin sxdx
π
0
r
2 s
=
π s2 + a2

The Inverse Fourier Cosine transform is

r Z∞
2
f (x) = Fs (s)) sin sxds
π
0
r Z∞ r
2 2 s
= sin sxds
π π s2 + a2
0
Z∞
2 s
= sin sxds
π s2 + a2
0
Z∞
s sin sx π

0
Z∞
s2 + a2
ds =
2
f (x)

ne t
0
s sin sx
s2 + a2
ds =
π −ax
2
e

n g .
W.k.t Fc [xf (x)] =
d
[Fs (f (x))]
e ri
ds

d
i n e
ng
h i
Fc [xe−ax ] = Fs e−ax
ds r

E
d 2 s
=

as y =
ds π s2 + a2
r

π
2 (s2 + a2 )(1) − s(2s)
(s2 + a2 )2

w . E =
r
2 a − s2
2

π (s2 + a2 )2

w w
W.k.t Fs [xf (x)] = −
d
ds
[Fc (f (x))]

d h i
Fs [xe−ax ] = − Fc e−ax
ds r
d 2 a
= −
ds π s2 + a2
r  
2 −1
= −a (2s)
π (s2 + a2 )2
r
2 2as
=
π (s2 + a2 )2

Find the Fourier Cosine and Sine transform of e−x

Page 24

Downloaded From : www.EasyEngineering.net


Downloaded From : www.EasyEngineering.net Umar Ali

Half Range Parseval’s Identity

Formula:

Z∞ Z∞
(i) Fc [f (x)]Fc [g(x)]ds = f (x)g(x)dx
0 0
Z∞ Z∞
2 2
(ii) [Fc [f (x)]] ds = [f (x)] dx
0 0
Z∞ Z∞
(iii) Fs [f (x)]Fs [g(x)]ds = f (x)g(x)dx
0 0
Z∞ Z∞
2 2
(iv) [Fs [f (x)]] ds = [f (x)] dx
0 0

R∞ dx
21. Evaluate
Solution:
0 (x2 + a2 )(x2 + b2 )
using transforms techniques.

ne t
.
r
2 a
W.k.t Fc [e−ax ] =
 −bx 
and Fc e =
r
2
π s2 + a2

i n g
Now,
π s2 + b2

e e r
Z∞

i n Z∞

ng
Fc [f (x)]Fc [g(x)]ds = f (x)g(x)dx
0 0
Z∞ r

E Z∞
r
2 a 2 b

a
0

s y
π s + a2
Z∞
2 π s + b2
2
ds =
0
Z∞
e−ax e−bx dx

.E
2ab 1
ds = e−(a+b)x dx
π (s2 + a2 )(s2 + b2 )

w
0 0
Z∞ ∞
π e−(a+b)x

w

1
ds =
(s2 + a2 )(s2 + b2 ) 2ab −(a + b) 0

w 0

=
π
−2ab(a + b)
π
[0 − 1]

2ab(a + b)
Z∞
dx π
∴ =
(x2 + a2 )(x2 + b2 ) 2ab(a + b)
0

R∞ dx
22. Evaluate using transforms techniques.
0 (x2 + a2 )2
Solution: r
−ax 2 a
W.k.t Fc [e ]=
π s2 + a2

Z∞ Z∞
2 2
Now, [Fc [f (x)]] ds = [f (x)] dx
0 0

Page 25

Downloaded From : www.EasyEngineering.net


Downloaded From : www.EasyEngineering.net Umar Ali

Z∞ r !2 Z∞
2 a 2
ds = e−ax dx
π s2 + a2
0 0
Z∞ Z∞
2a2 1
ds = e−2ax dx
π (s2 + a2 )2
0 0
Z∞ ∞
π e−2ax

1
ds =
(s2 + a2 )2 2a2 −2a 0
0
π
= [0 − 1]
2a2 (−2a)
π
=
4a3
Z∞
dx π
∴ =
(x2 + a2 )2 4a3
0

R∞ x2 dx
23. Evaluate using transforms.
(x2 + a2 )(x2 + b2 )
Solution:
0

2 s
r

ne t
.
−ax
W.k.t Fs [e ]= 2 2
r πs +a
 −bx 
and Fs e =
2 s
π s2 + b2

i n g
Z∞ Z∞

e e r
n
Now, Fs [f (x)]Fs [g(x)]ds = f (x)g(x)dx

Z∞ r
2
0

s
r
2 s
ds =
ng i 0
Z∞
e−ax e−bx dx

E
π s2 + a2 π s2 + b2
0 0

2
π
Z∞

as
(s2y +
s2
a2 )(s2 + b2 )
ds =
Z∞
e−(a+b)x dx

w . E 0
Z∞
s 2

(s + a )(s2 + b2 )
2 2
ds =
0

π e−(a+b)x
 ∞

2 −(a + b) 0

w
0
π
= [0 − 1]

w Z∞
x2 dx
=
−2(a + b)
π
2(a + b)
π
∴ =
(x2 + a2 )(x2 + b2 ) 2(a + b)
0

R∞ x2 dx
24. Evaluate using Parseval’s identity.
0 (x2 + a2 )2
Solution: r
2 s
W.k.t Fs [e−ax ] =
π s2 + a2

Z∞ Z∞
2 2
Now, [Fs [f (x)]] ds = [f (x)] dx
0 0

Page 26

Downloaded From : www.EasyEngineering.net


Downloaded From : www.EasyEngineering.net Umar Ali

Z∞ r !2 Z∞
2 s 2
ds = e−ax dx
π s2 + a2
0 0
Z∞ Z∞
2 s2
ds = e−2ax dx
π (s2 + a2 )2
0 0
Z∞ ∞
s2 π e−2ax

ds =
(s + a2 )2
2 2 −2a 0
0
π
= [0 − 1]
2(−2a)
π
=
4a
Z∞
x2 dx π
∴ =
(x2 + a2 )2 4a
0

R∞ dx
25. Evaluate using transforms techniques.
(x2 + 4)(x2 + 25)

t
0

26. Evaluate
R∞
0
dx
(x2 + 1)2
using transforms Parseval’s identity method.

. ne
27. Evaluate
R∞ λ2 dλ
(λ2 + 1)(λ2 + 4)
using Parseval’s identity.

i n g
r
0

28. Evaluate
R∞
0
x2 dx
(x2 + 1)2
using Parseval’s identity.

n e e
ng i
y E
. E as
w w
w

Page 27

Downloaded From : www.EasyEngineering.net


Downloaded From : www.EasyEngineering.net Umar Ali

Part-A

1. State Fourier integral theorem.


Solution:
Fourier integral theorem
If f(x) is a given function defined in (−l, l) and satisfies the following conditions,

• f(x) is well defined and single valued except at finite number of points in (−l, l)

• f(x) is periodic in (−l, l)

• f(x) and f 0 (x) are piecewise continuous in (−l, l)


R∞
• |f (x)|dx converges.
−∞
1 R∞ R∞
then f (x) = f (t) cos λ(t − x)dtdλ
π 0 −∞

2. State and prove Change of scale property for Fourier Transform.


Statement:

ne t
1 s
If F [f (x)] = F (s) then F [f (ax)] = F
a a
,a > 0

n g .
Proof :
The Fourier transform of f (x) is

e ri
i n√
1
Z∞
e
f (x)eisx dx

g
F (s) = F [f (x)] =

En
−∞
Z∞
1
f (ax)eisx dx

y
F [f (ax)] = √

s

−∞

. E dx =
dta
Put ax = t ⇒ adx = dt If x = −∞ ⇒ t = −∞
If x = ∞ ⇒ t = ∞

w w a

w ∴ F [f (ax)] = √
1

Z∞
f (t)eis( a )
t dt
a
−∞
Z∞
1 1
f (t)ei( a )t dt
s
= √
a 2π
−∞
1 s
= F
a a

3. State and prove the Shifting Property.


If F [f (x)] = F (s) then F [f (x − a)] = eias F (s).
Also F [eiax f (x)] = F (s + a) and F [e−iax f (x)] = F (s − a)
Proof :

Page 28

Downloaded From : www.EasyEngineering.net


Downloaded From : www.EasyEngineering.net Umar Ali

The Fourier transform of f (x) is

Z∞
1
F (s) = F [f (x)] = √ f (x)eisx dx

−∞
Z∞
1
F [f (x − a)] = √ f (x − a)eisx dx

−∞

Put x − a = t If x = −∞ ⇒ t = −∞
⇒ dx = dt If x = ∞ ⇒ t = ∞

Z∞
1
∴ F [f (ax)] = √ f (t)eis(t+a) dt

−∞
Z∞
= √
1

−∞
f (t)eist eias dt

ne t
= e ias
F (s)
Z∞

n g .
i
1
Now F [eiax f (x)] √ eiax f (x)eisx dx

r
=

n √
1
−∞
Z∞

e e
f (x)ei(s+a)x dx

i
=

ng
−∞
= F (s + a)

E
Z∞
1

as y
Now F [e−iax f (x)] = √

e−iax f (x)eisx dx
−∞
Z∞

w . E = √
1

= F (s − a)
−∞
f (x)ei(s−a)x dx

w w
4. If F [f (x)] = F (s) then F [f (x) cos ax]= ?
Proof :
The Fourier transform of f (x) is

Z∞
1
F (s) = F [f (x)] = √ f (x)eisx dx

−∞
Z∞
1
F [f (x) cos ax] = √ f (x) cos axeisx dx

−∞
Z∞
e + e−iax
 iax 
1
= √ f (x) eisx dx
2π 2
−∞
Z∞ Z∞
 
1 1 1
= √ f (x)eiax eisx dx + √ f (x)e−iax eisx dx
2 2π 2π
−∞ −∞

Page 29

Downloaded From : www.EasyEngineering.net


Downloaded From : www.EasyEngineering.net Umar Ali

Z∞ Z∞
 
1 1 1
= √ f (x)ei(s+a)x dx + √ f (x)ei(s−a)x dx
2 2π 2π
−∞ −∞
1
= [F (s + a) + F (s − a)]
2


 eikx , a < x < b;
5. Find the Fourier transform of f (x) =
 0, x ≤ a or x > b
Solution:
The Fourier transform of f (x) is

Z∞
1
F (s) = F [f (x)] = √ f (x)eisx dx

−∞
Zb
1
= √ eikx eisx dx

t
a

= √
1

Zb
ei(s+k)x dx

. ne
g
a

n
 i(s+k)x b
1 e
= √

1
2π i(s + k) a
1 h

e ri
i
= √
2π i(s + k)

i n e
ei(s+k)b − ei(s+k)a

ng
6. Find Fc [xf (x)] and Fs [xf (x)]
Solution:

y E
as
r Z∞
2
W.K.T Fc [f (x)] = f (x) cos sxds

E
π

w . d
ds
Fc [f (x)] =
r
2 d
π ds
0
Z∞
f (x) cos sxds

w w =
r
2
π
Z∞

0
0

f (x)(− sin sx)xds

= −Fs [xf (x)]


d
Fs [xf (x)] = − Fc [f (x)]
ds

r Z∞
2
W.K.T Fs [f (x)] = f (x) sin sxds
π
0
r Z∞
d 2 d
Fs [f (x)] = f (x) sin sxds
ds π ds
0
r Z∞
2
= f (x) cos sxxds
π
0
= Fc [xf (x)]

Page 30

Downloaded From : www.EasyEngineering.net


Downloaded From : www.EasyEngineering.net Umar Ali

d
Fc [xf (x)] = Fs [f (x)]
ds

e−as
7. Find the function f (x) whose sine transform is
s
Solution:
e−as
Given: Fs (s) =
s
The Inverse Fourier Sine transform is

r Z∞
2
f (x) = Fs (s) sin sxds
π
0
r Z∞
2 e−as
= sin sxds
π s
0
Differentiating w.r.t ’x’
r ∞ 
Z −as
d[f (x)] 2 d  e
= sin sxds
dx π dx
Z∞
s
0

ne t
.
r
−as
2 e ∂
= (sin sx)ds
π
0
Z∞
s ∂x

i n g
r
r
2 e−as
cos sx · sds

e
=
π s

=
r
2
0
Z∞

i n
e−as cos sxds e
ng
π
0
r

E
d[f (x)] 2 a
=

as y
dx

f (x) =
π a2 + x2
r Z
2
π
a
a + x2
2
dx

w . E = a
r
r

2
21
πa
tan−1
x
x
a

w w
8. Find the Fourier Sine transform of
1
=
π
tan−1
a

x
Solution:
The Fourier Sine transform is

r Z∞
2
Fs [f (x)] = f (x) sin sxds
π
0
r Z∞
2 1
= sin sxds
π x
0
r Z∞
2 sin sx
= ds
π x
0
r

=
π2

Page 31

Downloaded From : www.EasyEngineering.net


Downloaded From : www.EasyEngineering.net Umar Ali

r
π
=
2

9. State the convolution theorem and Parseval’s identity for Fourier Transfrom.
Solution:
Convolution Theorem:
If F [f (x)] = F (s) and F [g(x)] = G(s) then F [f (x) ∗ g(x)] = F (s) · G(s)
Parseval’s identity:
R∞ R∞
If F [f (x)] = F (s) then |F (s)|2 ds = |f (x)|2 dx
−∞ −∞

Other important questions:


2
1. Verify the convolution theorem under Fourier transform from f (x) = g(x) = e−x .
Solution:
The Convolution of f (x) and g(x) is

ne t
F [f (x) ∗ g(x)] = F (s)G(s)..........(1)

n g .
i
where F [f (x)] = F (s) and F [g(x)] = G(s)

To find F (s)G(s)
2
Given f (x) = e−x and g(x) = e−x
2

e e r
2 2 1 s2
F [e−a x ] = √ e− 4a2
i n
ng
W.K.T,
a 2
Put a = 1
2

E 1 s2
F [e−x ] = √ e− 4 = F (s)

y
as
2
−x2 1 − s2
Similarly F [e ] = √ e 4 = G(s)
2

Now

w . E 1
2
s2 1
F (s)G(s) = √ e− 4 √ e− 4
2
s2

w w
To find F [f (x) ∗ g(x)]
1 s2
F (s)G(s) = e− 2 ........(2)
2

Z∞
1
f (x) ∗ g(x) = √ f (t)g(x − t)dt

−∞
Z∞
1 2 2
= √ e−t e−(x−t) dt

−∞
Z∞
1 2
+(x−t)2 )
= √ e−(t dt

−∞
Z∞
1 2
+x2 −2xt+t2 )
= √ e−(t dt

−∞
Z∞
1 2
−2xt+x2 )
= √ e−(2t dt

−∞

Page 32

Downloaded From : www.EasyEngineering.net


Downloaded From : www.EasyEngineering.net Umar Ali

Z∞
1 2
−2xt+x2 ) − x2
2 x2
= √ e−(2t e e 2 dt

−∞
x2 Z

e− 2 2
−2xt+x2 − x2 )
2
= √ e−(2t dt

−∞
x2 Z∞
e− 2 x2 x2
 
−2 t2 −2 x
2 t+ 2 − 4
= √ e dt

−∞
x2 Z∞
e− 2 x2
 
−2 t2 −2 x
2 t+ 4
= √ e dt

−∞
x2 Z∞
e− 2 2
e−2(t− 2 ) dt
x
= √

−∞

√ x
 √
Put u = 2 t− 2 ⇒ du = 2dt
if t = −∞ ⇒ u = −∞
if t = ∞ ⇒u=∞

ne t
f (x) ∗ g(x) =
− x2
e

2 Z∞
e−u √
2 du

n g .

−∞
Z∞
2

e ri
e
2
− x2
e 2
= √
2 π
i n e−u du

ng
−∞
2
− x2√
e

y E = √
2 π
π

as
x2
e− 2
=

w . E Now F [f (x) ∗ g(x)] = F


2
"
e− 2
2
x2
#

w w =

=
1
2
h
F e− 2
1 − s2
2
x2
i

e 2 ...........(3)

From (2) and (3),


F [f (x) ∗ g(x)] = F (s)G(s)
Hence Convolution theorem is verified.

Note:
The Convolution of two functions f (x) and g(x) is
1 R∞
f (x) ∗ g(x) = √ f (t)g(x − t)dt
2π −∞
2. State and prove the Convolution and Parseval’s identity Theorem.
Convolution Theorem:
If F [f (x)] = F (s) and F [g(x)] = G(s) then F [f (x) ∗ g(x)] = F (s) · G(s)

Page 33

Downloaded From : www.EasyEngineering.net


Downloaded From : www.EasyEngineering.net Umar Ali

Proof:

Z∞
1
F [f (x) ∗ g(x)] = √ f (x) ∗ g(x)eisx dx

−∞
Z∞ Z∞
 
1  √1
= √ f (t)g(x − t)dt eisx dx
2π 2π
−∞ −∞
Z∞ Z∞
 
1 1
= √ √ g(x − t)eisx dx f (t)dt
2π 2π
−∞ −∞
Z∞
1
= √ F [f (x − t)]f (t)dt

−∞
Z∞
1
= √ F [f (x)]eist f (t)dt

−∞
= F [f (x)]F [g(x)]

= F (s)G(s)

ne t
Parseval’s identity:

n g .
If F [f (x)] = F (s) then
R∞
−∞
|F (s)|2 ds =
R∞
−∞
|f (x)|2 dx

e ri
e
Proof:

i n
ng
By Convolution Theorem

F [f (x) ∗ g(x)]

E
= F (s)G(s)

a 1
s y Z∞
f (x) ∗ g(x) = F −1 [F (s)G(s)]
1
Z∞
F (s)G(s)e−isx ds

.E
√ f (t)g(x − t)dt = √
2π 2π
−∞ −∞

w w Z∞
Put x = 0

f (t)g(−t)dt =
Z∞
F (s)G(s)ds

w −∞

Put g(−t) = f (t) and G(s) = F (s)


Z∞
−∞

Z∞
f (t)f (t)dt = F (s)F (s)ds
−∞ −∞
Z∞ Z∞
|f (t)|2 dt = |F (s)|2 ds
−∞ −∞

Z∞ Z∞
2
⇒ |F (s)| ds = |f (x)|2 dx, Put t = x
−∞ −∞

Page 34

Downloaded From : www.EasyEngineering.net


Downloaded From : www.EasyEngineering.net Umar Ali




 0, for x < 0
1

3. Find the Fourier integral representation of f (x) defined as f (x) = for x = 0


 2
 −x
e for x > 0
Verify the representation directly at the point x = 0.
Solution:
The Fourier transform is

Z∞
1
F (s) = F [f (x)] = √ f (x)eisx dx

−∞
Z∞
1
= √ e−x (cos sx + i sin sx)dx

0
∞
Z∞

Z
1  e−x cos sxdx + i e−x sin sxdx
= √

0 0

t
 
1 1 s
=

=


2π s2 + 1
1 1 + is
+i 2
s +1

. ne
2π s2 + 1

i n g
The Inverse Fourier transform is

Z∞
e e r
f (x) = √
1

i n
F (s)e−isx ds

ng

−∞
Z∞
1 1 1 + is

s y E
= √

Z∞
−∞

2π s2 + 1
(cos sx − i sin sx)ds

.E a =
2

0
cos sx + s sin sx
s2 + 1
ds

w
Z∞
1 cos sx + s sin sx
= ds

w w
Verification:
π
0
s2 + 1

Put x = 0,

Z∞
1 1
f (0) = ds
π s2 +1
0
1
= [tan−1 x]∞
0
π

=
π2
1
f (0) =
2

Page 35

Downloaded From : www.EasyEngineering.net

Das könnte Ihnen auch gefallen